You are on page 1of 134

Li ni u

Trong nhng nm gn y, s pht trin ca Tin hc lm thay i nhiu ngnh


truyn thng ca L thuyt s (trong cun sch ny, chng ta thng dng t S
hc). Nu nh trc thp k 70, s hc vn c xem l mt trong nhng ngnh l
thuyt xa ri thc tin nht, th ngy nay, nhiu thnh tu mi nht ca s hc c
ng dng trc tip vo cc vn ca i sng, nh thng tin, mt m, k thut my
tnh. Mt phng hng mi ca s hc ra i v pht trin mnh m: s hc thut
ton. C th ni, l chic cu ni gia s hc vi tin hc. Vi vic s dng rng
ri my tnh trong nghin cu s hc, nhiu ngi cho rng, s hc ngy nay
thnh mt khoa hc thc nghim! iu th hin kh r trong nhng thut ton
xc sut c cp n trong cun sch ny.

V
V
n
n
M
M
a
a
tth
h
..C
C
o
o
m
m

Mc ch ca cun sch nh ny l cung cp cho ngi c mt s kin thc s b


v s hc thut ton. Cun sch khng i hi ngi c mt kin thc chun b
no v l thuyt s. V th cng c th gi n l Nhp mn thut ton vo s hc.
iu c ngha l, trong nhiu con ng khc nhau i vo s hc, ta chn con
ng thut ton: cc nh l, khi nim ca s hc c trnh by cng vi cc thut
ton xy dng chng. Trong nhiu trng hp, cc thut ton c km theo nh gi
s b v phc tp.
Cun sch nhm mt s i tng kh rng ri: nhng sinh vin, nghin cu sinh v
s hc v tin hc, nhng ngi quan tm n l thuyt v ng dng ca s hc hin
i. Nhiu phn ca cun sch c th c ch cho hc sinh cc lp chuyn ton v
chuyn tin hc.
Chng u tin ca cun sch c dnh gii thiu vi nh ngha c bn nht
ca l thuyt thut ton. Ba chng tip theo trnh by nhng vn c s ca s
hc. Chng 5, ngoi vic chun b kin thc cho nhng phn tip theo, c bnh lun
t nhiu v vai tr ca s tng t gia s v a thc trong s pht trin ca s hc
hin i.
ngi c c th hnh dung phn no cc ng dng ca s hc thut ton, cun
sch dnh chng 6 ni v l thuyt mt m. Mt vi ng dng gn y ca l
thuyt ng cong elliptic vo mt m c trnh by trong chng 7. Cng c th
xem Chng 7 l mt nhp mn ngn v s cp vo l thuyt ng cong elliptic,
mt trong nhng l thuyt phong ph nht ca Hnh hc i s s hc.
Cui mi chng u c mt s bi tp dnh cho c gi mun c cun sch mt
cch tch cc. Mt s bi tp mang tnh cht luyn tp v tnh ton thc hnh, mt
s khc l m rng l thuyt. Tr chng cui v ng cong elliptic, cc chng
cn li u c km theo hng dn thc hnh tnh ton bng chng trnh MAPLE.
Phn hng dn thc hnh ny do T Th Hoi An bin son. Cui cun sch c
phn t kim tra kin thc dnh cho nhng c gi hc gio trnh ny vi s tr gip
ca my tnh.
Do nhiu nguyn nhn khc nhau, cun sch chc chn cn rt nhiu thiu st. Tc
gi hy vng nhn c nhng li ph bnh ca bn c.
H ni, 1998
H Huy Khoi

Chng 1.

thut ton
1. nh ngha.

V
V
n
n
M
M
a
a
tth
h
..C
C
o
o
m
m

C th nh ngha thut ton theo nhiu cch khc nhau. y chng ti khng c
nh trnh by cht ch v thut ton nh trong mt gio trnh logic, m s hiu khi
nim thut ton theo mt cch thng thng nht.
Thut ton l mt qui tc , vi nhng d liu ban u cho, tm c li gii sau
mt khong thi gian hu hn.
minh ho cch ghi mt thut ton, cng nh tm hiu cc yu cu ra cho thut
ton, ta xt trn cc v d c th sau y.
Cho n s X[1], X[2],..., X[n], ta cn tm m v j sao cho m=X[j] = max X[k], v j l
1 k n

ln nht c th. iu c ngha l cn tm cc i ca cc s cho, v ch s ln


nht trong cc s t cc i.
Vi mc tiu tm s cc i vi ch s ln nht, ta xut pht t gi tr X[n]. Bc th
nht, v mi ch c mt s, ta c th tm thi xem m=X[n] v j=n. Tip theo , ta so
snh X[n] vi X[n-1]. Trong trng hp n-1=0, tc n=1, thut ton kt thc.
Nu X[n-1] X[n] , ta chuyn sang so snh X[n] vi X[n-2] .Trong trng hp ngc
li, X[n-1] chnh l s cc i trong hai s xt, v ta phi thay i m v j: t m=
X[n-1], j=n-1. Vi cch lm nh trn, mi bc, ta lun nhn c s cc i
trong nhng s xt. Bc tip theo l so snh n vi nhng s ng trc, hoc
kt thc thut ton trong trng hp khng cn s no ng trc n.
Thut ton m t trn y c ghi li nh sau:
Thut ton tm cc i.

M1. [Bc xut pht ] t jn,

kn-1, m X[n].

M2. [ kim tra xong?] Nu k=0, thut ton kt thc.


M3. [So snh] Nu X[k]m,chuyn sang M5.
M4. [Thay i m]
l cc i)

t j k, m X[k]. (Tm thi m ang

M5. [Gim k] t k k-1, quay v M2.


Du dng ch mt php ton quan trng l php thay ch (replacement).

Trn y ta ghi mt thut ton bng ngn ng thng thng. Trong trng hp thut
ton c vit bng ngn ng ca my tnh, ta c mt chng trnh.
Trong thut ton c nhng s liu ban u, c cho trc khi thut ton bt u
lm vic: cc u vo (input). Trong thut ton M, u vo l cc s X[1], X[2],...,
X[n].
Mt thut ton c th c mt hoc nhiu u ra (ouput). Trong thut ton M, cc
u ra l m v j.
C th thy rng thut ton va m t tho mn cc yu cu ca mt thut ton ni
chung, l:

V
V
n
n
M
M
a
a
tth
h
..C
C
o
o
m
m

1. Tnh hu hn.Thut ton cn phi kt thc sau mt s hu hn bc. Khi thut


ton ngng lm vic, ta phi thu c cu tr li cho vn t ra. Thut ton M r
rng tho mn iu kin ny, v mi bc, ta lun chuyn t vic xt mt s sang
s ng trc n, v s cc s l hu hn.
2. Tnh xc nh. mi bc, thut ton cn phi xc nh, ngha l ch r vic cn
lm. Nu i vi ngi c, thut ton M cha tho mn iu kin ny th l li
ca ngi vit!
Ngoi nhng yu t k trn, ta cn phi xt n tnh hiu qu ca thut ton. C rt
nhiu thut ton, v mt l thuyt l kt thc sau hu hn bc, tuy nhin thi gian
hu hn vt qu kh nng lm vic ca chng ta. Nhng thut ton s
khng c xt n y, v chng ta ch quan tm nhng thut ton c th s dng
tht s trn my tnh.
Cng do mc tiu ni trn, ta cn phi ch n phc tp ca cc thut ton.
phc tp ca mt thut ton c th o bng khng gian, tc l dung lng b nh ca
my tnh cn thit thc hin thut ton, v bng thi gian, tc l thi gian my
tnh lm vic. Trong cun sch ny, khi ni n phc tp ca thut ton, ta lun
hiu l phc tp thi gian.

2. phc tp thut ton.

D nhin, thi gian lm vic ca my tnh khi chy mt thut ton no khng ch
ph thuc vo thut ton, m cn ph thuc vo my tnh c s dng. V th,
c mt tiu chun chung, ta s o phc tp ca mt thut ton bng s cc php
tnh phi lm khi thc hin thut ton. Khi tin hnh cng mt thut ton, s cc
php tnh phi thc hin cn ph thuc vo c ca bi ton, tc l ln ca u
vo. V th, phc tp ca thut ton s l mt hm s ca ln ca u vo.
Trong nhng ng dng thc tin, chng ta khng cn bit chnh xc hm ny, m ch
cn bit c ca chng, tc l cn c mt c lng tt ca chng.
Khi lm vic, my tnh thng ghi cc ch s bng nhng bng n sng, tt:
bng n sng ch s 1, bng n tt ch s 0. V th thun tin nht l dng h m
c s 2, trong biu din mt s, ta ch cn dng hai k hiu 0 v 1. Mt k hiu
0 hoc 1 c gi l mt bit (vit tt ca ch binary digit). Mt s nguyn n biu
din bi k ch s 1 v 0 dc gi l mt s k-bit. Trong chng tip theo, ta s thy
rng, s t nhin n s l mt s k-bit vi k=[log2n] ( du[ ] k hiu phn nguyn ca
mt s).
2

phc tp ca mt thut ton c o bng s cc php tnh bit. Php tnh bit l
mt php tnh logic hay s hc thc hin trn cc s 1-bit 0 v 1.
c lng phc tp ca thut ton, ta dng khi nim bc O-ln.
nh ngha 1.1: Gi s f(n) v g(n) l hai hm xc nh trn tp hp cc s nguyn
dng. Ta ni f(n) c bc O-ln ca g(n), v vit f(n)=O(g(n)) hoc f=O(g), nu tn
ti mt s C >0 sao cho vi n ln, cc hm f(n) v g(n) u dng, ng thi
f(n) < Cg(n).
V d. 1) Gi s f(n) l a thc;
f(n)=adnd + ad-1nd-1 + ...+a1n+a0,

V
V
n
n
M
M
a
a
tth
h
..C
C
o
o
m
m

trong ad > 0. D chng minh rng f(n)=O(nd).


2) Nu f1(n)=O(g(n)), f2(n)=O(g(n)) th f1+f2=O(g).
3) Nu f1=O(g1), f2=O(g2), th f1.f2=O(g1.g2).
4) Nu tn ti gii hn hu hn

lim

f ( n)
g ( n)

th f=O(g).

5)Vi mi s >0, log n=O(n ).

nh ngha 1.2. Mt thut ton c gi l c phc tp a thc, hoc c thi


gian a thc, nu s cc php tnh cn thit khi thc hin thut ton khng vt qu
O (logd n), trong n l ln ca u vo, v d l s nguyn dng no .
Ni cch khc, nu u vo l cc s k-bit th thi gian thc hin thut ton l O(kd),
tc l tng ng vi mt a thc ca k.
Cc thut ton vi thi gian O(n ), >0, c gi l cc thut ton vi phc
tp m, hoc thi gian m.
Ch rng, nu mt thut ton no c phc tp O(g), th cng c th ni n
phc tp O(h) vi mi hm h > g. Tuy nhin, ta lun lun c gng tm c lng tt
nht c th c trnh hiu sai v phc tp thc s ca thut ton.
Cng c nhng thut ton c phc tp trung gian gia a thc v m. Ta thng
gi l thut ton di m. Chng hn, thut ton nhanh nht c bit hin nay
phn tch mt s nguyn n ra tha s l thut ton c phc tp
exp( log n log log n ).
Khi gii mt bi ton, khng nhng ta ch c gng tm ra mt thut ton no , m
cn mun tm ra thut ton tt nht. nh gi phc tp l mt trong nhng
cch phn tch, so snh v tm ra thut ton ti u. Tuy nhin, phc tp khng
phi l tiu chun duy nht nh gi thut ton. C nhng thut ton, v l thuyt
th c phc tp cao hn mt thut ton khc, nhng khi s dng li c kt qu

(gn ng) nhanh hn nhiu. iu ny cn tu thuc nhng bi ton c th, nhng


mc tiu c th, v c kinh nghim ca ngi s dng.
Chng ta cn lu thm mt im sau y. Mc d nh ngha thut ton m chng
ta a ra cha phi l cht ch, n vn qu cng nhc trong nhng ng dng thc
t! Bi vy, chng ta cn cn n cc thut ton xc sut , tc l cc thut ton
ph thuc vo mt hay nhiu tham s ngu nhin. Nhng thut ton ny, v
nguyn tc khng c gi l thut ton, v chng c th, vi xc sut rt b, khng
bao gi kt thc. Tuy nhin, thc nghim ch ra rng, cc thut ton xc sut thng
hu hiu hn cc thut ton khng xc sut. Thm ch, trong rt nhiu trng hp,
ch c cc thut ton nh th l s dng c.

V
V
n
n
M
M
a
a
tth
h
..C
C
o
o
m
m

Khi lm vic vi cc thut ton xc sut, ta thng hay phi s dng cc s ngu
nhin. Khi nim chn s ngu nhin cng cn c chnh xc ho. Thng th
ngi ta s dng mt my sn xut s gi ngu nhin no . Tuy nhin, trong
cun sch ny, chng ti khng cp n vn ni trn, m mi ln ni n vic
chn s ngu nhin, ta s hiu l iu thc hin c trn my.
Cng cn lu ngay rng, i vi cc thut ton xc sut, khng th ni n thi
gian tuyt i, m ch c th ni n thi gian hy vng (expected ).
hnh dung c phn no phc tp ca cc thut ton khi lm vic vi
nhng s ln, ta xem bng di y cho khong thi gian cn thit phn tch mt
s nguyn n ra tha s bng thut ton nhanh nht c bit hin nay (ta xem my
tnh s dng vo vic ny c tc 1 triu php tnh trong 1 giy)

S ch s thp phn

S php tnh bit

Thi gian

50

1,4.1010

3,9 gi

75

9,0.1012

104 ngy

100

2,3.1015

74 nm

200

1,2.1023

3,8.109 nm

300

1,5.1029

4,9.1015 nm

500

1,3.1039

4,2.1025 nm

T bng trn y, ta thy rng, ngay vi mt thut ton di m, thi gian lm vic
vi cc s nguyn ln l qu lu. V th ni chung ngi ta lun c gng tm nhng
thut ton a thc.
L thuyt v phc tp thut ton l mt l thuyt rt phong ph. Trong cun sch
ny, chng ti khng ly mc tiu trnh by l thuyt lm trng tm. c gi quan
tm n l thuyt thut ton c th tm c cc sch trong phn Ti liu tham kho.

Chng 2.

S nguyn
1. Biu din s nguyn v cc php tnh s hc
1.1 H c s.

V
V
n
n
M
M
a
a
tth
h
..C
C
o
o
m
m

Mc d hu hu ht c gi quen thuc vi cch biu din s nguyn trong c s


tu , chng ti nhc li s qua vn phn ny, thun tin cho vic trnh
by cc thut ton v s nguyn.
nh l 2.1. Gi s b l mt s nguyn ln hn 1. Khi mi s nguyn n c th vit
duy nht di dng
n=akbk + ak-1bk-1 +...+ a1b1 +a0,

trong aj l s nguyn, 0 aj k-1, vi j=0,1,...,k v h s u tin ak 0.


Chng minh. Ta ch cn thc hin lin tip php chia n cho b:
n=bq0 +a0, 0 a0 b-1.

Nu q0 >b, ta tip tc chia q0 cho b c

q0=bq1 +a1, 0 a1 b-1.

Tip tc qu trnh , ta c:

q1=bq2 +a2, 0 a2 b-1

q2=bq3 +a3, 0 a3 b-1


... ... ...

qk-1=b.0 +ak, 0 ak b-1.

Qu trnh kt thc v ta lun c: n>q0>q1>... 0.

Chng ti dnh cho c gi vic chng minh n c dng nh trong pht biu ca nh
l, v biu din l duy nht.
S b ni trong nh l c gi l c s ca biu din. Cc h biu din c s 10 v 2
tng ng c gi l h thp phn v nh phn. Cc h s aj c gi l cc ch s.
V sau ta dng bit ch ch s nh phn.
Nu s nguyn n biu din trong c s b c k ch s, th t chng minh trn, ta c :
bk-1 n bk.
Nh vy s ch s ca n c tnh theo cng thc:

k=[logb n]+1=[log n / log b]+1,


trong , k hiu log dng ch logarit c s e. Trong c s tu , ta c: k=O(logn).
phn bit cc biu din ca s nguyn trong nhng h c s khc nhau, ta thng
dng cch vit (akak-1...a1a0)b ch s n= akbk + ak-1bk-1 +...+ a1b1 +a0,
V d. 1). i vi s 1994 trong h thp phn, ta c (1994)10=(11111001010)2.

V
V
n
n
M
M
a
a
tth
h
..C
C
o
o
m
m

2). Trong my tnh, bn cnh h c s 2, ngi ta cng thng dng h c s


8 hoc 16. L do ch yu l v vic chuyn mt s vit c s ny sang c s kia
trong 3 c s c thc hin mt cch d dng. V d, mun chuyn mt s cho
trong c s 2 sang c s 8, ta ch vic nhm t phi sang tri tng khi 3 ch s, ri
chuyn s c vit trong khi sang dng thp phn. Chng hn, s
(1110010100110)2 c tch thnh cc nhm 1,110,010,100,110. T ta c:
(1110010100110)2=(16246)8.

Ta c th lm tng t chuyn s cho thnh s vit trong c s 16, ch cn


nhm thnh tng b 4 ch s. Ch rng, trong trng hp ny, cn thm vo cc k
hiu mi ch cc ch s t 10 n 15.
Ta nhc li rng my tnh s dng cch vit nh phn, hoc l cc bit. My tnh
no cng c gii hn v ln ca cc s c th a vo tnh ton. Gii hn
c gi l c t ca my, k hiu qua w. C t thng l mt lu tha ca 2, chng
hn 235.
thc hin cc php tnh s hc vi nhng s nguyn ln hn c t, ta lm nh
sau. Mun a mt s n > w vo my, ta vit n di dng c s w, v khi n c
biu din bng nhng s khng vt qu c t. V d, nu c t ca my l 235, th
a vo mt s c ln c 2350-1, ta ch cn dng 10 s nh nh hn c t ca
my, bng cch biu din n trong c s 235. Nh ni trong v d 1, vic chuyn
mt s t c s 2 sang c s 235 c thc hin d dng bng cch nhm tng khi
35 ch s.
T qui tc ca cc php tnh s hc, ta thy rng:

1) cng hoc tr hai s nguyn k bit, ta cn O(k) php tnh bit.

2) nhn hoc chia hai s k bit theo qui tc thng thng, ta cn O(k2) php tnh
bit.
Trong nhng thp k gn y, ngi ta tm ra nhng thut ton nhn vi phc tp
b hn nhiu so vi cch nhn thng thng. iu th v l, nu thot nhn th cc
thut ton phc tp hn quy tc nhn thng thng. Tuy nhin, khi lm vic vi
nhng s rt ln, cc thut ton ny cho php thc hin vic nhn hai s vi mt
thi gian b hn hn so vi quy tc thng thng.

1.2 Thut ton nhn nhanh hai s.


Ta s dng tnh cht ht sc n gin ca php nhn: nu a=a1+a2, b=b1+b2, th
ab=a1b1+a2b2+a2b1+a1b2. iu ng ch y l, thay cho vic nhn hai s
nguyn n bit, ta thc hin vic nhn cc s c ch s nh hn, cng vi mt s php

cng (i hi s php tnh bit t hn l php nhn). Thc ra iu ny khng c g


mi: ngay trong quan nim ban u, php nhn a vi b l php cng b ln s a!.
Tuy nhin c mt thut ton nhn nhanh, ta khng th cng b ln s a, m phi
tm c mt cch ti u no tch b v a thnh nhng phn nh hn. Nhng
thut ton trnh by di y cho chng ta mt s cch lm vic phn chia nh
vy.
Gi s mun nhn hai s nguyn 2n bit,
a=(a2n-1a2n-2...a1a0)2,
b=(b2n-1b2n-2...b1b0)2.

V
V
n
n
M
M
a
a
tth
h
..C
C
o
o
m
m

Ta vit a=2nA1+A0, b=2nB1+B0, trong


A1=(a2n-1a2n-2...an)2, A0=(an-1an-2...a1a0)2,
B1=(b2n-1b2n-2...bn)2, B0=(bn-1bn-2...b1b0)2.

Khi ta c:

ab=(22n+2n)A1B1+2n(A1 - A0)+(2n+1)A0B0.

(1.1)

Nh vy, vic nhn hai s a,b 2n bit c a v vic nhn cc s n bit, cng vi
cc php cng, tr v dch chuyn (nhn mt s vi mt lu tha bc n ca 2 c
thc hin bng cch dch s sang tri n v tr).
nh l 2.2. Thut ton 2.1 c phc tp l O(nlog23).

Chng minh. Gi M(n) l s cc php tnh bit ti a cn thit khi thc hin nhn hai
s nguyn n bit bng thut ton 2.1. T cng thc (1.1) ta c:
M(2n) 3M(n)+Cn,

trong C l mt hng s khng ph thuc n. t c=max(C,M(2)).


Bng quy np, d chng minh c rng

M(2k) c(3k-2k).

T ta c

M(n)=M(2log2n) M(2[log2n]+1) c(3[log2n]+1-2[log2n]+1) 3c.3[log2n]

3c.3log2n=3cnlog23.

nh l c chng minh.

Vi thut ton 2.1, ta thy rng, ngay ch vi cch phn chia n gin s nguyn
thnh hai phn vi s ch s bng nhau, ta nhn c mt thut ton gim ng
k thi gian thc hin php nhn. D nhin, cch phn chia nh vy cn xa vi cch
phn chia ti u.
Ta s chng t rng cch phn chia nh trn c th tng qut ho nhn c
nhng thut ton nhn vi phc tp nh hn nhiu.

Cng nh trc y, ta s k hiu qua M(n) s cc php tnh bit cn thit thc hin
php nhn hai s nguyn n bit. Trc tin, ta chng minh cng thc sau: vi mi s
t nhin n, tn ti thut ton sao cho:
M((r+1)n) (2r+1)M(n)+Cn,

(1.2)

vi C l mt hng s no . Nh vy, nh l 2.2 l trng hp ring vi r=1.


Gi s cn nhn hai s (r+1)n bit:
a=(a(r+1)n-1...a1a0)2,
b=(b(r+1)n-1...b1b0)2.
Ta tch mi s a,b thnh r+1 s hng:

V
V
n
n
M
M
a
a
tth
h
..C
C
o
o
m
m

a=Ar2rn+...+A12n+A0

b=Br2rn+...+B12n+B0,

trong Aj,Bj l cc s n bit.

Ta nhn xt rng, vic biu din mt s nguyn di dng c s no cng gn


ging nh vit s di dng a thc, trong cc ch s chnh l cc h s ca
a thc. V vy vic nhn hai s c th thc hin tng t nh vic nhn a thc. Ta
xt cc a thc sau:
A(x)=Arxr+...+A1x+A0,
B(x)=Brxr+...+B1x+B0,

W(x)=A(x)B(x)=W2rx2r+...+W1x+W0.

T nh ngha cc a thc trn ta c: a=A(2n),b=B(2n), ab= W(2n). Nh vy, ta d


dng tnh c tch ab nu bit c cc h s ca a thc W(x).
Cng thc (1.2) s c chng minh nu ta tm c mt thut ton tnh cc h s
ca W(x) m ch s dng 2r+1 php nhn cc s n bit v mt s php tnh khc vi
phc tp O(n). iu c th lm bng cch tnh gi tr ca a thc W(x) ti
2r+1 im sau y:
W(0)=A(0)B(0), W(1)=A(1)B(1),..., W(2r)=A(2r)B(2r).

Ch rng, cc s Aj,Bj khng nht thit l cc s n bit, nhng vi r c nh, chng


c s ch s nhiu nht l r+t, vi mt t c nh no . D thy rng, c th nhn
hai s (r+t)-bit vi khng qu M(n)+c1n php tnh bit, trong c1 l hng s (ch
cn tch s (n+t)-bit thnh hai phn n-bit v t-bit, v nhn xt rng, khi t c nh,
vic nhn s t-bit vi s n-bit i hi khng qu cn php tnh bit).
Khi c cc gi tr W(j),(j=0,1,...2r), ta tm c a thc W(x) theo cng thc
Lagrange:
2r

W(x)= ( 1) j W(j)
j=0

x(x - 1)...(x - j + 1)(x - j - 1)...(x - 2r)


.
j!(2r - j)!

Nh vy, cc h s ca W(x) s l t hp tuyn tnh (vi h s khng ph thuc n)


ca cc gi tr W(j), v do , tnh oc bng O(n) php tnh bit.
Ta chng minh c cng thc sau:
M((r+1)n) (2r+1)M(n)+Cn.
Lp lun tng t nh trong chng minh nh l 2.1 ta c:
M(n) C3nlogr+1(2r+1)<C3n1+logr+12.
Vi mi >0 b tu , khi cc tha s c s ch s rt ln, ta c th chn r ln
sao cho logr+12< . Ta c nh l sau:

V
V
n
n
M
M
a
a
tth
h
..C
C
o
o
m
m

nh l 2.3. Vi mi >0, tn ti thut ton nhn sao cho s php tnh bit M(n) cn
thit nhn hai s n bit tho mn bt ng thc
M(n)<C( )n1+ ,

vi hng s C( ) no c lp vi n.

Nhn xt. C th chng minh c rng, vi cch chn r tt, ta c thut ton
nhn hai s n-bit sao cho
M(n)=O(nlog2nloglog2n).

Chng minh nh l khng kh, nhng kh di (xem [Kr]).

2. S nguyn t.

nh ngha 2.4. S nguyn t l s nguyn ln hn 1, khng chia ht cho s nguyn


dng no ngoi 1 v chnh n. S nguyn ln hn 1 khng phi l s nguyn t
c gi l hp s.
D chng minh c rng, s cc s nguyn t l v hn (Bi tp 2.14).

Nh ta s thy trong nhng chng tip theo, bi ton xc nh mt s cho trc c


phi l s nguyn t hay khng c nhiu ng dng trong thc tin. i vi nhng s
nh, bi ton d nhin khng c g kh. Tuy nhin, khi lm vic vi nhng s ln,
ta cn phi tm ra nhng thut ton hu hiu, ngha l c th thc hin c trn my
tnh trong mt khong thi gian chp nhn c. Khi ni n nhng s ln, ta
thng hiu l nhng s nguyn dng c khong 100 ch s thp phn tr ln.
c th tm ra nhng thut ton xc nh nhanh mt s c phi l s nguyn t hay
khng, ta cn hiu su sc tnh cht cc s nguyn t. Trong chng ny, ta ch i
vo cc tnh cht c bn nht.
nh l sau y cho mt thut ton n gin xc nh cc s nguyn t.
nh l 2.5. Mi hp s n u c c nguyn t nh hn

n.

Tht vy, v n l mt hp s nn ta c th vit n=ab, trong a v b l cc s


nguyn vi 1<a b<n. R rng ta phi c a hoc b khng vt qu n , gi s l
a. c nguyn t ca a cng ng thi l c nguyn t ca n.
T nh l trn, ta c thut ton sau y tm ra cc s nguyn t nh hn hoc
bng s n cho trc.

V
V
n
n
M
M
a
a
tth
h
..C
C
o
o
m
m

Sng Eratosthenes. Trc tin, ta vit dy cc s t nhin t 1 n n. Trong dy


gch i s 1, v n khng phi l s nguyn t. S nguyn t u tin ca dy l 2.
Tip theo ta gch khi dy s tt c nhng s chia ht cho 2. S u tin khng
chia ht cho 2 l 3: chnh l s nguyn t. Ta li gch khi dy cn li nhng s
no chia ht cho 3. Tip tc nh th, ta gch khi dy nhng s chia ht cho mi s
nguyn t b hn n . Theo nh l trn, nhng s cn li ca dy l tt c cc s
nguyn t khng vt qu n. Tht vy, cc hp s khng vt qu n, theo nh l
trn, u phi c c nguyn t nh hn n , v do b gch khi dy s trong
mt bc no ca thut ton.
Sng Eratosthenes, mc d cho ta thut ton xc nh mi s nguyn t khng vt
qu mt s cho trc, rt t c s dng xc nh xem mt s cho c phi l
s nguyn t hay khng. Nguyn nhn l v thut ton c phc tp qu ln:
kim tra n, ta phi thc hin php chia cho tt c cc s nguyn t khng vt qu
n.
Ta hy xt s qua v phc tp ca thut ton ni trn. Vi mi s thc dng x
cho trc ta k hiu (x) s cc s nguyn t khng vt qu x. Khi , theo nh l
Hadamard-Vale-Poussin ta c:
lim ( x ) /
x

x
= 1.
log x

Nh vy, s cc s nguyn t khng vt qu


n l vo khong
n /log n =2 n /logn. chia n cho m, ta cn O(log2n. log2m) php tnh bit. Nh
vy, s cc php tnh bit cn thit kim tra n c phi l s nguyn t hay khng t
nht l (2 n /logn)(Clog2n)=C n ( y ta dng c lng rt s lc log2 m 1).
Nh vy, nu n vo c khong 100 ch s thp phn, s cc php tnh bit phi dng
s vo c 1050. Vi nhng my tnh thc hin mt triu php tnh trong mt giy,
thi gian cn thit s vo khong 3,1.1036 nm!
Ta kt thc tit ny bng nh l quan trng sau y, thng c gi l nh l c
bn ca s hc .
nh l 2.6. Mi s nguyn t ln hn 1 u phn tch c mt cch duy nht thnh
tch cc s nguyn t, trong cc tha s c vit vi th t khng gim.
Chng minh. Gi s tn ti nhng s khng vit c thnh tch cc s nguyn t.
Gi n l s b nht trong cc s . Nh vy, n phi l hp s, n=a.b, vi a, b<n. Do
nh ngha ca n cc s a v b phn tch c thnh tch cc s nguyn t, ngha l n
cng phn tch c. Mu thun vi gi thit.
Cn phi chng minh phn tch l duy nht. Gi s ta c:
10

n=p1 p2 ...ps=q1q2...qr,
trong pi, qj l cc s nguyn t. Gin c nhng s nguyn t bng nhau c mt
trong hai v, ta c ng thc
pi1pi2...piu=qj1qj2...qjv,,
trong khng c s nguyn t no c mt c hai v. Nh vy, v tri chia ht cho
qj1, v do phi tn ti mt tha s ca tch chia ht cho qj1: iu v l, v y l
tch cc s nguyn t khc vi qj1.

V
V
n
n
M
M
a
a
tth
h
..C
C
o
o
m
m

Phn tch nh trn ca cc s nguyn c gi l phn tch ra tha s nguyn t. Khi


n l mt s rt ln, vic kim tra xem n l s nguyn t hay hp s, v nu l hp s
th tm phn tch ca n ra tha s nguyn t, l mt bi ton ht sc kh khn.
Trong nhng phn tip theo ca cun sch, ta s tm hiu nhiu thut ton lm
vic , cng nh cc ng dng ca n trong thc tin.

3. Thut ton Euclid.

Mt trong nhng thut ton c bn v lu i nht ca ton hc l thut ton Euclid.


Thut ton cho php xc nh c chung ln nht ca hai s nguyn cho trc.
Khi trnh by thut ton Euclid, ta nhc li s qua khi nim ng d. Nhng tnh
cht cn dng ca ng d v cc tnh cht c bn ca c chung ln nht c cho
trong cc bi tp ca chng ny.
Gi s m l mt s nguyn dng. Ta ni hai s nguyn a v b l ng d vi nhau
modulo m nu m chia ht hiu a-b ( ta dng cch vit m | (a-b)). ch quan h
ng d, ta dng k hiu a b (mod m).
Nh vy, a b (mod m) khi v ch khi tn ti s nguyn k sao cho a=b+km.
Quan h ng d l mt trong nhng quan h c bn ca s hc, v ta s gp thng
xuyn trong nhng phn tip theo ca cun sch. Trong thut ton Euclid, ta ch
dng quan h din t ngn gn v phn d ca php chia.
Thut ton sau y cho php tnh c chung ln nht (CLN) d ca hai s nguyn
khng m a v b (k hiu l d=(a,b)).

Thut ton Euclid

E1. [Kt thc?] Nu b=0, in ra a v kt thc thut ton.


E2. [Chia Euclid] t r a mod b, a b, b r v quay v
bc 1.
V d: tnh d=(24,63) bng thut ton Euclid.
Ta c: d=(24,63) = (15,24)=(9,15)=(6,9)=(3,6)=(0,3)=3.

11

nh l sau y va cho ta mt chng minh tnh ng n ca thut ton Euclid,


va cho mt c lng v phc tp ca n.
nh l Lam. S php chia cn thit tm CLN ca hai s nguyn dng bng
thut ton Euclid khng vt qu 5 ln s ch s thp phn ca s b trong hai s
cho.
Chng minh. Gi s a>b l hai s nguyn dng cho trc. Bng thut ton Euclid,
ta c: a=r0, b=r1 v:
r0=r1q1+r2, 0 r2<r1
r1=r2q2+r3, 0 r3<r2

V
V
n
n
M
M
a
a
tth
h
..C
C
o
o
m
m

... ............

rn-2=rn-1qn-1+rn, 0 rn<rn-1
rn-1=rnqn

Nh vy, ta lm n php chia. Trong cc php chia , ta c: q1, q2, ...,qn-1 1,


2, rn <rn-1. T suy ra:

qn

rn 1=f2,

rn-1 2rn 2f2=f3

rn-2 rn-1+ rn f3+f2=f4

rn-3 rn-2+ rn-1 f4+f5=f6


.................

r2 r3+ r4 fn-1+fn-2=fn

b=r1 r2+ r3 fn+fn-1=fn+1

Ch rng, dy s {fn} nhn c chnh l dy s Fibonaci quen thuc trong s hc.


i vi dy s ny, bng quy np, d chng minh c lng sau y:
fn>(

1 + 5 n-2
) .
2

T bt ng thc b fn+1 ta c:

log10b (n-1)log10(

1+ 5
)>(n-1)/5
2

nh l c chng minh.
H qu 2.6. Gi s a<b, khi s cc php tnh bit cn thit thc hin thut ton
Euclid l O((log2a)3).

12

Tht vy, s php chia phi lm l O(log2a), v mi php chia cn O((log2a)2) php
tnh bit.
Thut ton Euclid, mc d ra i hng nghn nm, vn l thut ton tt nht
tm CLN ca hai s nguyn cho trc! Cho n nm 1967, J.Stein xy dng c
mt thut ton kh thun tin tm CLN trong trng hp cc s cho c vit
di dng nh phn. u im ch yu ca thut ton ny l ta khng cn lm cc
php tnh chia (thc ra ta c lm php chia s chn cho 2, nhng trong c s 2 th
l php dch chuyn s cho sang phi mt v tr). Thut ton da trn nhng nhn
xt n gin sau (xem phn bi tp cui chng):
1) Nu a,b l cc s chn, th (a,b)=2(a/2,b/2).
2) Nu a chn, b l, th (a,b)=(a/2,b).

V
V
n
n
M
M
a
a
tth
h
..C
C
o
o
m
m

3) Nu a,b u l th a-b chn v |a-b|<max(a,b).


4) (a,b)=(a-b,b).

Thut ton c m t nh sau ( chng ti dnh phn chng minh cho c gi).

Thut ton tm CLN ca hai s nguyn dng a,b.

E1. (Tm lu tha ca 2) t k 0 v lp lin tip php


tnh sau cho n khi t nht mt trong hai s a, b l:
t k k+1, a a/2, b b/2.
E2. (Xut pht). ( bc xut pht ny, a, b u
c chia cho 2k, v c t nht mt trong hai s l l).
Nu a l, t t -b v chuyn sang E4. Nu ngc li,
t t a.
E3. (Chia i t). (Ti thi im ny, t chn, khc 0).
t t t/2.
E4. (t c chn hay khng?) Nu t chn quay v E3.
E5. (Sp xp li max(a,b)). Nu t>0, t a t; nu
ngc li, t b -t. Nh vy, s ln nht trong hai s
c thay bi |t|.
E6. (Tr) t t a-b. Nu t 0, quay li E3. Nu ngc
li thut ton kt thc v in ra a.2k.
Ngoi thut ton Euclid ni trn, trong nhiu trng hp, ta cn n thut ton
Euclid m rng. Thut ton ny khng nhng cho ta thut ton tm CLN ca hai s
a, b, m cn cho ta biu din d=(a,b) di dng t hp tuyn tnh ca a, b:
d=ma+nb, trong m, n l cc s nguyn.
Trc ht, ta chng minh b sau:
B 2.7: CLN ca cc s nguyn a v b l s d dng nh nht biu din c
di dng t hp tuyn tnh ca a v b.

13

Tht vy, gi s d l s nguyn dng nh nht biu din c di dng d=ma+nb.


Ta chng t d l c chung ca a v b. Xt php chia a=dq+r, trong 0 r<d. R
rng r cng l mt t hp tuyn tnh ca a v b, nn do d l s nguyn dng nh
nht c tnh cht , r=0. Tng t, d l c ca b. D thy rng, mi c chung
khc ca a v b cng l c ca d: vy d chnh l c chung ln nht.
Khi cho hai s a, b, tm biu din ca d nh trong b , ta thng l nh sau:
vit a=bv+q, 0 q<b. Sau ,li vit b=uq+r=u(a-bv)+r, 0 r<q. Tip tc qu trnh
, do cc s d q, r gim dn nn ta thu c biu din cn thit. iu va ni c
th hin trong thut ton sau y, m chng minh cht ch c dnh cho c gi.

Thut ton Euclid m rng.

V
V
n
n
M
M
a
a
tth
h
..C
C
o
o
m
m

Cho hai s nguyn khng m u, v, tm (u1,u2,u3) sao cho (u,v)=u3=uu1+vu2. Trong


tnh ton, ta thm vo cc n ph (v1,v2,v3), (t1,t2,t3) v lun c trong mi bc cc
ng thc sau y:
ut1+vt2=t3, uv1+vv2=v3, uu1+vu2=u3.

Ed1.(Xut
(0,1,v).

pht).

(u1,u2,u3) (1,0,u),

(v1,v2,v3)

Ed2. (Kim tra v3=0?) Nu v3=0, thut ton kt thc.


Ed3. (Chia, tr). t q [u3/v3], v sau t
(t1,t2,t3) (u1,u2,u3)-q(v1,v2,v3), (v1,v2,v3) (t1,t2,t3) v
quay v bc 2.
V d. Cho a=63, b=24. Dng thut ton Euclid ta c:
- Bc 1. u1=1, u2=0, u3=63, v1=0, v2=1, v3 =24.

- Bc 2. q=2, u1 =0, u2=1, u3=24, v1=1, v2=-2, v3=15.


- Bc 3. q=1, u1=1, u2=-2, u3=15, v1=-1, v2=3, v3=9.
- Bc 4. q=1, u1=-1, u2=3, u3=9, v1=2, v2=-5, v3=6.
- Bc 5. q=1, u1=2, u2=-5, u3=6, v1=-3, v2=8, v3=3.

- Bc 6. q=2, u1=-3, u2=8, u3=3, v1=8, v2=-21, v3=0.


Ta c biu din: 3=(-3)64+8.24.

4. nh l Trung Quc v phn d:


Gi s m1,m2,...,mr l cc s nguyn dng nguyn t cng nhau tng cp. Khi h
ng d:
x1 a1(mod m1),
x2 a2(mod m2),
14

... ... ...


xr ar(mod mr).
C nghim duy nht modulo M=m1m2...mr.
Chng minh. Trc ht ta xy dng mt nghim ca h.
Gi s Mk=M/mk= m1m2...mk-1mk+1...mr. Ta bit rng (Mk,mk)=1 v (mj,mk)=1 vi mi
j k. Nh vy, theo bi tp 2.18 ta c th tm mt nghch o yk ca Mk modulo mk,
tc l Mkyk 1 (mod mk).
t
x=a1M1y1+ a2M2y2 +...+ arMryr .

V
V
n
n
M
M
a
a
tth
h
..C
C
o
o
m
m

Ta thy rng x ak(mod mk) vi mi k v mk |Mj vi j k nn Mj 0 (mod mk) khi j k.


Nh vy, x chnh l mt nghim ca h ang xt.
Ta chng t rng nghim va xy dng l duy nht modulo M. Gi s x0, x1 l hai
nghim ca h. Khi , vi mi k, x0 x1 ak (mod mk), cho nn mk | (x0-x1). Theo bi
tp 2.17, M | (x0-x1). nh l c chng minh.
nh l Trung Quc v phn d lin quan bi ton ni ting Hn Tn im binh.
Tng truyn rng, kim tra qun s, Hn Tn thng ra lnh cho qun s xp
thnh hng 3, hng 5, hng 7 v thng bo cho ng cc s d. Khi bit cc s d v
c sn thng tin gn ng v s qun ca mnh, Hn Tn dng nh l trn y
suy ra s qun chnh xc.
nh l Trung Quc v phn d c s dng trong my tnh lm vic vi nhng
s ln. a mt s nguyn ln tu vo my tnh v lm cc php tnh s hc vi
chng, ta cn c nhng k thut c bit. Theo nh l Trung quc v phn d, khi
cho trc cc modun nguyn t cng nhau m1,m2,...,mr, mt s dng n<M=
m1m2...mr c xc nh duy nht bi cc thng d dng b nht ca n theo
modulo mj vi j=1,2,...,r. Gi s rng c t ca my ch l 100, nhng ta cn lm cc
php tnh s hc vi nhng s nguyn c 106. Trc tin ta tm cc s nguyn nh
hn 100, nguyn t cng nhau tng cp, sao cho tch ca chng vt qu 106. Chng
hn, ta c th ly m1=99, m2=98, m3=97, m4=95. Ta chuyn cc s nguyn b hn 106
thnh nhng b 4 s theo thng d dng b nht modulo m1,m2,m3,m4 ( lm c
iu ny, ta cng phi lm vic vi nhng s nguyn ln! Tuy nhin iu ch cn
lm mt ln vi input, v mt ln na vi ouput). Nh vy, chng hn cng cc s
nguyn, ta ch cn cng cc thng d dng b nht ca chng modulo m1,m2,m3,m4.
Sau li dng nh l Trung Quc v phn d tm b 4 s tng ng vi tng.
V d. Ta mun tnh tng x=123684, y=413456 vi mt my tnh c t l 100. Ta c:
x 33(mod 99), 8(mod 98), 9(mod 97), 89(mod 95)
y 32(mod 99), 92(mod 98), 42(mod 97), 16(mod 95)
Nh vy,
x+y 65(mod 99), 2(mod 98), 51(mod 97), 10(mod 95)

15

By gii ta dng nh l Trung Quc v phn d tm x+y modulo


M=99.98.97.95=89403930. Ta c: M1=M/99=903070, M2=M/98=912288,
M3=M/97=921690, M4=M/95=941094. Ta cn tm ngc ca Mi(mod yi) vi
i=1,2,3,4, tc l gii h phng trnh ng d sau y (Bng thut chia Euclid):
903070y1 91y1 1(mod 99)
912285y2 3y2 1(mod 98)
921690y3 93y3 1(mod 97)
Ta tm c: y1 37(mod 99), y2 38(mod 98), y3 24(mod 97), y4 4(mod 95).
Nh vy,

V
V
n
n
M
M
a
a
tth
h
..C
C
o
o
m
m

x+y=65.903070.37+2.912285.33+51.921690.24+10.941094.4=3397886480
537140(mod 89403930)

V 0<x+y<89403930, ta suy ra x+y=537140.

Rt c th c gi cho rng, cch cng hai s s dng nh l Trung Quc v phn d


qu phc tp so vi cch cng thng thng. Tuy nhin, cn ch rng, trong v d
trn y, ta lm vic vi cc s nh. Khi cc s cn cng c ln vt xa c t ca
my, cc quy tc cng thng thng khng cn p dng c na.
Ni chung c t ca my tnh l lu tha rt ln ca 2, chng hn 235. Nh vy, s
dng nh l Trung Quc v phn d, ta cn cc s nh hn 235 nguyn t cng nhau
tng cp. tm cc s nguyn nh vy, thun tin nht l dng cc s dng 2m-1,
trong m l s nguyn dng. Cc php tnh s hc vi nhng s c dng nh vy
tng i n gin da vo b sau.
B 2.8. Nu a v b l cc s nguyn dng th thng d dng b nht modulo 2b1 ca 2a-1 l 2r-1, trong r l thng d dng b nht ca a modulo b.
Tht vy, nu a=bq+r, trong r l thng d dng b nht ca a modulo b, th ta
c
(2a-1)=(2bq+r-1)=(2b-1)(2b(q-1)+r+...+2b+r+2r)+(2r-1).

H qu 2.9. Nu a v b l cc s nguyn dng, th c chung ln nht ca 2a-1 v


2b-1 l 2(a,b)-1.
H qu 2.10. Cc s nguyn 2a-1 v 2b-1 nguyn t cng nhau khi v ch khi a v b
nguyn t cng nhau.
Chng ti dnh vic chng minh hai b ny cho c gi.
Ta c th s dng h qu trn y tm cc s nh hn 235, nguyn t cng nhau
tng cp, sao cho tch ca chng ln hn mt s cho. Gi s ta cn lm cc php
tnh s hc vi nhng s nguyn c c 2184. Ta t: m1=235-1, m2=234-1, m3=233-1,
m4=231-1, m5=229-1, m6=223-1. V s m ca 2 trong cc s trn nguyn t vi nhau
tng cp, nn theo h qu trn, cc s chn cng nguyn t vi nhsu tng cp. Ta
c tch m1 m2 m3 m4 m5 m6>2184. By gi ta c th lm cc php tnh s hc vi
nhng s c n 2184.
16

V
V
n
n
M
M
a
a
tth
h
..C
C
o
o
m
m

Trong cc my tnh hin i, vic thc hin nhiu php tnh c tin hnh ng
thi. V th vic s dng nh l Trung Quc v phn d nh trn li cng tin li:
thay cho vic lm cc php tnh vi cc s nguyn ln, ta lm nhiu php tnh ng
thi vi nhng s nguyn b hn. iu gim ng k thi gian tnh ton.

17

Thut ton gii phng trnh ng d bng nh l Trung Quc


T chng minh nh l Trung Quc v phn d, ta c thut ton sau y gii h
phng trnh ng d x xi (mod mi), trong mi, 1 i k l cc s nguyn t vi
nhau tng cp, xi l cc s nguyn cho trc. Trong thut ton trnh by sau y,
chng ta tm ra cch trnh phi lm vic vi cc s ln nh Mi v aiMi.
Thut ton.
1. (Xut pht). t j 2, C1 1. Hn na ta sp xp li
cc s mi theo th t tng dn.

V
V
n
n
M
M
a
a
tth
h
..C
C
o
o
m
m

2. (Tnh ton s b). t p m1m2...mj-1(mod mj). Tnh


(u,v,d) sao cho up+vmj=d=UCLN(p,mj) bng thut ton
Euclid m rng.
Ed. Nu d>0, in ra thng bo: cc mi khng nguyn t cng
nhau tng cp. Nu ngc li, t Cj u, j j+1 v
chuyn sang bc 3 nu j k.
3. (Tnh cc hng s ph). t y1 x1 mod m1, v mi
j=2,...,k tnh:
yj (xj-(y1+m1(y2+m2(y3+...+mj-2yj-1)...))Cjmod mj.
4. (Kt thc). In ra

x y1+m1(y2+m2(y3+...+mk-1yk)...), v kt thc thut ton.

5. Mt s ng d c bit.

nh l Wilson. p l s nguyn t khi v ch khi (p-1)! -1 (mod p).

Chng minh. Trc tin, gi s p l s nguyn t. Khi p=2, ta c


(p-1)! 1 -1(mod 2). By gi gi s p l s nguyn t ln hn 2. Theo bi tp 2.18,
vi mi s nguyn a vi 1 a p-1, tn ti nghch o a , 1 a p-1, vi
a a 1(mod p). Theo bi tp 2.13, trong s cc s nguyn dng nh hn p, ch c 1
v p-1 l nghch o vi chnh n. Nh vy ta c th nhm cc s nguyn t 2 n
p-2 thnh (p-3)/2 cp s nguyn, tch ca mi cp ng d vi 1 modulo p. Nh vy
ta c:
2.3.....(p-3)(p-2) 1 (mod p)
Nhn hai v vi 1 v p-1 ta c:
(p-1)! 1.2.3...(p-2)(p-1) 1(p-1) -1(mod p)
Ngc li gi s p tho mn ng d pht biu trong nh l v a l mt c s ca
p, a<p. Khi , a | (p-1)!. Nhng theo gi thit, p | (p-1)!+1, t suy ra a=1, v l
c chung ca p v (p-1)!. Vy p l s nguyn t, nh l c chng minh.

18

nh l Wilson c th c dng kim tra mt s c phi l s nguyn t hay


khng. Tuy nhin , d thy rng, thut ton da theo nh l Wilson kh c th s
dng vi nhng s nguyn ln, bi v s cc php tnh bit i hi qu cao.
n gin, ta gi cng vic xem xt mt s cho c phi l s nguyn t hay
khng l kim tra nguyn t. nh l sau y c nhiu ng dng trong kim tra
nguyn t.
nh l Fermat b. Nu p l s nguyn t v a l s khng chia ht cho p th
ap-1 1(mod p).
Chng minh. Xt p-1 s nguyn a, 2a,..., (p-1)a. Cc s u khng chia ht cho p
v khng c hai s no ng d modulo p. Nh vy, cc thng d dng b nht ca
chng phi l 1, 2,... p-1, xp theo th t no . T ta c:

V
V
n
n
M
M
a
a
tth
h
..C
C
o
o
m
m

a.2a.....(p-1) 1...(p-1) (p-1)!(mod p)

tc l

ap-1(p-1)! 1(mod p)

V ((p-1)!,p)=1 nn ta c ap-1 1(mod p).

H qu 2.11. Nu p l s nguyn t v a l s nguyn dng th ap a(mod p).


H qu 2.12. Nu p l s nguyn t v a l s nguyn khng chia ht cho p th ap-2 l
nghch o ca a modulo p.
H qu 2.13. Nu a v b l cc s nguyn dng, p nguyn t, p|a th cc nghim
ca ng d thc tuyn tnh ax b(mod p) l cc s nguyn x sao cho
x ap-2b(mod p).

6. S gi nguyn t.

Theo nh l Fermat, nu n l s nguyn t v b l s nguyn tu , th bn b(mod n).


Do nu tn ti s b sao cho bn / b(mod n) th n phi l hp s. Trong nhiu ng
dng , chng ta li cn n cc thut ton ch ra mt s n l s nguyn t. Trong
trng hp ny, ta khng th dng nh l Fermat b, v nh l ngc ca n khng
ng. Tuy nhin, nu mt s nguyn tho mn cc gi thit ca nh l Fermat b th
c nhiu kh nng n l mt s nguyn t! Ta c nh ngha sau y.
nh ngha 2.14. Gi s b l mt s nguyn dng. Nu n l hp s nguyn dng
v bn b(mod n) th n c gi l s gi nguyn t c s b.
Trong trng hp (n,b)=1, ta thng dng nh ngha tng ng: bn-1 b(mod n).
V d. S nguyn 561=3.11.17 l s gi nguyn t c s 2. Tht vy, p dng nh l
Fermat
b, ta c 2560=(22)280 1(mod 3), 2560=(210)56 1(mod 11),
560
16 35
2 =(2 ) 1(mod 17). T suy ra (bi tp 2.12) 2560 1(mod 561).

19

Ni chung cc s gi nguyn t t hn nhiu so vi cc s nguyn t. Chng hn, c


tt c 4550525112 s nguyn t b hn 1010, nhng ch c 14884 s gi nguyn t c
s 2 trong khong . S kin ny gii thch cch ni trn: Cc s tho mn nh l
Fermat b c nhiu kh nng l s nguyn t. Tuy nhin i vi mi c s tu , s
cc s gi nguyn t l v hn. Chng hn, ta chng minh iu i vi c s 2.
nh l 2.15. C v s s gi nguyn t c s 2.
Chng minh. Gi s n l mt s gi nguyn t c s 2, ta s chng t rng, m=2n-1
cng l s gi nguyn t c s 2. Theo gi thit, n l hp s, chng hn n=dt
(1<d,t<n), v 2n-1 1(mod n). D thy rng m l hp s, v (2d-1) | (2n-1)=m. Do n
l gi nguyn t, tn ti k sao cho 2n-2=kn. Ta c 2m-1=2kn, v do ,
m=(2n-1)|(2nk-1)=2m-1-1, tc l 2m-1 1(mod m). Vy s m l gi nguyn t c s 2.

V
V
n
n
M
M
a
a
tth
h
..C
C
o
o
m
m

Nh vy, kim tra mt s c phi l s nguyn t hay khng, trc tin ta xem n
c l gi nguyn t c s 2 hay khng, sau c th tip tc kim tra i vi cc c
s khc. Tuy nhin, tn ti cc s gi nguyn t vi mi c s, l cc s
Carmichael.
nh ngha 2.16. Hp s nguyn n tho mn bn-1 1(mod n) vi mi s nguyn
dng b sao cho (n,b)=1 c gi l s Carmichael.
V d. S nguyn 561=3.11.17 l mt s Carmichael. Tht vy, nu (b,561)=1 th
(b,3)=(b,11)=(b,17)=1. Theo nh l Fermat b, ta c b2 1(mod 3), b10 1(mod 11),
b16 1(mod 17). Do , vit 560=2.280=10.56=16.35 ta c:
b560=(b2)280 1(mod 3),

b560=(b10)56 1(mod 11),


b560=(b16)35 1(mod 17).

T suy ra (bi tp 2.12): b560 1(mod 561).

Gi thuyt sau y mi c chng minh rt gn y ([AGP]): tn ti v hn s


Carmichael.
nh l sau y cho mt cch tm s Carmichael.

nh l 2.17. Nu n=q1q2...qk, trong qj l cc s nguyn t khc nhau tho mn


(qj-1) |(n-1), th n l s Carmichael.
Tht vy, gi s b l s nguyn dng, (b,n)=1. Khi , (b,qj)=1 vi mi j, v
q 1
b j 1(mod q j ) . V (qj-1) |(n-1) nn bn-1 1(mod qj), v do , bn-1 1(mod n).
Phn o ca nh l trn y cng ng, tuy nhin c chng minh hi di nn ta
s b qua. c gi no quan tm c th tm c trong [Ro].
Nh vy, vic kim tra nguyn t s kh khn khi gp phi cc s Carmicheal. Tuy
nhin, ta c th khc phc bng cch sau y. Nu gp ng d bn-1 1(mod n), ta
chuyn sang xt ng d b(n--1)/2 x(mod n). Nu n l s nguyn t th x 1 hoc
x -1(mod n), ngc li th n phi l hp s (bi tp 2.22).

20

V d, vi s Carmicheal b nht 561 ta c: 5(561-1)/2=5280 67(mod 561). Vy, 561 l


hp s.
V sau, ta s cp n nhng thut ton kim tra nguyn t hin i. Trong phn
ny, thy thm ng dng ca cc nh l ng d va trnh by, ta tm hiu vi
thut ton n gin.
nh ngha 2.18. Gi s n l s nguyn dng l, n-1=2st, trong s l s nguyn
khng m, t l s nguyn dngj l. Ta ni n tri qua c kim tra Miller c s b,
nu hoc bt 1(mod n), hoc b2 t -1(mod n), vi j no , 0 j s-1.

V
V
n
n
M
M
a
a
tth
h
..C
C
o
o
m
m

Ta chng t rng, nu n l s nguyn t th n tri qua c kim tra Miller


c s b
k
s kt
vi mi s b sao cho n|b. Tht vy, gi s n-1=2st. t xk=b(n-1)/2 =b2 , vi
k=0,1,...,s. V n l s nguyn t nn x0 1(mod n). Do x12 1(mod n), tc l
x1 1(mod n) hoc x1 -1(mod n). Tip tc qu trnh nh vy ta s i n kt lun
rng, hoc xk 1(mod n) vi k=0,1,...,s, hoc xk -1(mod n) vi mt s nguyn k no
. Nh vy n tri qua c kim tra Miller c s b.
D thy rng, nu n tri qua c kim tra Miller c s b th n s l s gi nguyn t
c s b. Ta c nh ngha sau.
nh ngha 2.19. n c gi l s gi nguyn t mnh c s b nu n l hp s v
tri qua c kim tra Miller c s b.
Nh vy cc s gi nguyn t mnh li cn t hn cc s gi nguyn t. Tuy nhin, ta
c nh l sau.
nh l 2.20. Tn ti v s s gi nguyn t mnh c s 2.

Tht vy, gi s n l mt s gi nguyn t c s 2. Khi , 2n-1=nk vi s nguyn l k


no . t N=2n-1, ta c
N-1=2n-2=2(2n-1-1)=2nk;

ngha l n l hp s. Mt khc,

2(N-1)/2=2nk=(2n)k 1(mod N).

Vy vi mi s gi nguyn t n, ta xy dng c s gi nguyn t mnh N v cc


s n khc nhau cho ta cc s N khc nhau: nh l c chng minh, bi v c v s
gi nguyn t c s 2.
Ta c th dng kim tra Miller kim tra nguyn t nhng s khng ln lm. Ta
bit rng, s gi nguyn t mnh l c s 2 b nht l 2047. Nh vy, nu n l v
n<2047, th n l nguyn t nu n tri qua kim tra Miller. Tng t nh vy, s
1373653, l s gi nguyn t mnh l b nht c s 2 v 3, c dng kim tra
nguyn t nhng s b hn n. i vi c s 2,3 v 5, s gi nguyn t mnh l b
nht l 25326001, trong trng hp c s 2,3,5,7,s tng ng l 3215031751.
Trong nhng s nh hn 25.109, ch c mt s gi nguyn t l vi c s 2,3,5,7,
l 3251031751. Nh vy, nu n<25.109 l s l tri qua kim tra Miller, th n l s
nguyn t nu n khc vi 3251031751.

21

Cch lm trn y ch p dng c khi cn kim tra nguyn t nhng s khng ln.
i vi nhng s ln, ta c th dng thut ton xc sut da trn nh l sau y:
nh l 2.21. Nu n l mt hp s dng l th tn ti khng qu (n-1)/4 c s b,
1 b n-1, sao cho n tri qua c kim tra Miller i vi cc c s .
nh l trn y c chng minh da vo khi nim ch s m ta khng trnh by
y. c gi no quan tm c th tm c trong[Ro]. Nh nh l 2.21, ta c th kt
lun n l mt hp s nu thy n tri qua kim tra Miller vi hn (n-1)/4 c s. Tuy
nhin, vic kim tra nh th i hi qu nhiu thi gian.

V
V
n
n
M
M
a
a
tth
h
..C
C
o
o
m
m

T nh l 2.21 suy ra rng, nu s b c chn ngu nhin trong khong 1 b n-1


th n tri qua kim tra Miller c s b vi xc sut b hn 1/4. Nh vy, nu ta chn k
s ngu nhin th xc sut n tri qua kim tra Miller i vi k c s s b hn
1/4k. Khi k ln, v d k=20, xc sut qu nh, nn vi n tri qua vi 20 c s
ngu nhin th c th tin hu chc chn rng n l s nguyn t. T ta c thut
ton xc sut sau y.

Thut ton Rabin-Miller (1980)

Cho N 3 l, thut ton sau y xc nh rng N l mt hp s, hoc in ra thng


bo N l s nguyn t vi xc sut ln hn 1-1/420.
RM1. (Xut pht). t q N-1, t 0, v nu q chn t
q q/2, t t+1 (by gi ta c N-1=2tq, vi q l). Sau
t c 20.
RM2. (Chn a mi). Chn ngu nhin s a trong khong
1<a<N. t e 0, b aq mod N. Nu b=1, chuyn sang RM4.
RM3.(Bnh phng). Nu b / 1(mod N) v e<t-2, ta t
b b2 mod N, e e+1. Nu b N-1, in ra thng bo n l
hp s v kt thc thut ton.
RM4. t c c-1. Nu c>0, chuyn sang RM2. Nu c=0, in
ra thng bo N l s nguyn t.

7. Phn s lin tc.

Gi s a,b l cc s nguyn dng, a>b. Khi , phn s a/b c th vit di dng:


c
a
1
= a0 + 0 = a0 + .
b
b
b
c0

Phn s b/c0 li c th biu din di dng tng t nh vy, v cui cng ta nhn
c:

22

a
= ao +
b

1
a1 +

1
... a n 1 +

1
an

Cch vit nh trn c gi l biu din s hu t a/b di dng phn s lin tc.
n gin k hiu, ta thng dng cch vit a/b=[a0;a1,a2,...,an]. Phn s lin tc
[a0;a1,a2,...,an] c gi l phn s lin tc hu hn.

V
V
n
n
M
M
a
a
tth
h
..C
C
o
o
m
m

Dng thut ton Euclid, c th biu din mi s hu t di dng phn s lin tc


hu hn. Tht vy, ta c a=a0b+c0, b=a1c0+c1,.... Ngc li, r rng mi phn s hu
hn lin tc l mt s hu t.
Ta cng c th biu din mt s thc tu di dng phn s lin tc. Tuy nhin
trong trng hp ny, phn s lin tc c th khng hu hn. Cch lm cng hon
ton tng t nh khi lm vi cc s hu t.
Gi s x l s thc tu . t a0=[x], phn nguyn ca x, v x0=x-a0 l phn l ca x.
Tip theo , ta t a1=[1/x0], x1=1/x0-a1. Tm li i vi mi s i>1, t ai=[1/xi-1],
xi=1/xi-1-ai. Nu bc th i no , xi=0 th qu trnh kt thc (iu ny xy ra khi
v ch khi x l s hu t). Ngc li, ta c biu din x di dng phn s lin tc v
hn: [a0;a1,a2,...,an,...].
Nhiu khi thun tin, ta dng cch vit sau y:
x= a 0 +

1
1
1
+
+...+
+...
a1 + a 2 +
an +

Cc phn s lin tc nh ngha nh trn vi cc s ai nguyn cn c gi l cc


phn s lin tc n gin. Khi khng i hi ai l cc s nguyn, m c th l cc s
thc tu , ta cng dng cch vit
x=[a0;a1,a2,...,an]=

a0 +

1
1
1
+
+...+ .
a1 + a 2 +
an

Khi c mt phn s lin tc x=[a0;a1,a2,...,an,...], ta gi cc s sau y l cc phn s


hi t ring ca x:
Ck= [a0;a1,a2,...,ak].

nh l 2.22. Gi s a0,a1,...,an l cc s thc, trong a0,a1,...,an>0. t p0=a0,


q0=1, p1=a0a1+1, q1=a1, v vi mi k 2, pk=akpk-1+pk-2, qk=akqk-1+qk-2. Khi i
vi cc phn s hi t ring Ck ta c:
Ck= [a0;a1,a2,...,ak]= pk/qk.
Chng minh. Ta chng minh bng qui np. Vi k=0, C0=a0=p0/q0. Vi k=1,
1
C1=[a0;a1]=a0+ =p1/q1.
a1
23

Ta c:
Ck+1=[a0;a1,a2,...,ak+1]= a 0 +

=[a0;a1,a2,...,ak-1,ak+

1
a k +1

1
1
1
+
+...+
a1 + a 2 +
a k +1

(a k +

) p k 1 + p k 2
a k +1
]=
.
1
(a k +
)q k 1 + q k 2
a k +1

theo gi thit qui np. Tnh ton n gin da vo nh ngha cc s pk,qk, ta c:

V
V
n
n
M
M
a
a
tth
h
..C
C
o
o
m
m

Ck+1=pk+1/qk+1.
nh l c chng minh.

nh l 2.23. Vi mi k 1, ta c:

pkqk+1-pk-1qk=(-1)k-1.

T ta suy ra ngay rng, cc s pk,qk nguyn t cng nhau.


nh l 2.24. Ta c:

C1>C3>C5>...
c0<C2<C4>...

C2j+1>C2k, vi mi j,k
lim Ck=x.

Chng minh cc nh l trn (bng quy np) c dnh cho c gi. C th thy
rng, tn gi phn s lin tc ring c gii thch bng nh l trn y.
nh l 2.25. Gi s n l mt s t nhin khng chnh phng v pk,qk l cc phn s
hi t ring ca n . Ta t 0 = n , v cc s k , Qk, Pk c nh ngha theo
cng thc sau:

k = (Pk+ n )/ Qk,
ak=[ k ],

Pk+1=akQk-Pk
Qk+1=(n-Pn+12)Qk
Khi ta c:
pk2-n qk2=(-1)k-1Qk+1.
Chng minh. p dng nh l va chng minh, ta c:

24

n = 0 =[a0;a1,a2,...,ak+1]=

a k +1 p k + p k 1
.
a k +1q k + q k 1

T , v ak+1= (Pk+1+ n )/ Qk+1, ta c:


n=

( Pk +1 + n ) p k + Qk +1 p k 1
( Pk +1 + n )q k + Qk +1q k 1

Vy,
nqk+(Pk+1qk+Qk+1qk-1)

n =(Pk+1pk+Qk+1pk-1) +pk n .

V
V
n
n
M
M
a
a
tth
h
..C
C
o
o
m
m

T suy ra:
nqk= Pk-1pk+ Qk+1pk-1,
pk= Pk+1qk+ Qk+1qk-1.

Nhn ng thc th nht vi qk, ng thc th hai vi pk v tr ng thc th hai cho


ng thc th nht, ta thu c kt qu cn chng minh.
Sau y ta s p dng phn s lin tc tm mt thut ton phn tch s nguyn ra
tha s nguyn t. Ni chnh xc hn, ta s xy dng mt thut ton vi s t
nhin n cho trc, tm c s khng tm thng (khc 1 v n).
Ta xut pht t nhn xt n gin sau y: Nu ta tm c cc s x, y sao cho
x-y 1 v x2-y2=n th ta tm c s c khng tm thng ca n, v
n= x2-y2=(x-y)(x+y).
By gi, gi s ta c kt qu yu hn, chng hn tm c x,y sao cho x2 y2(mod n)
v 0<x<y<n, x+y n.
Khi n l mt c ca tch (x-y)(x+y), v r rng n khng l c ca x+y cng nh
x-y. Nh vy cc c s chung d1=(n,x-y) v d2=(n,x+y) l cc c s khng tm
thng ca n. Cc c s ny tm c mt cch nhanh chng nh thut ton Euclid.
nh l 2.25 cho ta phng php tm cc s x,y cn thit.
Theo nh l 2.25 ta c:

pk2 (-1)k-1Qk-1 (mod n).

Nh vy, vn l phi tm c cc Qk vi ch s chn, v l mt s chnh phng.


Mi ln tm c mt s Q k nh vy, ta tm c mt c ca n (cng c th xy ra
trng hp c l tm thng: cc pk,Qk khng nht thit b hn n nn iu kin
n khng phi l c ca x+y v x-y c th khng tho mn).
V d. 1). Phn tch s 1037 ra tha s bng cch s dng phn s lin tc.
ta c = 1037 =32,2..., Q1=1,Q2=49, p1=129. Nh vy, 1292 49(mod 1037). Do
, 1292-72=(129-7)(129+7) 0(mod 1037). Tnh cc c chung ln nht, ta c:
(129-7,1037)=61, (129+7,1037)=17. Ta c hai c ca 1037, v trong trng hp
ny c khai trin 1037=61.17.

25

V
V
n
n
M
M
a
a
tth
h
..C
C
o
o
m
m

2) Phn tch 1000009. Ta tnh c Q1=9, Q2=445, Q3=873, Q4=81. Nh vy,


p32 92(mod 100009): ta khng thu c c khng tm thng. Tnh ton tip tc,
ta c: Q18=16 l mt s chnh phng, p17=494881. Bng thut ton m t, ta tm
c cc c s 293, 3413.

26

Bi tp v tnh ton thc hnh chng 2


I. Bi tp.
2.1. Chuyn s (1999) t c s 10 sang c s 7, s (6105) t c s 7 sang c s 10.
2.2. Chuyn cc s 10001110101 v 11101001110 t c s 2 sang c s 16 (k hiu
cc ch s ca c s 16 bi 0, 1, 2, 3, 4, 5, 6, 7, 8, 9, A, B, C, D, E).
2.3. Chng minh rng mi vt nng khng qu 2k-1 (vi trng lng l s nguyn)
u c th cn bng mt ci cn hai a, s dng cc qu cn 1, 2, 22,...,2k-1.
2.4. Chng minh rng mi s nguyn u c th biu din duy nht di dng

V
V
n
n
M
M
a
a
tth
h
..C
C
o
o
m
m

ek3k+ek-13k-1+...+e13+e0,

trong ej=-1, 0 hoc 1; j=0,1,...,k.

2.5. Chng minh rng, mi s thc R, 0 <1 u c th biu din duy nht
di dng c s b

= c j / b j , 0 cj b-1,
j =1

tho mn iu kin: vi mi N, tn ti n N cn b-1.

2.6. p dng bi 2.5, vit trong c s 2 vi 10 ch s sau du phy.

2.7. a) Chng minh rng mi s nguyn dng n u c biu din Cantor duy nht
di dng sau:
n=amm!+am-1(m-1)!+...+a22!+a11!.

b) Tm khai trin Cantor ca 14, 56, 384.

2.8. Gi s a l s nguyn (trong c s 10) vi bn ch s sao cho khng phi mi


ch s l nh nhau. a l s nhn c t a bng cch vit cc ch s theo th t
gim dn, a l s nhn c bng cch vit cc ch s theo th t tng dn. t
T(a)=a-a. V d: T(1998)=9981-1899.
a) Chng minh rng s nguyn duy nht (khng phi 4 ch s u nh nhau) sao cho
T(a)=a l a=6174.
b) Chng minh rng nu a l s nguyn dng 4 ch s, khng phi mi ch s u
nh nhau, th dy a, T(a), T(T(a)),... nhn c bng cch lp php ton T, s dng
s 6174 (c gi l hng s Kapreka)
2.9. c lng thi gian cn thit tnh n!.
2.10. c lng thi gian cn thit chuyn mt s k-bit sang h thp phn.
2.11. a) Chng minh rng, nu A, B l cc ma trn vung cp n th tm tch AB
(theo quy tc nhn ma trn thng thng) ta cn n3 php nhn.

27

b) Chng minh rng c th nhn hai ma trn vung cp hai m ch cn 7 php nhn,
nu s dng ng nht thc sau y:

a11

a 21

a12 b11

a 22 b21

b12
=
b22

a 11 b11 + a 12 b 21

x + ( a 11 a 21 )(b 22 b12 ) a 22 (b11 b 21 b12

x + ( a 21 + a 22 )( b12 b11 ) + ( a 11 + a 12 a 21 a 22 ) b 22
+ b 22 )

x + ( a 11 a 21 )( b 22 b12 ) + ( a 21 + a 22 )( b12 b11 )

trong x=a11b11-(a11-a21-a22)( b11-b12+b22).

V
V
n
n
M
M
a
a
tth
h
..C
C
o
o
m
m

c) Bng quy np v tch ma trn 2n 2n thnh 4 ma trn n n, chng minh rng c


th nhn hai ma trn 2k 2k ch vi 7k php nhn v khng t hn 7k+1 php cng.
d) T c) suy ra rng c th nhn hai ma trn vung cp n vi O(nlogc7) php tnh bit
nu mi phn t ca ma trn c di c bit, vi hng s c no .
2.12. Dng sng Eratosthenes tm mi s nguyn t b hn 1998.

2.13. Cho Qn=p1p2...pn+1, trong p1,p2,..., pn l n s nguyn t u tin. Tm c


nguyn t b nht ca Qn, vi n=1,2,3,4,5,6.
Trong dy Qn c v hn hay hu hn s nguyn t?

2.14. Chng minh rng tn ti v hn s nguyn t.

2.15. Chng minh rng nu c nguyn t b nht p ca mt s nguyn dng n


vt qu 3 n th n/p l s nguyn t.
2.16. Chng minh rng khng tn ti mt b ba nguyn t no p, p+2, p+4 ngoi
3,5,7.
2.17. Chng minh rng nu a|x, b|x v a, b nguyn t cng nhau th a.b|x.

2.18. Chng minh rng nu a,m nguyn t cng nhau th tn ti nghch o m mod b.
2.19. Cho a,b,c,m l cc s nguyn, m dng. Gi s d l CLN ca c v m. Khi ,
nu ac bc(mod m) th a b(mod m/d).
2.20. Gi s r1, r2,...,rm l mt h thng d y modulo m, a l mt s nguyn,
nguyn t cng nhau vi m, b l s nguyn tu . Chng minh rng ar1+b, ar2+b ,...,
arm+b cng l mt h y cc thng d modulo m
2.21. Gi s a b(mod mj), j=1,2,..., k, trong mj l cc s nguyn t cng nhau
tng cp. Chng minh rng a b(mod m1m2...mk).
2.22. Cho p l s nguyn t. Chng minh rng a2 1(mod p) khi v ch khi
a 1(mod p).
2.23. Chng minh rng vi mi s nguyn khng m m,n v mi s nguyn a>1, ta
c
(am-1, an-1)=a(m,n)-1.

28

2.24. a) Chng minh rng c th tm c chung ln nht ca hai s nguyn dng


bng thut ton sau

2(a / 2,b / 2)

(a,b)=
(a / 2, b)
(a b, b)

nu a = b
nu a , b ch n
nu a ch n, b l
nu a , b l

b) Dng thut ton trn tm (2106, 8318).


2.25. Chng minh rng, vi mi n, tm c n s t nhin lin tip sao cho mi s
u c c l s chnh phng.

V
V
n
n
M
M
a
a
tth
h
..C
C
o
o
m
m

2.26. Gi s n=p1p2...pk, trong pj l cc s nguyn t, v n l mt s Carmichael.


Chng minh rng k 3. p dng kt qu tm ra s Carmichael nh nht.
2.27. Chng minh rng, nu 6m+1, 12m+1, 18m+1 u l s nguyn t th
(6m+1)(12m+1)(18m+1) l s Carmichael.
Chng minh cc s sau y l s Carmichael:

1729, 294409, 56052361, 118901521, 172947529.

2.28. Chng minh rng 6601 l mt s Carmichael.

2.29. Chng minh rng n=2047=23.89 l s gi nguyn t mnh c s 2.

2.30. Cho b,m l cc s nguyn nguyn t cng nhau, a,c l cc s nguyn dng.
Chng minh rng, nu ba 1(mod m), bc 1(mod m) v d=(a,c) th bd 1(mod m).
2.31. Cho p l s nguyn t, p|bm-1. Chng minh rng, hoc p|bd-1 vi d no l
c thc s ca m (khc m), hoc d 1(mod m). Nu p>2, m l th trong trng hp
sau, ta c p 1(mod 2n).
2.32. p dng bi tp trn phn tch ra tha s cc s 211-1=2047, 213-1=8191,
312-1=531440, 235-1=34355738367.
2.33. Tm phn s lin tc ca cc s

2 , 3,

5, (1+ 5 )/2.

2.34. Bit phn s lin tc ca e l

e=[2;1,2,1,1,4,1,1,6,1,1,8,...]

a) Tm 8 phn s hi t ring u tin ca e.


b) Tm xp x hu t tt nht ca e c mu s b hn 100.
2.35. Cho l mt s v t. Chng minh rng, hoc | -pk/qk|<1/2qk2, hoc
| -pk+1/qk+1|<1/2qk+12.
2.36. Cho f(x) l mt a thc tu vi h s nguyn. Chng minh rng tn ti v hn
s nguyn k sao cho f(k) l hp s.

29

II. Thc hnh tnh ton trn my


i vi tt c cc chng, tnh ton thc hnh trn my tnh vi chng trnh Maple
c bt u bng dng lnh:
[>with(numtheory);

Cc php ton s hc ( php cng [+], php tr [-], php nhn [*], php chia [/],
php lu tha [^], khai cn bc hai [sqrt(.)],...) c vit v thc hin theo th t
quen bit.

V
V
n
n
M
M
a
a
tth
h
..C
C
o
o
m
m

Lun lun ghi nh rng cui dng lnh phi l du chm phy (;) hoc du (:). Mun
thc hin dng lnh no th phi a con tr v dng lnh (sau du chm phy) v
nhn phm [Enter]. Hy thc hin cc dng lnh theo ng trnh t trc sau, v mt
s tnh ton trong cc bc sau c th yu cu kt qu t cc bc trc.

II. 1. Thc hnh kim tra mt s l s nguyn t

kim tra mt s n c phi l s nguyn t hay khng ta thc hin lnh nh sau:
[>isprime(n);

Sau du (;) n phm Enter. Nu trn mn hnh hin ra ch true th n l s


nguyn t, nu trn mn hnh hin ra ch false th n l hp s.
Th d: S 2546789 c phi l s nguyn t hay khng?
[>isprime(n);

False

Vy 2546789 khng phi l s nguyn t.

II. 2. Thc hnh tm c chung ln nht

thc hnh tm c chung ln nht ca hai s a v b, hy vo dng lnh c c


php nh sau:
[>gcd(a,b);

Sau du (;) n phm Enter th vic tm c chung ln nht s c thc hin v s


c ngay kt qu.
Th d: Tm c s chung ln nht ca 2 s 157940 v 78864.

Thc hin bng cu lnh sau:

[> gcd(157940,78800);
20

Vy c chung ln nht ca 157940 v 78864 l 20.

II. 3. Phn tch ra tha s nguyn t


phn tch s n ra tha s nguyn t ta thc hin lnh sau:

30

[>ifactor(n);

Sau du (;) n phm Enter th vic phn tch n ra tha s nguyn t s c thc
hin v s c ngay kt qu.
Th d: Phn tch s 122333444455555666666777777788888888999999999 ra tha
s nguyn t.

Ta thc hin nh sau:

[>

V
V
n
n
M
M
a
a
tth
h
..C
C
o
o
m
m

ifactor(122333444455555666666777777788888888999999999);

(3)(12241913785205210313897506033112067347143)(3331)

Ta cng c th dng lnh trn kim tra xem mt s n c phi l s nguyn t hay
khng

II. 4. Thc hnh kim tra mt s l s Carmichael


Ta nh li nh l 2. 17 nh sau:

nh l 2.17. Nu n=q1q2...qk, trong qj l cc s nguyn t khc nhau tho mn


(qj-1) |(n-1), th n l s Carmichael.

Do kim tra xem mt s n c phi l s Carmichael hay khng ta thc hin


theo cc bc sau:
Bc 1: Phn tch n thnh tch cc tha s nguyn t, ta thc hin bng dng lnh:
[>ifactor(n);

Sau du (;) n phm Enter trn mn hnh s hin ra kt qu phn tch n ra tha s
nguyn t. Nu n l hp s v c dng n=q1q2...qk, trong qj l cc s nguyn t
khc nhau th thc hin tip bc kim tra th 2. Nu khng th c th khng nh n
khng phi l s Carmichael.
Bc 2:. Thc hin cc php tnh chia (n-1):(qj-1), ta thc hin bng dng lnh sau:

[>(n-1)/(qj-1);

Sau du (;) n phm Enter trn mn hnh s hin ra kt qu thng ca php chia.
Nu vi mi j=1,2, ..., k cc thng tm c l cc s nguyn th ta khng nh n l
s Carmichael, nu khng th tr li khng phi.
Th d 1: S 6601 c phi l s Carmichael hay khng?

Thc hin kim tra nh sau:


[>ifactor(6601);
(7)(23)(41)

31

6601 c phn tch thnh cc tha s nguyn t khc nhau, vy c th nghi ng n


l s Carmichel. kim tra xem n c thc s l s Carmichel hay khng, ta thc
hin cc lnh sau:
[>(6601-1)/(7-1);
1100
[>(6601-1)/(23-1);
300
[>(6601-1)/(41-1);
165

V
V
n
n
M
M
a
a
tth
h
..C
C
o
o
m
m

Vy 6601 l s Carmichael.

Th d 2: S 6 c phi l s Carmichael hay khng?

Thc hin kim tra nh sau:


[>ifactor(6);

(2)(3)

[>(6-1)/(2-1);

[>(6-1)/(3-1);

5
2

Vy 6 khng phi l s Carmichael.

Th d 3: S 45 c phi l s Carmichael hay khng?

Thc hin kim tra nh sau:


[>ifactor(45);

(3)2(5)

S 45 khng tho mn bc th nht.


Vy 45 khng phi l s Carmichael.

II. 5. Thc hnh kim tra mt s l gi nguyn t


Cho hai s nguyn dng n, b. kim tra xem n c phi l s gi nguyn t c s
b hay khng ta thc hin cc bc nh sau:
Bc 1: Kim tra n l hp s, ta thc hin dng lnh:
[>isprime(n);

32

Sau du (;) n phm Enter. Nu trn mn hnh hin ra ch true th n l s


nguyn t, nu trn mn hnh hin ra ch false th n l hp s. Nu n l s nguyn
t th n khng phi l s gi nguyn t c s b. Nu ngc li thc hin tip bc 2.
Bc 2: Kim tra ng d thc bn-b 0(mod n), thc hin bng dng lnh:
[>b&^n-b mod n;

Sau du (;) n phm Enter trn mn hnh s hin ra kt qu. Nu l s 0 th n l


s gi nguyn t c s b.
Th d1: S 561 c phi l s gi nguyn t c s 2 hay khng?

Ta thc hin cc lnh sau:

V
V
n
n
M
M
a
a
tth
h
..C
C
o
o
m
m

[>isprime(561);

false

[>2&^561-2 mod 561;

Vy 561 l s gi nguyn t c s 2.

Th d 2: S 12241913785205210313897506033112067347143 c phi l s gi
nguyn t c s 8 hay khng?

Ta thc hin cc lnh sau:

[>ispime(12241913785205210313897506033112067347143);
true

S 12241913785205210313897506033112067347143 l mt s nguyn t. Do
12241913785205210313897506033112067347143 khng phi l s gi nguyn t
c s 8.
Th d 3: S 326 c phi l s gi nguyn t c s 3 hay khng?

Ta thc hin cc lnh sau:


[>isprime(326);

false

[>3&^326-3 mod 326;

Vy 326 l khng phi l s gi nguyn t c s 3.

II. 6. Thc hnh kim tra mt s l s gi nguyn t mnh


Cho n l s nguyn dng l, b l s nguyn dng. kim tra n c phi l s gi
nguyn t mnh c s b hay khng ta thc hin theo cc bc sau:
Bc 1: Kim tra n l hp s, ta thc hin bng dng lnh:

33

[>isprime(n);

Sau du (;) n phm Enter. Nu trn mn hnh hin ra ch true th n l s


nguyn t, nu trn mn hnh hin ra ch false th n l hp s. Nu n l s nguyn
t th n khng phi l s gi nguyn t mnh c s b. Nu ngc li thc hin tip
bc 2.
Bc 2: Phn tch n-1 ra tha s nguyn t, ta thc hin bng dng lnh:
[>ifactor(n-1);

Sau du (;) n phm Enter trn mn hnh s hin ra s phn tch ca n-1 v ta thu
c kt qu c dng n-1=2st, trong s l s nguyn dng, t l s nguyn dng
l.

V
V
n
n
M
M
a
a
tth
h
..C
C
o
o
m
m

Bc 3: Kim tra ng d thc bt-1 0(mod n). Vo lnh


[>b&^t-1 mod n;

Sau du (;) n phm Enter trn mn hnh s hin ra kt qu. Nu l s 0 th n l


s gi nguyn t mnh c s b, nu kt qu l mt s khc 0 ta thc hin tip bc 4.
Bc 4: Kim tra cc ng d thc
dng lnh:

(b 2 t + 1) 0(mod n) vi j=0,...s-1, ta thc hin

[>seq (b&^((2^j)t)+1 mod n, j=0..s-1);

Sau du (;) n phm Enter trn mn hnh s hin ra dy kt qu. Nu trong dy kt


qu c mt s l s 0 th n l s gi nguyn t mnh c s b.
Th d: S 2047 c phi l s gi nguyn t mnh c s 2 hay khng?

Thc hin kim tra nh sau:


[>isprime(2047);

false

Do n l hp s. Tip tc thc hin lnh


[>ifactor(n-1);

(2)(3)(11)(31)

Tip tc thc hin lnh

[>2&^(3*11*31)-1 mod 2047;

Vy 2047 l s gi nguyn t mnh c s 2.

II. 7. Thc hnh biu din mt s di dng phn s lin tc


1. Biu din s n di dng phn s lin tc theo cch thng thng vi s thng
trong biu din l k, ta dng lnh:
[>cfrac(n,k);

34

Sau du (;) n phm Enter trn mn hnh s xut hin kt qu.


Th d: Biu din di dng phn s lin tc theo cch thng thng vi 6
thng.

Ta thc hin lnh:


[> cfrac (Pi,6);
1

3+

7+

15 +

V
V
n
n
M
M
a
a
tth
h
..C
C
o
o
m
m

1+

292 +

1+

1
1+...

2. Biu din s n di dng phn s lin tc theo cch n gin vi s ch s trong


biu din l k, ta dng lnh:
[>cfrac(n,k,quotients);

Sau du (;) n phm Enter trn mn hnh s xut hin kt qu.

Th d: Biu din di dng phn s lin tc theo cch vit n gin vi 100 ch
s biu din.

Ta thc hin lnh:

[> cfrac (Pi,100,quotients);

[3,7,15,1,292,1,1,1,2,1,3,1,14,2,1,1,2,2,2,2,1,84,2,1,1,
15,3,13,1,4,2,6,6,99,1,2,2,6,3,5,1,1,6,8,1,7,1,2,3,7,1,
2,1,1,12,1,1,1,3,1,1,8,1,1,2,1,6,1,1,5,2,2,3,1,2,4,4,16,
1,161,45,1,22,1,2,2,1,4,1,2,24,1,2,1,3,1,2,1,1,10,2,...]
3. Biu din s n di dng phn s lin tc theo chu k tun hon, ta dng lnh:
[>cfrac(n,periodic);

Sau du (;) n phm Enter trn mn hnh s xut hin kt qu.

Th d: Biu din 31/2 di dng phn s lin tc theo chu k tun hon.

Ta thc hin lnh:


[>cfrac (3^(1/2),'periodic');
1

1+

1+

2+
1+

1
2 +...

35

4. Biu din s n di dng phn s lin tc theo chu k tun hon n gin, ta dng
lnh:
[>cfrac (n,'periodic','quotients');

Sau du (;) n phm Enter trn mn hnh s xut hin kt qu.


Th d: Biu din 31/2 di dng phn s lin tc theo chu k tun hon n gin.

Ta thc hin lnh:


[> cfrac (3^(1/2),'periodic','quotients');
[[1], [1, 2]]

V
V
n
n
M
M
a
a
tth
h
..C
C
o
o
m
m

II. 8. Thc hnh tm phn s hi t th k ca mt s


thc hnh tm phn s hi t th k ca mt s n, ta thc hin theo cc lnh sau:
Buc 1: Biu din n di dng phn s lin tc
[> cf:= cfrac(n);

Sau du (;) n phm Enter trn mn hnh s xut hin s biu din
Bc 2: Tnh phn s hi t th k
[> nthconver(cf,k);

Sau du (;) n phm Enter trn mn hnh s xut hin ra kt qu.

Trong qu trnh thc hin ta khng cn bit kt qu hin th bc 1, do c th


thay du (;) bng du (:) dng lnh u tin ([>cf:=cfrac(n):). Khi trn
mn hnh s hin ra du nhc ([>) thc hin tip lnh th 2.
Th d: Tnh phn s hi t th 5 ca e.

Ta thc hin nh sau:

[> cf:= cfrac(exp(1));

cf : = 2 +

1+

2+

1+

1+

4+

1+

1+
6+
[> nthconver(cf,5);

36

1
1+...

87
32
Nh vy, phn s hi t th 5 ca e l

87
.
32

II. 8. Thc hnh i c s


1. thc hnh i mt s n t c 10 sang c s b ta dng dng lnh sau:
[>convert(n,base,b);

V
V
n
n
M
M
a
a
tth
h
..C
C
o
o
m
m

Sau du (;) n phm Enter trn mn hnh s hin ln mt dng kt qu. Ch rng
kt qu a ra trn mn hnh c vit theo th t ngc li.
Th d 1: i s 24564 t c s 10 sang c s 6.

Ta thc hnh nh sau:

[>convert(24564,base,6);

[0, 2, 4, 5, 0, 3]

Vy ta c s l (305420)6.

Ch : Trong trng hp c s b >10, ta vn thc hin dng lnh i c s nh


bnh thng. Tuy nhin, sau khi nhn c kt qu, trnh nhm ln ta thc hin
vic t tng ng cc s ln hn 10 vi cc k hiu no . Ta xem v d sau:
Th d 2: i s 45676 t c s 10 sang c s 15, trong t 10=A,
11=B,12=C,13=D,14=E.

Ta thc hnh nh sau:

[>L:=convert(45676,base,6):

[>subs(10=A,11=B,12=C,13=D,14=E,L);

[1, 0, 8, D]

Vy ta c s l (D801)15.

2. thc hnh i mt s n t c s a sang c s b ta dng dng lnh sau:


[> convert(n,base,a,b);

Sau du (;) n phm Enter trn mn hnh s hin ln mt dng kt qu. Ch rng
kt qu a ra trn mn hnh c vit theo th t ngc li.
Th d: i s 305420 trong c s 6 sang c s 10.

Ta thc hin dng lnh

37

[> convert([0,2,4,5,0,3],base,6,10);

[4, 6, 5, 4, 2]

V
V
n
n
M
M
a
a
tth
h
..C
C
o
o
m
m

Vy ta c kt qu l (24564)10

38

Chng 3

Cc hm s hc
Khi nghin cu cc s nguyn, ta thng lm vic vi cc i lng nh: s cc c
ca mt s nguyn t cho trc, tng cc c ca n, tng cc lu tha bc k ca cc
c,... Ngoi nhng v d cn c rt nhiu hm s hc quan trng khc. Trong
chng ny, ta ch xt s qua mt vi hm quan trng. Phn ln ca chng c
ginh cho hm Euler, l mt trong nhng hm s hc quan trng nht.

V
V
n
n
M
M
a
a
tth
h
..C
C
o
o
m
m

1. nh ngha.

nh ngha 3.1. Hm s hc tc l hm xc nh trn tp hp cc s nguyn dng.


nh ngha 3.2. Mt hm s hc f c gi l nhn tnh nu vi mi n, m nguyn t
cng nhau, ta c f(mn)=f(m)f(n). Trong trng hp ng thc ng vi mi m,n
(khng nht thit nguyn t cng nhau), hm f c gi l nhn tnh mnh.
Nhng v d n gin nht v hm nhn tnh (mnh) l: f(n)=n v f(n)=1.

D chng minh tnh cht sau y: nu f l mt hm nhn tnh, n l s nguyn dng


c khai trin thnh tha s nguyn t dng n=p1a1p2a2...pkak, th f(n) c tnh theo
cng thc
f(n)=f(pa1)f(pa2)...f(pak).

2. Phi hm Euler.

Trong cc hm s hc, hm Euler m ta nh ngha sau y c vai tr rt quan trng.


nh ngha 3.3. Phi- hm Euler (n) l hm s hc c gi tr ti n bng s cc s
khng vt qu n v nguyn t cng nhau vi n.
V d. T nh ngha ta c: (1)=1, (2)=1, (3)=2, (4)=2, (5)=4, (6)=2,
(7)=6, (8)=4 , (9)=6, (10)=4.
T nh ngha trn y ta c ngay h qu trc tip: S p l nguyn t khi v ch khi
(p)=p-1.
Nu nh l Fermat b cho ta cng c nghin cu ng d modulo mt s nguyn t,
th Phi-hm Euler c dng xt ng d modulo mt hp s. Trc khi i vo
vn , ta cn mt s nh ngha sau.

39

nh ngha 3.4. H thng d thu gn modulo n l tp hp (n) s nguyn sao cho


mi phn t ca tp hp nguyn t cng nhau vi n , v khng c hai phn t no
ng d vi nhau modulo n.
Ni cch khc t h thng d y modolo n, lp h thng d thu gn, ta ch gi
li nhng gi tr no nguyn t cng nhau vi n.
V d. Cc s 1,2,3,4,5,6 lp thnh h thng d thu gn modulo 7. i vi modulo 8,
ta c th ly 1,3,5,7.

V
V
n
n
M
M
a
a
tth
h
..C
C
o
o
m
m

nh l 3.5. Nu r1,r2,...,r ( n ) l mt h thng d thu gn modulo n, v a l s


nguyn dng, (a,n)=1, th tp hp ar1,ar2,...,ar ( n ) cng l h thng d thu gn
modulo n.
Chng ti dnh chng minh nh l ny cho c gi.

nh l trn y c dng chng minh m rng ca nh l Fermat b.

nh l Euler. Nu m l s nguyn dng v a l s nguyn t cng nhau vi n th


a ( m) 1(mod m).
Chng minh. Ta lp lun hon ton tng t nh trong nh l Fermat b. Gi s
r1,r2,...,r ( m) modulo m, lp nn t cc s nguyn dng khng vt qu m v nguyn
t cng nhau vi m. Theo nh l 3.5, ar1,ar2,...,a r ( m) cng l mt h thng d thu
gn. Khi thng d dng b nht ca h ny s l tp hp r1,r2,..., r ( m) sp xp
theo mt th t no . Ta c:
ar1ar2...a r ( m) r1r2... r ( m) (mod m).

Nh vy,

( m)

r1,r2,...,r ( m) r1r2... r ( m) (mod m).

T suy ra nh l.

nh l Euler c th dng tm nghch o modulo m. Chng hn nu a v m l cc


s nguyn t cng nhau, ta c a.a ( m) 1 1(mod m), tc l a ( m) 1 chnh l nghch
o ca a modulo m. T cng suy ra nghim ca phng trnh ng d tuyn
tnh ax b(mod m), vi (a,m)=1 l x a ( m) 1 b(mod m).
nh l 3.6. Phi hm Euler l hm nhn tnh.

Chng minh. Gi s m, n l hai s dng nguyn t cng nhau. Ta cn chng t rng


(mn)= (m) (n). Ta sp xp tt c cc s nguyn dng khng vt qu nm
thnh bng sau:
1

m+1

2m+1

...

(n-1)m+1

m+2

2m+2

...

(n-1)m+2

... ... ... ... ... ... ... ... ... ... ... ... ... ... ... ... ... ...

40

m+r

2m+r

...

(n-1)m+r

... ... ... ... ... ... ... ... ... ... ... ... ... ... ... ... ... ...
m

2m

3m

...

mn

Gi s r l s nguyn khng vt qu m, v (m,n)=d>1. Khi trong hng th r


khng c s no nguyn t cng nhau vi mn. V th tnh (mn), ta ch cn quan
tm cc s trong hng th r vi (r,m)=1. Cc s trong hng ny u nguyn t cng
nhau vi m. Mt khc d thy rng cc s trong hng ny lp thnh mt h thng d
y modulo n. Do c ng (n) s trong hng nguyn t cng nhau vi n, tc
l trong hng c (n) s nguyn t cng nhau vi mn. C thy c (n) hng nh
vy, nh l c chng minh.

V
V
n
n
M
M
a
a
tth
h
..C
C
o
o
m
m

Nh tnh cht ny ta c ngay cng thc Phi-hm Euler.

nh l 3.7. Gi s n=p1a1p2a2...pkak l phn tch ca n thnh tha s nguyn t. Khi


ta c:

(n)=n(1-

1
1
1
)(1 )...(1 )
p1
p2
pk

Chng minh. Do Phi-hm Euler l hm nhn tnh nn ta ch cn chng minh rng,


vi mi s nguyn t p, (pk)=pk-pk-1.
Tht vy, cc s nguyn dng khng vt qu pk v khng nguyn t cng nhau vi
p phi c dng sp vi s nguyn dng no . C ng pk-1 s nh vy. Do , s cc
s khng vt qu pk v nguyn t cng nhau vi pk ng bng pk-pk-1. Tnh cht
quan trng sau y ca Phi-hm thng dc s dng v sau.
nh l 3.8. Gi s n l mt s nguyn dng. Khi

(d ) =n
d |n

trong tng c ly theo mi c ca n.

Chng minh. Ta phn cc s nguyn t 1 n n thnh tng nhm Cd: m Cd khi v


ch khi (m,n)=d, tc l khi v ch khi (m/d, n/d)=1. Nh vy, s phn t ca Cd ng
bng s cc s nguyn khng vt qu n/d v nguyn t cng nhau vi n/d, tc l
bng (n/d). Ta c
n= (n / d )
d |n

Khi d chy qua mi c ca n th n/d cng chy qua mi c ca n: nh l c


chng minh.
Nhn xt. Cc tnh cht ca Phi-hm Euler c s dng tnh ng d ca nhng
lu tha rt ln. Chng hn, ta cn tnh an mod k, trong n l mt s nguyn ln.
Gi s ta c

41

k= p1 1 p2 2 ... p s s .
Khi a ( pi i 1(mod pi i ). Nu N l bi chung nh nht ca cc ( pi i ) th
aN 1(mod k). Do , vit n=Nq+r vi r<N, ta c an ar(mod k).
)

Ta xt mt v d bng s. Tnh 21000000 mod 77. Ta c: 77=11.7, (7)=6, (11)=10.


Bi chung nh nht ca 6 v 10 l 30. Ta c 230 1(mod 77). Mt khc,
1000000=30.33333+10. Vy
21000000 210 23(mod 77).

V
V
n
n
M
M
a
a
tth
h
..C
C
o
o
m
m

3. S hon ho v s nguyn t Mersenne.

Tit ny dnh m t mt dng c bit ca s nguyn t, c vai tr quan trng


trong l thuyt v ng dng.
Ta bt u bng mt s hm s hc quan trng.

nh ngha 3.9. Hm (n), s cc c, c gi tr ti n bng s cc c dng ca n;


hm (n), tng cc c, c gi tr ti n bng tng cc c dng ca n. Ni cch
khc, ta c:

(n)= 1 ,
d |n

(n)=

d .
d |n

V d, nu p l mt s nguyn t th (p)=2, (p)=p+1.


nh l 3.10. (n) v (n) l cc hm nhn tnh.
D thy rng, nh l trn suy ra t b sau.

B 3.11. Nu f l hm nhn tnh, th F(n)= f (d ) cng l hm nhn tnh.


d |n

Tht vy, gi s m, n l cc s nguyn dng nguyn t cng nhau. Ta c:


F(mn)= f (d ) .
d |mn

V (m,n)=1, mi c d ca mn c th vit duy nht di dng d=d1d2 trong d1,d2


tng ng l c ca m,n, v d1,d2 nguyn t cng nhau. Do ta c
F(mn)=

f (d d
1

d1 |m ,d 2 |n

V f l hm nhn tnh v (d1,d2)=1 nn:

42

F(mn)= f (d 1 ) f (d 2 ) = f (d 1 ) f (d 2 ) =F(n)F(m)
d1 |m

d 2 |n

nh l c chng minh.
S dng nh l trn, ta c cng thc sau y cho cc hm (n) v (n).
nh l 3.12. Gi s n c phn tch sau y ra tha s nguyn t n=p1a1p2a2...pkak. Khi
ta c:
k

(n) =

1
p aj+1
j
pj 1

j=1

V
V
n
n
M
M
a
a
tth
h
..C
C
o
o
m
m

(n)=(a1+1)(a2+1)...(ak+1)= (a j + 1)
j =1

Chng ti dnh chng minh ny cho c gi.

Do cc quan nim thn b, ngi c Hy Lp quan tm n cc s nguyn bng tng


tt c cc c dng thc s ca n. H gi cc s l cc s hon ho.
nh ngha 3.13. S nguyn dng n dc gi l s hon ho nu (n)=2n.
V

d.

Cc

6,

28

(12)=1+2+4+7+14+28=56

cc

hon

ho:

(6)=1+2+3+6=12,

nh l sau y c bit t thi Hy lp.

nh l 3.14. S nguyn dng chn n l s hon ho khi v ch khi n=2m-1(2m-1),


trong m l mt s nguyn sao cho m 2 v 2m-1 l nguyn t.
Chng minh. Trc tin, gi s rng, m c dng nh trn. V l hm nhn tnh, ta
c: (n)= (2m-1) (2m-1). T cng thc ca hm v gi thit 2m-1 l nguyn
t, d thy rng (2m-1)=2m-1, (2m-1)=2m, v do (n)=2n.
Ngc li, gi s n l s hon ho chn. Vit n=2st, trong s,t l cc s nguyn
dng, t l, ta c:

(n)= (2st)= (2s) (t)=(2s+1-1) (t)

V n l s hon ho, (n)=2n=2s+1t.

Nh vy, 2s+1| (t), gi s (t)=2s+1q. Ta c ng thc


(2s+1-1)2s+1q=2s+1t,
tc l q|t v q t. Mt khc ta c:

t+q=(2s+1-1)q+q=2s+1q= (t)
Ta chng t rng, q=1. Tht vy, nu ngc li, t c t nht 3 c khc nhau l 1, t,
q, do (t) t+q+1, mu thun ng thc va chng minh. Vy (t)=t+1, ngha
l t l s nguyn t. nh l c chng minh.
Nh vy tm cc s hon ho, ta cn tm cc s nguyn t dng 2m-1.

43

nh ngha 3.15. Gi s m l mt s nguyn dng, khi Mm=2m-1 c gi l s


Mersenne th m. Nu p l s nguyn t, v Mp cng nguyn t, th Mp c gi l s
nguyn t Mersenne.
V d. M2,M3,M5,M7 l cc s nguyn t Mersenne, trong khi M11 l hp s. C nhiu
nh l khc nhau dng xc nh s nguyn t Mersenne. Chng hn nh nh l
sau y, ta c th kim tra nhanh chng da vo dng ca cc c s ca s nguyn
t Mersenne.
nh l 3.16. Nu p l mt s nguyn t l, th mi c ca s nguyn t Mersenne
Mp u c dng 2kp+1, trong k l s nguyn dng.

V
V
n
n
M
M
a
a
tth
h
..C
C
o
o
m
m

q|(2qChng minh. Gi s q l mt s nguyn t ca Mp. Theo nh l Fermat b,


1
-1). Theo h qu 1.9, (2p-1,2q-1-1)=2(p,q-1)-1. c chung ny ln hn 1, v n l mt
bi ca q. Do , (p,q-1)=p, v p l mt s nguyn t. Ta c q=mp+1, v v q l nn
m=2k, nh l c chng minh.
Sau y l vi v d cho thy ng dng ca nh l trn.

V d 1. xt xem M13=213-1=8191 c phi l s nguyn t hay khng, ta cn xem


cc php chia cho nhng s nguyn t khng vt qu 8191 =90,504... Mt khc,
theo nh l trn, mi c nguyn t u phi c dng 26k+1. Nh vy ch cn th
vi hai s 53 v 79: ta thy M13 l s nguyn t.
V d 2. Xt M23=8388607. Ta cn xt cc php chia ca n cho cc s nguyn t
dng 46k+1. S u tin 47 l c ca n: M23 l hp s.
C nhiu thut ton c bit kim tra nguyn t cc s Mersenne. Nh , ngi
ta pht hin c nhng s nguyn t rt ln. Mi ln c mt s nguyn t
Mersenne, ta li c mt s hon ho. Cho n nay, ngi ta bit c rng, vi
p 132049, ch c 30 s nguyn t Mersenne, v tnh c chng. S nguyn t
Mersenne tm c gn y nht l s M216091, gm 65050 ch s.
Gi thuyt sau y vn cn cha c chng minh.

Gi thuyt 3.17. Tn ti v hn s nguyn t Mersenne.

Ngi ta bit c rng, trong khong t 1 n 10200 khng c s hon ho l.


Tuy nhin cu hi sau y vn cha c tr li.
Cu hi 3.18. Tn ti hay khng cc s hon ho l?

4. Cn nguyn thu.
Khi xt cc s phc l cn bc n ca n v, ta thng ch nhng s no khng
phi l cn ca n v vi bc thp hn. Nhng s gi l cn nguyn thu ca n
v. i vi cc s nguyn, ta cng c khi nim hon ton tng t v cn v cn
nguyn thu ca n v.

44

nh ngha 3.19. Gi s a v m l cc s nguyn dng nguyn t cng nhau. Khi


s nguyn nh nht x tho mn ng d ax 1(mod m) c gi l bc ca a
modulo m. Ta vit x= ordma .
Ta ch rng, s x nh vy tn ti v theo nh l Euler, a (m) 1(mod m).
nh l 3.20. Gi s a v n l cc s nguyn t cng nhau, n>0. Khi s nguyn x
l nghim ca ng d ax 1(mod m) khi v ch khi x l mt bi ca bc ca a
modulo n.
Chng minh. Gi s x tho mn ng d trn. Ta vit x=q ordna+r, trong 0 r<x.
T ta c ar 1(mod m). V ordna l s dng nh nht c tnh cht nn r=0: x
l mt bi ca bc ca a modulo n. iu ngc li l r rng.

V
V
n
n
M
M
a
a
tth
h
..C
C
o
o
m
m

H qu 3.21. Nu a v n l cc s nguyn t cng nhau, n>0, th ordna | (n).


H qu 3.22. Nu a v n l cc s nguyn t cng nhau, n>0, th ai aj(mod n) khi
v ch khi i j(mod n).
Chng minh cc h qu trn c dnh cho c gi.

Do h qu 3.21, nu r v n l nguyn t cng nhau th bc ca r khng vt qu


(n). Cc s c bc ng bng (n) gi vai tr quan trng trong nhiu vn khc
nhau ca s hc. Ta c nh ngha sau.
nh ngha 3.23. Nu r v n l cc s nguyn t cng nhau, n>0, v nu
ordnr = (n) th r c gi l cn nguyn thu modulo n.
Ch rng khng phi mi s u c cn nguyn thu. Chng hn, xt n=8. Cc s
nh hn 8 v nguyn t cng nhau vi 8 l 1, 3, 5, 7, ng thi ta c ord81=1, bc
ca cc s cn li bng 2, trong khi (8)=4. Vn nhng s nguyn no th c cn
nguyn thu s c xt v sau.
nh l 3.24. Nu r, n nguyn t cng nhau, n>0, v nu r l cn nguyn thu
modulo n, th cc s sau y lp thnh h thng d thu gn modulo n:
r1,r2,...,r (n).

Chng minh. V (r,n)=1, cc s trn nguyn t cng nhau vi n. Ta ch cn chng t


rng, khng c hai s no ng d vi nhau modulo n. Gi s ri rj(mod n). Theo h
qu 3.22, i j(mod (n)). T suy ra i=j, v i, j khng vt qu (n). nh l c
chng minh.
nh l 3.25. Nu ordma=t v u l s nguyn dng, th ordm(au)= t / (t,u).
Chng minh. t v=(t,u), t=t1v, u=u1v, s= ordm(au). Ta c
(au)t 1 =(au1v)t/v=(at)u1 1(mod m).
Do , s|t1. Mt khc, (au)s=aus 1(mod m) nn t|su. Nh vy, t1v | u1vs, do , t1|u1s.
V (u1, t1)=1, ta c t1|s. Cui cng, v s|t1, t1|s nn s=t1=t/v=t/(t, u), chng minh
xong.

45

H qu 3.26. Gi s r l cn nguyn thy modulo m, trong m l s nguyn ln


hn 1. Khi ru l cn nguyn thy modulo m nu v ch nu (u, (m))=1.
Tht vy, ordmru=ord mr/(u, ordmr)= (m)/(u, (m)): h qu c chng minh.
nh l 3.27. Nu s nguyn dng m c cn nguyn thu, th n c tt c ( (m))
cn nguyn thu khng ng d nhau.
Tht vy, nu r l mt cn nguyn thu th r, r2, ..., r (m) l mt h y cc

V
V
n
n
M
M
a
a
tth
h
..C
C
o
o
m
m

thng d thu gn modulo m. S cn nguyn thu modulo m ng bng s cc s u


tho mn (u, (m))=1, v c ng ( (m)) s u nh th. nh l c chng
minh.

5. S tn ti ca cn nguyn thu.

Trong tit ny, ta s xc nh nhng s nguyn c cn nguyn thu. Trc tin ta s


chng minh rng mi s nguyn t u c cn nguyn thu. lm vic , ta cn
mt vi kin thc v ng d a thc.
Gi s f(x) l a thc vi h s nguyn. S c c gi l nghim ca a thc f(x)
modulo m nu f(c) 0(mod m). D thy rng, nu c l mt nghim th mi s ng
d vi c modulo m cng l nghim.
i vi s nghim ca mt a thc modulo mt s nguyn, ta cng c tnh cht
tng t nh s nghim ca mt a thc.
nh l Lagrange. Gi s f(x)=anxn+...+a1x+a0 l a thc vi h s nguyn, n>o,
ng thi an / 0(mod p). Khi f(x) c nhiu nht n nghim modulo p khng ng
d tng cp.
Chng minh. Ta chng minh bng qui np. Khi n=1, nh l l r rng. Gi s nh l
chng minh vi a thc bc n-1 c h s ca lu tha cao nht khng chia ht cho
p, v gi s rng a thc f(x) c n+1 nghim modulo p khng ng d tng cp
c0,c1,...,cn. Ta c f(x)-f(c0)=(x-x0)g(x), trong g(x) l a thc bc n-1 vi h s cao
nht l an. V vi mi k, 0 k n, ck-c0 / 0 (mod p), trong khi f(ck)-f(c0)=
(ck-c0)g(ck) 0(mod p), nn ck l nghim ca g(x) modulo p: tri vi gi thit quy
np. nh l c chng minh.
nh l 3.28. Gi s p l s nguyn t v d l mt c ca p-1. Khi a thc xd-1
c ng d nghim modulo p khng ng d tng cp.
Chng minh. Tht vy, gi s p-1=de. Ta c xp-1-1=(xd-1)g(x). Theo nh l Fermat
b, xp-1-1 c p-1 nghim modulo p khng ng d tng cp. Mt khc, mi mt
nghim phi l nghim ca xd-1 hoc l ca g(x). Theo nh l Lagrange, g(x) c
nhiu nht p-d-1 nghim khng ng d tng cp, v th xd-1 phi c t nht
(p-1)-(p-d-1)=d nghim. Li theo nh l Lagrange, xd-1 c khng qu d nghim, vy
n c ng d nghim modulo p khng ng d tng cp. nh l dc chng minh.

46

nh l trn y s c s dng trong chng 5 khi xy dng cc trng hu hn.


nh l 3.29. Gi s p l s nguyn t, d l c dng ca p-1. Khi , s cc s
nguyn khng ng d bc d modulo p l (d).
Chng minh. Gi s F(d) l s cc s nguyn dng bc d modulo p v b hn p. Ta
cn chng t rng F(d)= (d). V (d)=p-1 nn d|p-1, t ta c
p-1=

F (d )

d | p 1

Mt khc ta c:
p-1=

(d )

V
V
n
n
M
M
a
a
tth
h
..C
C
o
o
m
m

d | p 1

theo cng thc ca Phi-hm. Nh vy nh l s c chng minh nu ta chng t


c rng F(d) (d) nu d|p-1.
Khi F(d)=0, iu ni trn l tm thng. Gi s F(d) 0, tc l tn ti s nguyn a
bc d modulo p. Khi , cc s nguyn a, a2,...,ad khng ng d modulo p. R rng
rng, mi lu tha ca a l mt nghim ca xd-1 0(mod p), m s nghim khng ng
d ng bng d, nn mi nghim modulo p ng d vi mt trong cc lu tha ca
a. Do , v phn t tu bc d l mt nghim ca phng trnh xd-1 0(mod p) nn
phi ng d vi mt trong cc lu tha ca a. Mt khc, theo nh l 3.24, lu tha
k ca a c bc d khi v ch khi (k,d)=1. C ng (d) s k nh vy, v do suy ra
F(d) ) (d), nh l c chng minh.
H qu 3.30. Mi s nguyn t u c cn nguyn thu.

Tht vy, gi s p l s nguyn t. Khi c (p-1) s nguyn bc p-1 modulo p


(nh l 3.28) khng ng d tng cp. Theo nh ngha, mi s l mt cn
nguyn thu: p c (p-1) cn nguyn thu.
Phn cn li ca chng c ginh tm tt c cc s nguyn dng c cn
nguyn thu.
nh l 3.31. Nu p l mt s nguyn t l vi cn nguyn thu r, th hoc r, hoc
r+p l cn nguyn thu modulo p2.
Chng minh. V r l cn nguyn thu modulo p nn ta c
ordpr= (d)=p-1.

Gi s n= ord p 2 r. Ta c rn 1(mod p2), v do rn 1(mod p). Nh vy, bc p-1 ca


r l mt c ca n. Mt khc, n l bc ca r modulo p2 nn n l c ca
(p2)=p(p-1). V n|p(p-1) v p-1|n nn d dng suy ra rng, hoc n=p-1, hoc
n=p(p-1). Nu n=p(p-1) th r l cn nguyn thu modulo p2, v ord p2 r= (p2). Trong
trng hp cn li, n=p-1, ta c rp-1 1(mod p2). t s=r+p. Cn phi chng minh
rng s l cn nguyn thu modulo p2. V s r(mod p), s cng l cn nguyn thu

47

modulo p. Nh vy, theo chng minh trn ord p2 s hoc bng p-1, hoc bng p(p-1).
Ta s chng t rng, bc khng th l p-1. Ta c
sp-1=(r+p)p-1 rp-1+(p-1)prp-2(mod p2) 1+(p-1)prp-2 1-prp-2(mod p2)
T ta c th thy rng, sp-1 / 1(mod p2). Tht vy, nu ngc li th
prp2
p-2
2
0(mod p ), nn r 0(mod p). iu ny khng th c, v p /| r do r l cn nguyn
thu modulo p. Nh vy ord p2 s=p(p-1)= (p2), tc s=r+p l cn nguyn thu
modulo p2.
By gi ta xt lu tha tu ca s nguyn t

V
V
n
n
M
M
a
a
tth
h
..C
C
o
o
m
m

nh l 3.32. Gi s p l mt s nguyn t l, khi pk c cn nguyn thu vi mi


s nguyn dng k. Hn na, nu n l cn nguyn thu modulo p2 th r l cn nguyn
thu modulo pk vi mi s ngyn dng k.
Chng minh. T nh l 3.31, p c cn nguyn thu r sao cho cng l cn nguyn
thu modulo p2, v do
rp-1 / 1 (mod p2).

Ta s chng minh r cng l cn nguyn thu modulo pk vi mi s nguyn dng k.


Bng quy np c th thy rng
rp

k 1

( p 1)

/ 1 (mod pk)

(*)

vi mi s nguyn dng k. Gi s

n= ord p k r

Ta c n | (pk)=pk-1(p-1). Mt khc

rn / 1 (mod pk),

v rn / 1 (mod p).

Do p-1= (p) |n (nh l 3.30). V (p-1) |n v n|pk-1(p-1) nn n=pt(p-1), trong


t l s nguyn dng 0 t k-1. Nu n=pt(p-1) vi t k-2 th
rp

k 2

( p 1)

= (r p ( p 1) ) p

k 2 t

1 (mod p k ) ,

mu thun. Vy ord p k r =pk-1(p-1)= (pk), r cng l cng nguyn thuca pk.


Chng minh (*): k=2: ng. Gi s (*) ng vi s nguyn dng k 2. Khi
rp

k 2

( p 1)

/ 1 (mod p k ) .

V (r,p)=1, ta thy (r,pk-1)=1. Do , t nh l Euler ta c


rp

k 2

( p 1)

r( p

Vy tn ti s nguyn d sao cho


48

k 1

rp

trong p /| d, v theo gi thit r p

k 2

k 2

( p 1)

( p 1)

=1+dpk-1,

/ 1 (mod p k ) .

Ta ly lu tha bc p ca hai v phng trnh trn v nhn c


rp

k 1

( p 1)

p
= (1 + dp k 1 ) p = 1 + p(dp k 1 ) + p 2 (dp k 1 ) 2 +...+ (dp k 1 ) p
2
1 + dp k (mod p k +1 ).

V p /| d nn ta c
k 1

( p 1)

/ 1 (mod p k +1 ) ,

V
V
n
n
M
M
a
a
tth
h
..C
C
o
o
m
m

rp

chng minh xong.

V d: r=3 l cn nguyn thu modulo 7k vi mi s nguyn dng k.

nh l 3.33: Nu s nguyn dng n khng phi l lu tha ca mt s nguyn t


hoc hai ln lu tha mt s nguyn t, th n khng c cn nguyn thu.
Chng minh. Gi s n l s nguyn dng vi phn tch ra tha s nguyn t nh sau
t

n = p1 1 p2 2 ... pm m .

Gi s n c cn nguyn thu r, tc l (n,r)=1 v ordnr= (n). V (r,n)=1 nn


(r,pt)=1 trong pt l mt trong cc lu tha nguyn t c mt trong phn tch trn.
Theo nh l Euler,
t

r ( p ) 1 (mod p t ).

Gi s U l bi chung nh nht ca ( p1 1 ), ( p2 2 ),..., ( pm m ),


t

U=[ ( p1 1 ), ( p2 2 ),..., ( pm m ) ].
t

V ( pi i ) | U nn
t

rU 1(mod pi i )

Vi I=1, 2, ..., m. Do

ordnr= (n) U.
Mt khc,

(n)= ( p1t p2 t ... pmt ) = ( p1t ) ( p2 t )... ( pmt ) .


1

T ta c

( p1t ) ( p2 t )... ( pmt ) [ ( p1 t ), ( p2 t ),..., ( pm t ) ],


1

49

Tc l ( p1 t1 ), ( p2 t2 ),..., ( pm tm ) phi nguyn t cng nhau tng i mt. Do


(pt)=pt-1(p-1) nn (pt) chn nu p l, hoc nu p=2 v t 2. Vy, cc s

( p1t ), ( p2 t ),..., ( pm t ) khng nguyn t cng nhau tng cp, tr trng hp


1

m=1 (v do n l lu tha ca s nguyn t), hoc m=2 v n=2pt, trong p l s


nguyn t l v t l s nguyn dng.
nh l 3.34: Nu p l s nguyn t l v t l s nguyn dng, th 2pt c cn
nguyn thu. C th l, nu r l cn nguyn thu modulo pt th r, (tng ng, r+pt),
l cn nguyn thu modulo 2pt khi r l, (tng ng, khi r chn).
Chng minh: Gi s r l cn nguyn thu modulo pt, khi
t

V
V
n
n
M
M
a
a
tth
h
..C
C
o
o
m
m

r ( p ) 1 (mod p t ) ,
v khng c lu tha no nh hn (pt) tho mn ng d.
Do (2pt)= (2) (pt)= (pt) nn

r ( 2 p ) 1 (mod p t ).

Khi r l,

r ( 2 p ) 1 (mod 2) .

T ta c r ( 2 p ) 1 (mod 2 p t ) . V khng c lu tha b hn ca r tho mn ng


d nn r chnh l cn nguyn thu ca 2pt.
Khi r chn, r+pt l. Do ,

(r + p t ) ( 2 p ) 1 (mod 2) .

V r+pt r (mod pt) nn

(r + pt ) ( 2 p ) 1 (mod p t ) .

Do

(r + pt ) ( 2 p ) 1 (mod 2 p t ) ,

v v khng c lu tha b hn no ca (r+pt) tho mn ng d, ta suy ra r+pt l


cn nguyn thu modulo 2pt.
nh l 3.35: Nu a l s nguyn l, k 3 l s nguyn th
a (2

)/2

= a2

k 2

1 (mod 2k).

Chng minh. Ta chng minh bng quy np. Gi s a l s nguyn l, a=2b+1.Ta c


a2=4b(b+1)+1. V b hoc b+1 chn nn 8 | 4b(b+1)+1, tc l
a2 1 (mod 8).

50

Nh vy, nh l ng khi k=3. Gi s


a2

k 2

1 (mod 2k)

Khi tn ti s nguyn d sao cho

a2

k 2

=1+d.2k.

T ta c:
k 1

a 2 =1+d.2k+1+d2.22k,

tc l
k 1

1 (mod 2k+1).

V
V
n
n
M
M
a
a
tth
h
..C
C
o
o
m
m

a2

T nh l trn ta suy ra rng, cc lu tha 2k vi k 3 khng c cn nguyn thu.


Nh vy, trong cc lu tha ca 2 ch c 2 v 4 l c cn nguyn thu. Kt hp iu
ny vi cc nh l 3.32, 3.33, 3.34, ta c nh l sau y
nh l 3.36: S nguyn dng n c cn nguyn thu khi v ch khi
n=2, 4, pt, 2pt,

trong p l s nguyn t l, t l s nguyn dng.

51

Bi tp v tnh ton thc hnh chng 3


I. Bi tp
3.1. Hm Mbius c nh ngha nh sau: (n)=(-1)k, nu n khng chia ht cho s
chnh phng no khc 1, v k l s cc c nguyn t ca n; (1)=1, (n)=0 khi n
c c l s chnh phng khc 1.
Chng minh rng, vi mi n>1,

(d ) =0.

V
V
n
n
M
M
a
a
tth
h
..C
C
o
o
m
m

d |n

3.2 (Bin i ngc Mbius ). Cho f(n) l mt hm s hc. t


F(n)= f (d ) .
d |n

Chng minh rng:


1)

f(n)=

(d ) F(n/d).
d |n

2) Nu f l hm nhn tnh th F cng l hm nhn tnh.

3.3. Dng bin i ngc Mbius v cng thc n= (n/d), chng minh rng
d |n

1) (pk)=pk-pk-1 vi p l s nguyn t.
2)

(n) l hm nhn tnh.

3.4. Cho l hm nhn tnh v l hm Mbius. Chng minh rng, nu cc c


nguyn t ca n l p1,p2,...pk th

(d ) (d)=(1- (p ))(1- (p ))...(1- (p ))


1

d |n

(nu n=1, ta xem v phi l 1)

3.5. Hm k(n) (tng lu tha bc k ca cc c s ca n) c nh ngha nh sau:

k(n)= d k .
d |n

1) Cho cng thc tnh k(p) vi p l s nguyn t.


2) Tnh k(ps) khi s l s nguyn dng.
3) Chng minh rng k(n) l hm nhn tnh.

52

4) T cho cng thc tnh k(n) khi n= p1 1 p2 2 ... ps s .


3.6. Tm tt c cc s t nhin n tho mn

(n)+ (n)=2n.
3.7. Chng minh rng n l mt hp s khi v ch khi

(n)>n+ (n) .
3.8. Chng minh rng nu hai s nguyn c tch cc c s khc nhau th hai s
nguyn khc nhau.

V
V
n
n
M
M
a
a
tth
h
..C
C
o
o
m
m

3.9.Tnh cc ng d sau y bng nhiu phng php khc nhau (chng hn bng
phng php bnh phng lin tip hoc nh nhn xt cui 2):
1. 31000000mod 165.

2. 51234567mod 221.

3. 71000000000mod541.

3.10. Chng minh rng 91 l s gi nguyn t c s 3 nhng khng gi nguyn t


Euler c s 3, v khng l s gi nguyn t c s 2.
3.11. Cho f(n) l hm nhn tnh gii ni. Chng minh rng tng

f ( n) / n

hi t tuyt i trong na mt phng Re s>1 (trong Re l k hiu phn thc ca


mt s), v tng trong min hi t bng tch v hn hi t sau y

(1 + f ( p) p

+...+ f ( p m ) p ms +...) ,

pP

(tch c ly trn tp hp tt c cc s nguyn t).

3.12. Chng minh rng, nu f l hm nhn tnh mnh gii ni th

f ( n) / n
n =1

1
.
s
pP 1 f ( p) / p

3.13. Chng minh ng thc sau i vi Zeta-hm Riemann:

1
.
s
p P 1 p

( s) = 1 / n s =
n =1

3.14. Chng minh rng nu n 2,4,p ,2 p , trong p l s nguyn t l th


a ( n )/ 2 1(mod n) .
3.15. Chng minh rng nu n chia ht cho 24 th (n) cng chia ht cho 24.

53

3.17. a) Chng minh rng nu p,q l cc s nguyn t l khc nhau th n=pq l s gi


nguyn t c s 2 khi v ch khi ordq2|p-1, ordp2|q-1.
b) Trong cc s sau y, s no l s gi nguyn t c s 2: 871, 1378, 2047, 2813.
3.18. Chng minh rng nu p,q l cc s nguyn t l khc nhau th n=pq l s gi
nguyn t c s 2 khi v ch khi MpMq=(2p-1)(2q-1) l s gi nguyn t c s 2.
3.19. a) Chng minh rng nu a thc f(x) bc n, h s nguyn, c qu n nghim
modulo p th mi h s ca f(x) u chia ht cho p.
b) Cho p l mt s nguyn t. Chng minh rng mi h s ca a thc
f(x)=(x-1)(x-2)...(x-p+1)-xp-1-x+1

V
V
n
n
M
M
a
a
tth
h
..C
C
o
o
m
m

chia ht cho p.

c) Dng cu b) chng minh nh l Wilson.

3.20. Tm tt c cc s t nhin n sao cho: (n)=12, 18, 24, 48, 52, 84.

3.21. Chng minh rng vi mi k>1, phng trnh (n)=k c v s nghim.


3.22. Tm n nh nht (n)=1, 2, 3, 6, 14, 100.
3.23. Tm cn nguyn thu modulo:

112 , 17 2, 132, 192, 3k, 13k, 11k, 17k.

3.24. Chng minh rng nu m c cn nguyn thu th ng d x2 1(mod m) ch c


nghim x= 1(mod m).
3.25. Chng minh rng mc d khng tn ti cn nguyn thu 2k, k 3, mi s
nguyn l ng d vi ng mt s nguyn dng ( 1) 5 , trong =0 hoc 1,
l s nguyn tho mn o 2 k 2 1 .
3.26. Gi s n l mt s c cn nguyn thu. Chng minh rng tch ca cc s
nguyn dng nh hn n v nguyn t cng nhau vi n ng d (-1) modulo n (khi n
l s nguyn t, ta c nh l Wilson).
3.27. Tm tt c cc nghim ca ng d sau:
a) x2+x+1 0(mod 7)

b) x2+5x+1 0(mod 7)

c) x2+3x+1 0(mod 7).

II. Thc hnh tnh ton trn my tnh


II. 1. Tnh Phi-hm Euler
tnh Phi-hm Euler ca mt s nguyn dng n ta thc hin dng lnh nh sau:
[> phi(n);

54

Sau du (;) n phm Enter mn hnh s hin ra kt qu.

Th d: Tnh Phi-hm Euler ca 65.


[> phi(65);
48

II. 2. Thc hnh tm cc s khi bit phi-hm Euler ca n


tm cc s khi bit Phi-hm Euler k ta thc hin dng lnh sau:

V
V
n
n
M
M
a
a
tth
h
..C
C
o
o
m
m

[>invphi(k);

Sau du (;) n phm Enter mn hnh s hin ra cc s cn tm.


Th d: Tm cc s khi bit Phi-hm Euler ca n l 4.
Ta thc hin nh sau:
[> invphi(4);

[5, 8, 10, 12]

Vy cc s c Phi-hm Euler bng 4 l 5, 8, 10, 12.

II. 3. Thc hnh kim tra s nguyn t Mersenne

Cho m l mt s nguyn dng, t Mm:=2m-1. kim tra xem Mm c phi l s


nguyn t Mersenne hay khng ta thc hin dng lnh nh sau:
[> mersenne(m);

Sau du (;) n phm Enter. Nu trn mn hnh xut hin kt qu l mt s th Mm


l s nguyn t Mersenne v Mm chnh bng s . Nu khng trn mn hnh s xut
hin ch false.
Th d 1: M7 c phi l s nguyn t Mersenne hay khng?
Ta thc hin dng lnh nh sau:
[> mersenne(7);

127

Vy M7=127 v l s nguyn t Mersenne.


Th d 2: M125 c phi l s nguyn t Mersenne hay khng?
[> mersenne(125);
false
Vy M125 khng phi l s nguyn t Mersenne.

55

Th d 3: M11 c phi l s nguyn t Mersenne hay khng?


[> mersenne(11);
false
Vy M11 khng phi l s nguyn t Mersenne.

II. 4. Tnh bc ca mt s theo mt modulo no


Cho m l mt s nguyn dng, n l mt s nguyn. tnh bc ca n modulo m ta
thc hin dng lnh nh sau:
[>order(n,m);

V
V
n
n
M
M
a
a
tth
h
..C
C
o
o
m
m

Sau du (;) n phm Enter. Nu m, n l cc s nguyn t cng nhau th trn mn


hnh s xut hin kt qu chnh l bc ca n theo modulo m. Nu m, n khng nguyn
t cng nhau th trn mn hnh s xut hin ch FAIL.
Th d 1: Tnh bc ca 13 theo modulo 100.
[> order(13,100);

20

Vy ord10013=20.

Th d 2: Tnh bc ca 5 theo modulo 8


[>order(5,8);

Vy ord85 =2.

Th d 3: Tnh bc ca 8 theo modulo 12.


[> order(8,12);

FAIL

II. 5. Tm cn nguyn thu

1. Cho n l mt s nguyn ln hn 1. tm cn nguyn thu u tin modulo n ta


thc hin dng lnh nh sau:
[> primroot(n);

Sau du (;) n phm Enter. Nu trn mn hnh hin ra kt qu l mt s th s


chnh l cn nguyn thu u tin modulo n. Nu mn hnh hin ra ch FAIL th n
khng c cn nguyn thu.
Th d 1: Tm cn nguyn thu modulo 41.
[> primroot(41);
6
Vy 6 l cn nguyn thu modulo 41.

56

Th d 2: Tm cn nguyn thu modulo 15.


[> primroot(15);
FAIL
Vy 15 khng c cn nguyn thu.
2. tm cn nguyn thu modulo n ln hn g ta thc hin dng lnh sau:
[> primroot(g,n);

V
V
n
n
M
M
a
a
tth
h
..C
C
o
o
m
m

Sau du (;) n phm Enter. Nu trn mn hnh hin ra kt qu l mt s th s


chnh l cn nguyn thu ln hn g u tin modulo n. Nu mn hnh hin ra ch
FAIL th n khng c cn nguyn thu. Ch , nu g=0 th hai lnh trn l nh
nhau.
Th d 1: Tm cn nguyn thu u tin ln hn 7 modulo 41.
[> primroot(7,41);

11

Vy 11 l cn nguyn thu ln hn 7 u tin modulo 41.

Th d 2: Tm cn nguyn thu u tin ln hn 2 modulo 8.


[> primroot(2,8);

FAIL

Vy 8 khng c cn nguyn thu ln hn 2.

II. 6. Thc hnh tnh hm (n)

tnh gi tr ca hm (n) ti n ta thc hin dng lnh nh sau:


[> tau(n);

Sau du (;) n phm Enter mn hnh s hin ra kt qu.


Th d 1: Tnh (-9).
[> tau(-9);

Th d 2: Tnh (100).
[> tau(100);

9
Vy s cc c dng ca 100 l 9.

II. 7. Thc hnh tnh hm (n)


tnh gi tr ca hm (n) ti n ta thc hin dng lnh nh sau:
[>sigma(n);

57

Sau du (;) n phm Enter mn hnh s hin ra kt qu.


Th d: Tnh (9).
[>sigma(9);
13
Vy tng cc c dng ca 9 l 13.

II. 8. Thc hnh tnh ng d thc, gii phng trnh ng d


1. tnh ng d ca a theo modulo n ta thc hin dng lnh nh sau:

V
V
n
n
M
M
a
a
tth
h
..C
C
o
o
m
m

[> a mod n;

Sau du (;) n phm Enter mn hnh s hin ra kt qu.


Th d: Tnh 51234567 mod 221

[> 5&^1234567 mod 221;

112

2. gii phng trnh ng d ta thc hin dng lnh nh sau:


[>msolve (cc phng trnh, modulo);

Sau du (;) n phm Enter, nu phng trnh ng d c nghim mn hnh s hin


ra kt qu.
Th d: Tm nghim ca ng d sau:

x2+x+1 0 (mod 7)

[>msolve(x^2+x+1=0,7);

x=4, x=2

Vy nghim ca phng trnh l x=2, x=4(mod 7).

58

Chng 4.

Thng d bnh phng.

V
V
n
n
M
M
a
a
tth
h
..C
C
o
o
m
m

Gi s p l mt s nguyn t l, a l s nguyn t cng nhau vi p. Vn t ra


l: khi no a l s chnh phng modulo p? Vn ny khng ch c gi tr l
thuyt, m nh ta s thy v sau, c nhiu ng dng quan trng. nghin cu vn
t ra, cng c quan trng l cc k hiu Legendre v Jacobi m ta s xt trong
chng ny.

1. K hiu Legendre.

nh ngha 4.1. Gi s m l s nguyn dng. S a c gi l mt thng d bnh


phng ca m nu (a,m)=1 v ng d x2 a(mod m) c nghim. Nu ngc li, ta
ni a l khng thng d bnh phng ca m.
Ta s chng t rng, nu a l mt s nguyn t l, trong s cc s 1, 2, ..., p-1 c
ng mt na l thng d bnh phng.
B 4.1. Gi s p l s nguyn t l, a l s nguyn khng chia ht cho p. Khi
ng d sau y khng c nghim, hoc c ng hai nghim khng ng d
modulo p:
x2 a(mod p).

Chng minh. Gi s c nghim x=x0. Khi , d chng minh rng x=-x0 l mt


nghim khng ng d vi x0. Ta s ch ra rng, nghim tu khc x=x1 ng d
vi x0 hoc -x0.
Tht vy, ta c: x02 x12(mod p), tc l x02-x12=(x0+x1)(x0-x1) 0(mod p). Do ,
hoc p|x0+x1, hoc p|x0-x1, iu phi chng minh.
nh l 4.3. Nu p l mt s nguyn t l, th trong cc s 1, 2, ..., p-1 c ng
(p-1)/2 thng d bnh phng.
Chng minh. tm tt c cc thng d modulo p trong cc s 1,2,...,p-1, trc tin
ta bnh phng cc s v xt cc thng d dng b nht modulo p ca cc kt
qu nhn c. Cc thng d dng b nht ny l tt c cc thng d bnh phng
trong cc s t 1 n p-1. Gi s a l mt thng d nh vy. V phng trnh ng
d x2 a(mod p) c ng hai nghim, nn trong s (p-1) bnh phng ang xt,
phi c hai bnh phng thng d a: S thng d bnh phng ng bng (p-1)/2.
xt cc thng d bnh phng, ngi ta thng dng cc k hiu quan trng m
ta s nghin cu trong chng ny.
59

nh ngha 4.4. Gi s p l mt s nguyn t l v a l mt s nguyn khng chia


a
ht cho p. K hiu Legendre c nh ngha nh sau:
p
a
p =

1, nu a l thng d bnh phng ca p


-1, nu ngc li.

V d. D tnh c:
4
11 =

5
11 =

9
11 = 1.

2 6
11 = 11 =

7
11 =

8
11 =

10
11 = 1.

V
V
n
n
M
M
a
a
tth
h
..C
C
o
o
m
m

1 3
11 = 11 =

Tiu chn sau y thng c dng chng minh cc tnh cht ca k hiu
Legendre.
nh l (Tiu chun Euler). Gi s p l s nguyn t l, v a l s nguyn dng
khng chia ht cho p. Khi :
a
(p-1)/2
(mod p).
p a

a
Chng minh. Trc tin, gi s rng =1. Khi , ng d x2 a(mod p) c
p
nghim x=x0. Theo nh l Fermat b, ta c:
a(p-1)/2=(x02)(p-1)/2=x0p-1 1(mod p)

a
Ch cn phi xt trng hp =-1. Khi , ng d x2 a(mod p) v nghim.
p
Vi mi i sao cho 1 i p-1, tn ti duy nht j (1 j p-1) ij a(mod p). R rng
i j, nn ta c th nhm cc s 1, ..., p-1 thnh (p-1)/2 cp vi tch tng cp ng
d a modulo p. Nhn cc cp ny vi nhau ta c:
(p-1)! a(p-1)/2(mod p).
T nh l Wilson ta c:
-1 a(p-1)/2(mod p).
nh l c chng minh.
Nhng tnh cht sau y cho php tnh c d dng k hiu Legendre.

60

nh l 4.5. Gi s p l mt s nguyn t l, a v b l cc s nguyn khng chia ht


cho p. Khi :
a
(i) Nu a b(mod p) th =
p
a b
(ii) =
p p

b
p .

ab
p .

a 2
(iii) = 1.
p

V
V
n
n
M
M
a
a
tth
h
..C
C
o
o
m
m

Chng minh. (i). Nu a b(mod p) th x2 a(mod p) c nghim nu v ch nu


a b
x2 b(mod p) c nghim. Do = .
p p
(ii). Bi tiu chun Euler ta c:

a
(p-1)/2
(mod p),
p a

b
(p-1)/2
(mod p).
p b

ab
(p-1)/2
(mod p).
p (ab)

Nh vy,

a b
ab
(p-1)/2 (p-1)/2
b
=(ab)(p-1)/2 (mod p).
p p a

p

V gi tr ca k hiu Legendre ch c th l 1 nn ta c ng thc cn chng


minh.
a
(iii) V = 1 nn t phn trn ta c
p

a 2 a a
= = 1 .
p p p

nh l trn cho thy rng tch ca hai thng d bnh phng hoc hai khng thng
d bnh phng l mt thng d bnh phng, tch ca mt thng d bnh phng
v mt khng thng d bnh phng l mt khng thng d bnh phng.
Tiu chun Euler cho bit khi no th cc s nguyn l nhn -1 l thng d bnh
phng.

61

nh l 4.6. Nu p l s nguyn t l th

1 1, khi p 1(mod 4)
p = 1, khi p 1(mod 4)

Chng minh. Theo tiu chun Euler ta c:

1
( p 1)/ 2
(mod p).
p ( 1)

Nu p 1(mod 4) th p=4k+1 vi k nguyn no . Nh vy,

V
V
n
n
M
M
a
a
tth
h
..C
C
o
o
m
m

(-1)(p-1)/2=(-1)2k+1=-1,

1
tc l =-1.
p

nh l 4.7. (B Gauss). Gi s p l s nguyn t l v (a,p)=1. Nu s l s cc


thng d dng b nht ca cc s nguyn a,2a,...((p-1)/2)a ln hn p/2, th
a
s
p =(-1) .

Chng minh. Trong s cc thng d dng b nht ca cc s nguyn a,2a,...,
((p-1)/2)a, gi s u1,u2,...,us l cc thng d ln hn p/2, v v1,v2,...,vt l cc thng
d nh hn p/2. V (ja,p)=1 vi mi j, 1 j ( p 2) / 2 , nn tt c cc thng d
dng b nht ni trn u nm trong tp hp 1,...,p-1.
Ta s chng t rng, p-u1,..., p-us, v1,...,vt chnh l tp hp cc s 1,...(p-1)/2, xp
theo th t no . C c thy (p-1)/2 s khng vt qu (p-1)/2, nn ch cn phi
chng minh rng khng c hai s no ng d vi nhau.
R rng khng c hai s ui no, cng nh khng c hai s vj no ng d vi nhau
modulo p. Tht vy, nu ngc li, ta s c ng d ma na(mod p) vi m, n dng
no khng vt qu (p-1)/2. V (a,p)=1 nn t suy ra m n(mod p): Mu
thun.
Tng t nh trn, c th thy rng khng c p-ui no ng d vi vj.
Vy ta c:

p 1
(p-u1)...(p-us)v1...vt
!(mod p).
2

T suy ra

p 1
(-1)su1...usv1...vt
!(mod p).
2
Mt khc, v u1,...us,v1,...vt l cc thng d dng b nht ca a,2a,...,((p-1)/2)a nn

62

p 1
u1...usv1...vt a(p-1)/2
!(mod p).
2
Nh vy ta c:

p 1
p 1
(-1)s a(p-1)/2
!

!(mod p).
2
2
V (p,((p-1)/2)!)=1 nn suy ra:
(-1)sa(p-1)/2 1(mod p),

V
V
n
n
M
M
a
a
tth
h
..C
C
o
o
m
m

tc l:

a(p-1)/2 (-1)s(mod p)

nh l suy ra t tiu chun Euler.

nh l 4.8. Nu p l mt s nguyn t l th

2
2
(p -1)/8
p =(-1)

Nh vy, 2 l thng d bnh phng ca mi s nguyn t dng p 1(mod 8)v


l khng thng d bnh phng ca mi s nguyn t dng p 3(mod 8).
Chng minh. p dng tiu chun Gauss, ta cn tnh s thng d dng b nht ln
hn p/2 ca dy s
1.2,2.2,...,((p-1)/2).2

V cc s u nh hn p nn cc thng d dng b nht ca mi s trng vi


chnh n. Nh vy, ta ch cn tnh s cc s ca dy ln hn p/2. S cc s l
s=(p-1)/2-[p/4] (trong [ ] ch phn nguyn). Nh vy ta c:

2
(p-1)/2-[p/4]
.
p =(-1)

D kim tra ng d thc sau y bng cch phn ra cc trng hp


p 1,3,5,7(mod 8):
(p-1)/2-[p/4] (p2-1)/8(mod 2)
T ta c:
2
2
(p -1)/8
(mod 2).
p (-1)

Tnh ton trc tip cho ng thc cn chng minh.

63

2. Lut thun nghch bnh phng.


p
q
nh l sau y cho ta mi lin h gia cc k hiu Legendre v . nh l
q
p
ny thng c s dng khi tnh ton vi cc k hiu Legendre.
nh l 4.9. (Lut thun nghch bnh phng). Gi s p v q l cc s nguyn t l,
khi ta c:

q
((p-1)/2).((q-1)/2)
.
p =(-1)

V
V
n
n
M
M
a
a
tth
h
..C
C
o
o
m
m

p
q

Trc ht ta chng minh b sau.

B 4.10. Gi s p l mt s nguyn t l, a l mt s l khng chia ht cho p.


Khi

a
T(a,p)
p =(-1) ,

trong

( p 1)/ 2

[ ja / p] .

T(a,p)=

j =1

Chng minh. Xt cc thng d dng b nht ca cc s nguyn a,2a,...,((p-1)/2)a.


Gi s u1,...us,v1,...vt tng ng l cc thng d ln hn v b hn p/2. Ta c:
ja=p[ja/p]+ phn d

trong phn d l mt trong cc s ui hoc vj. Cng tng v (p-1)/2 phng trnh,
ta c:
( p 1)/ 2

( p 1)/ 2

j =1

j =1

ja =

j =1

j =1

p[ ja / p] + u j + v j

Nh chng t trong chng minh b Gauss, cc s nguyn p-u1,..., p-us, v1,...,vt


chnh l tp hp cc s 1,...(p-1)/2, xp theo th t no .Vy ta c:
( p 1)/ 2

j =1

j =1

j =1

j =1

j =1

j = ( p u j ) + v j = ps u j + v j

T suy ra
( p 1)/ 2

( p 1)/ 2

( p 1)/ 2

j =1

j =1

j =1

j =1

ja j = p[ ja / p] ps + 2 u

64

T cng thc ca T(a,p), ta nhn c:

(a 1)

( p 1)/ 2

j =1

j =1

j = pT (a , p) ps + 2 u

V a, p l nn
T(a,p) s(mod 2)
B c chng minh bng cch p dng b Gauss.
By gi ta chng minh Lut thun nghch bnh phng.

V
V
n
n
M
M
a
a
tth
h
..C
C
o
o
m
m

Xt cc cp s nguyn (x,y) vi 1 x (p-1) /2 v 1 y (q-1)/2. C tt c


((p-1)/2)((q-1)/2) cp nh vy. Ta s chia cc cp thnh hai nhm tu thuc
ln ca qx v py.
Trc tin, d thy rng qx py i vi mi cp.

nh s cc cp s nguyn (x,y) vi 1 x (p-1)/2, 1 y (q-1)/2 v


qx > py, ta ch rng chng chnh l cc cp vi 1 x (p-1)/2, 1 y qx/p.
Vi mi gi tr c nh ca x, 1 x (p-1)/2, tn ti [qx/p] s nguyn tho mn
1 y qx/p. Nh vy s cc cp tho mn tnh cht ang xt l

( p 1)/ 2

[qj / p] .
j =1

Tip theo, ta xt cc cp tho mn 1 x (p-1)/2, 1 y (q-1)/2 v qx < py. L


( q 1)/ 2

lun tng t nh trn cho thy, s cc cp l

[ pj / q ] .
j =1

V c tt c l ((p-1)/2)((q-1)/2) cp, ta nhn c ng thc sau


( p 1)/ 2

( q 1)/ 2

j =1

j =1

[qj / p] +

[ pj / q ] =

((p-1)/2)((q-1)/2).

T nh ngha ca hm T, ta c:

(-1)T(p,q)+T(q,p)=(-1)((p-1)/2)((q-1)/2)

nh l c suy ra t b 4.10

Nhn xt. nh l trn y (Lut thun nghch bnh phng) thng c dng
p q
tnh k hiu Legendre. Chng hn, t nh l c th suy ra rng, =-1 nu
q p

p q
p q 3 (mod 4), v bng 1 trong cc trng hp cn li, tc l = nu
q p
p q
p q 3 (mod 4), v = trong cc trng hp c t nht mt trong hai s p
q p
hoc q ng d vi 1 modulo 4.

65

713
Ta xt mt v d bng s: tnh
.
1009
713
1009 =

23.31
1009 =

23 31
1009 1009

V 1009 1(mod 4) nn ta c:

23 1009 31 1009
1009 = 23 , 1009 31


Mt khc,

V
V
n
n
M
M
a
a
tth
h
..C
C
o
o
m
m

1009 20 2 2 5 2 2 5 5 23 3 5 2
23 = 23 = = 23 = 23 = 5 = 5 = 3 = 3 = 1

23 23
1009 17 31 14 2 7 7 17 3
7
4
31 = 31 = 17 = 17 = 17 17 = 17 = 7 = 7 = 3 = 3




2 2
= = 1
3

713
Vy,
=1.
1009

Lut thun nghch bnh phng cn c dng trong kim tra nguyn t. Ta c
nh l sau.
nh l 4.11. (Kim tra Pepin). S Fermat Fm l s nguyn t khi v ch khi
3(F m 1) / 2 -1 (mod F )
m

Chng minh. Ta nhc li nh ngha s Fermat: Fm=2 2 +1.

Gi s ng d pht biu trong nh l c tho mn. Khi ta c


3F m -1 1 (mod Fm)

Nh vy, nu Fm c c nguyn t p th

3F m -1 1 (mod p)

Do , ordp3 phi l mt c
ca Fm-1, tc phi l mt lu tha ca 2. T gi thit
m1
2
suy ra ordp3 /| (Fm-1)/2=2
. Vy ta c: ordp3=Fm-1. T suy ra Fm-1 p-1,
nhng v p l c ca Fm, nn c ngha l Fm=p: Fm l s nguyn t.
Ngc li, gi s Fm nguyn t. Theo lut thun nghch bnh phng, ta c:

66

3 Fm
F = 3 =
m

2
3 = 1

Mt khc, theo tiu chun Euler ta c:

3
( Fm 1)/ 2
(mod Fm)
F 3
m
nh l c chng minh.

V
V
n
n
M
M
a
a
tth
h
..C
C
o
o
m
m

Nhn xt. Dng tiu chun Pepin, d kim tra c rng F1,F2,F3,F4 l cc s
nguyn t, F5 l hp s.

3. K hiu Jacobi.

K hiu Jacobi l mt m rng ca k hiu Legendre, v c s dng tnh k


hiu Legendre, cng nh trong nhiu vn nghin cu cc s gi nguyn t.
nh ngha 4.12. Gi s n l s nguyn dng l, a nguyn t cng nhau vi n.
Nu n c phn tch ra tha s nguyn t l p1t 1 p2t 2 ...pmt m , ta nh ngha k hiu
Jacobi nh sau:
t

1
2
m
a a a a
n = p p p ,
1 2 m

trong v phi l cc k hiu Legendre.

Nh vy, trong trng hp n l s nguyn t th k hiu Jacobi trng vi k hiu


Legendre. Tuy nhin cn ch rng, khc vi k hiu Legendre, khi n l hp s, k
hiu Jacobi khng cho ta bit phng trnh ng d x2 a(mod p) c nghim hay
khng. Mc du vy, k hiu Jacobi c nhiu tnh cht tng t vi k hiu
Legendre.
nh l 4.13. Gi s n l s nguyn dng l, a v b l cc s nguyn t cng nhau
vi n. Khi :

a
(i) Nu a b(mod n) th =
n
ab
(ii) =
n

b
n .

a b
n n

1
(iii) =(-1)(n-1)/2
n
67

2
2
(iv) =(-1)(n -1)/8
n

Chng minh. Hai ng thc u tin d suy ra t nh ngha k hiu Jacobi v tnh
cht ca k hiu Legendre.
chng minh tnh cht th 3, ta nhn xt rng, do (pi-1) chn nn
(1+(pi-1))t1 1+ti(pi-1)(mod 4),
(1+ti(pi-1))(1+tj(pj-1)) 1+ti(pi-1)+ tj(pj-1)(mod 4).
T suy ra:

V
V
n
n
M
M
a
a
tth
h
..C
C
o
o
m
m

n 1+t1(p1-1)+t2(p2-1)+...+ tm(pm-1)(mod 4),

tc l,

(n-1)/2 t1(p1-1)/2+t2(p2-1)/2+...+ tm(pm-1)/2(mod 2)

H thc ny cng vi nh ngha cho ta ng thc (iii).


Chng minh (iv). Ta c:
t

1
2
m
2 2 2
2
t1 ( p12 1)/ 8 + t 2 ( p2 2 1)/ 8 + ...+ t m ( pm 2 1)/ 8
n = p p ... p = ( 1)
1 2
m

Lp lun tng t nh trong chng minh phn trn,ta c:

n2 1+t1(p12-1)+t2(p22-1)+...+ tm(pm2-1)(mod 64),

v khi (iv) suy ra t nh ngha.

nh l 4.14. (Lut thun nghch bnh phng i vi k hiu Jacobi). Gi s m,


n l cc s nguyn dng l, nguyn t cng nhau. Khi :
m1 n 1
n m
2 2
=

(
1
)
.
m n

Chng minh. Gi s m, n c phn tch ra tha s nguyn t dng:


m=p1a 1 p2a 2 ...psa s , n=q1b 1 q2b 2 ...qrb r . Dng nh ngha v lut thun nghch bnh
phng ca k hiu Legendre, ta c:

n m
m n = ( 1)
i =1 j =1
r

p j 1 qi 1
aj
bi
2
2

Nh trong chng minh nh l 4.13, (iii), ta c:

68

= ( 1) i =1 j =1

aj

p j 1
2

q 1
bi i
2

pj 1

a
j =1
r

i =1

m1
(mod 2),
2

qi 1 n 1

(mod 2).
2
2

T suy ra nh l.

4. Thut ton tnh k hiu Jacobi.

V
V
n
n
M
M
a
a
tth
h
..C
C
o
o
m
m

Gi s a, b l hai s nguyn dng nguyn t cng nhau, a>b. t R1=a, R2=b.


Dng thut chia Eulid v tch lu tha cao nht ca 2 trong phn d, ta c:
R0 = R1q1 + 2s 1 R2

R 1 = R 2 q 2 + 2 s2 R 3

................................

R n-3 = R n-2 q n-2 + 2 sn-2 R n-1


R n-2 = R n-1q n-1 + 2 sn-1 .1

trong sj l cc s nguyn khng m, Rj l s nguyn l b hn Rj-1.

Ta ch rng, s cc php chia i hi trong thut ton trn l khng vt qu s


php chia cn thit khi dng thut ton Euclid tm c chung ln nht ca hai s
a v b.
t:

R 1
R 1
R
1 R1 1 R2 1
R 1 Rn 1 1
R (a , b) = s1 1
+ s2 2
+...+ sn 1 n 1
+
.
+...+ n 2
.
.
8
8
8
2
2
2
2

Ta c nh l sau.

nh l 4.15. Gi s a,b l cc s nguyn dng v a>b. Khi ta c:


a
R(a,b)
b =(-1)

Chng minh. Theo cc phn (i), (ii), v (iv) ca nh l 4.13 ta c:


a
b =

R0 2 s1 R2
=
R =
1 R1

1
R 1
s1 1
R2
2 R2
8
1
(
)
=

R R
R .
1 1
1

Dng lut thun nghch bnh phng ca k hiu Jacobita c:

69

R1 1 R2 1
R2
R1
2
2
R = ( 1)
R .
1
2

Nh vy,
2

R1 1 R2 1
R 1
+ s1 1
a
R1
2
2
8
b = ( 1)
R

2

Tip tc qu trnh , ta i n cng thc cn chng minh.

V
V
n
n
M
M
a
a
tth
h
..C
C
o
o
m
m

H qu 4.16. Gi s a v b l cc s nguyn dng nguyn t cng nhau, a>b. Khi


a
, k hiu Jacobi c th tnh c vi O((log2b)3) php tnh bit.
b
Chng minh. Nh ta nhn xt, s cc php chia trong thut ton xc nh R(a,b)
khng vt qu s php chia trong thut ton Euclid tnh c chung ln nht ca
a v b. Theo nh l Lam, cn c O(log2b) php chia. Mi php chia cn khng
qu O((log2b)2) php tnh bit. Sau mi php chia, cp s Rj, sj tm c bi O(log2b)
php tnh bit (ch cn l cc php dch chuyn). Nh vy, khi bit a, b, ch cn
O((log2b)3) php tnh bit xc nh cc s Rj, sj. nng (-1) ln lu tha R(a,b)
nh trong nh l, ta ch cn s dng 3 ch s nh phn cui cng ca Rj v ch s
nh phn cui cng ca sj, v gi tr lu tha ca (-1) ch ph thuc vo tnh chn l
a
ca s m. Nh vy, khi c Rj, sj, ta ch cn O(log2b) xc nh . H qu
b
c chng minh.
Ta c thut ton sau y tnh k hiu Jacobi da vo cc nh l va chng minh.
a

Thut ton tnh k hiu Jacobi (v do , tnh k hiu Legendre khi b l


b
s nguyn t).

J1. (Kim tra b 0). Nu b=0, in ra 0 nu |a| 1, in ra 1


nu |a|=1 v kt thc thut ton.
J2. (Tch cc lu tha ca 2 khi b). Nu a v b u
chn, in ra 0 v kt thc thut ton. Ngc li, t
v 0, v khi b chn, t v v+1, b b/2.
Sau , nu v
2
chn, t k 1, ngc li, t k (-1)(a -1)/8. Cui cng,
nu b<0, t b -b, v nu hn na, a<0, t k -k.
J3. (Kt thc?). ( bc ny, ta c b l v b>0). Nu
a=0, in ra 0 nu b>1, in ra k nu b=1 v kt thut
ton. Ngc li, t v 0 v nu a chn, t v v+1,
2
a a/2. Nu v l, t k (-1) (b 1)/8 k .
J4. (S dng lut thun nghch). t k (-1)(a-1)(b-1)/4k.

70

Nhn xt. y, ta cn lu mt iu. Mc d trong thut ton c xut hin cc


php chia (a2-1)/8, (b2-1)/8, (a-1)(b-1)/4, v php nng (-1) ln lu tha , ta
khng cn lm cc php chia cng nh nng ln lu tha, v i hi qu nhiu php
tnh bit. V gi tr lu tha ca (-1)2ch ph thuc vo tnh chn l ca cc i lng
trn, nn chng hn i vi (-1)(a -1)/8, gi tr ch ph thuc a mod 8 v bng
mt trong nhng s ca dy sau y:
{0,1,0,-1,0,-1,0,1}.

Thut ton tnh cn bc 2 modulo p.

V
V
n
n
M
M
a
a
tth
h
..C
C
o
o
m
m

Trong nhiu ng dng (chng hn, xem Chng 7), ta cn phi tnh cn bc 2
modulo p, khi bit n tn ti. Tt nhin, mt trong cc phng php gii phng
trnh ng d x2 a(mod p), (a,p)=1 l kim tra tt c cc s t 1 n p-1. Tuy
nhin, khi lm vic vi p ln, phng php ny khng th p dng c (thi gian
i hi l O(p)).
Vi nhng s nguyn t dng p 3(mod 4), bi ton kh n gin. Ta c:
x a(p+1)/4(mod p).

Tht vy,

x a(p+1)/2 a.a(p-1)/2 a(mod p) .

Khi p / 3(mod 4), ta c p 1(mod 8) hoc p 5(mod 8). Trong trng hp


p 5(mod 8), li gii cng c th tm c khng kh khn. Tht vy, ta c:
a(p-1)/2 1(mod p),

do

a(p-1)/4 1(mod p).

D kim tra c rng, trong trng hp ng d tho mn vi du cng, nghim


phi tm l
x=a(p+3)/8(mod p).

Nu ng d tho mn vi du tr, dng nh l 4.8 ta c:


a(p-1)/2 -1(mod p).

T nghim phi tm l:
x=2a.(4a)(p-5)/8(mod p).
Nh vy ch cn phi xt trng hp p 1(mod 8). Cho n nay, mi ch c mt
thut ton (thut ton Shoof s dng ng cong elliptic) vi thi gian a thc. Tuy
nhin, trong thc t, thut ton rt kh s dng. Sau y chng ta tm hiu thut
ton xc sut ca Tonelli v Shanks.
Thut ton Tonelli-Shanks chnh l mt m rng t nhin ca cc trng hp ring
xt trn y.
71

Ta lun lun vit p-1=2e.q, vi q l.


Nu ta tm c phn t z v s nguyn chn k sao cho
aqzk 1(mod p)
th nghim cn tm s c cho bi
x=a(q+1)/2zk/2.
Ta s tm phn t z di dng z=nq.
Ta ch ra rng, phn t z nh vy tho mn iu kin t ra khi v ch khi n l mt
khng thng d bnh phng modulo p. Ta c:
e

V
V
n
n
M
M
a
a
tth
h
..C
C
o
o
m
m

(aq)2 = a ( p 1) 1(mod p),


do aq thuc vo nhm G cc phn t cp l mt c s ca 2e. Nh vy, tn
ti k, ch cn chn n l phn t sinh ca nhm G (khi , do a l mt khng thng
d bnh phng nn s m k phi
l chn). S nguyn n s l mt phn t sinh ca
e
2
4
2
G khi v ch khi n, n , n ,...,n ( 1(mod p)) khng ng d vi nhau modulo p. D
thy rng, iu xy ra khi v ch khi n l mt khng thng d bnh phng
modulo p.
xy dng thut ton, ta cn gii quyt hai vn : Tm phn t z, v tm s m k.
Phn th nht c gii quyt bng thut ton xc sut. Ta chn ngu nhin mt s
n
n
n, v tnh k hiu Legendre . Khi , nu =-1, ta t z=nq. Trong trng
p
p
hp ngc li, ta tip tc lm nh trn vi mt s ngu nhin khc cho n khi tm
c mt s n thch hp. V s cc thng d bnh phng bng (p-1)/2 nn mi ln
n
chn ngu nhin mt s n, xc sut c =-1 l 1/2.
p
Trong thc t, ta c th tm ra mt khng thng d bnh phng rt nhanh. Chng
hn, xc sut hai mi ln tht bi lin tip nh hn 10-6.
S m k kh tm hn. Tht ra, ta khng cn bit s m k, m cn bit a(q+1)/2zk/2.
Thut ton. Gi s p l mt s nguyn t l, n Z. Ta vit
p-1=2e.q vi q l.
1. (Tm phn t sinh). Chn ngu nhin s n
n
khi tho mn =-1. Sau t z nq(mod p).
p
2. (Xut pht).
b ax2(mod p ),

cho n

y z, r e, x a(p-1)/2(mod
x ax(mod p).

p),

3. (Tm s m). Nu b 1(mod p) in ra x v kt thc


thut ton, Trong trng hp ngc li, tm s m nh
m
nht sao cho m 1, b 2 1(mod p). Nu m=r, in ra thng
72

bo ni rng
modulo p.

a khng phi l thng d bnh phng


r m 1

4. (Thu hp s m). t t y 2
, y t2, r m, x xt,
b by (mi php tnh u modulo p)v chuyn sang bc
3.
Ch rng t khi bt u bc 3, ta lun lun c cc ng d modulo p:
ax x2, y 2

r 1

-1, b 2

r 1

1.

V
V
n
n
M
M
a
a
tth
h
..C
C
o
o
m
m

T suy ra rng, nu nhm con Gr cc phn t cp l mt c ca 2r, th y l phn


t sinh ca nhm Gr, b Gr-1, tc l b chnh phng trong Gr. V r thc s gim ti
mi bc lp ca thut ton, nn s bc lp nhiu nht bng e. Khi r 1, ta c
b=1, thut ton kt thc, v x l mt cn bc 2 ca a mod p.
C th thy rng, trung bnh, bc 3 v bc 4 i hi e2/4 php nhn modulo p, v
nhiu nht l e2 php nhn. Nh vy, thi gian chy thut ton l O(log4p).

5. S gi nguyn t Euler.

Gi s p l s nguyn t l v b l s nguyn khng chia ht cho p. Khi theo tiu


chun Euler ta c:
b
b(p-1)/2 (mod p).
p

Nh vy, kim tra mt s n c phi l nguyn t hay khng, ta c th ly mt


s b nguyn t cng nhau vi n, v kim tra xem ng d sau y c ng hay
khng:
b
b(n-1)/2 (mod n),
n

trong , v bn phi l k hiu Jacobi. Nu ng d thc khng ng th n phi


l hp s. Nu ng d thc trn y nghim ng, vn cha kt lun c n c
phi l nguyn t hay khng, nhng c nhiu kh nng n l s nguyn t.
nh ngha 4.18. S nguyn dng n c gi l s gi nguyn t Euler c s b
nu n l mt hp s v ng d thc sau y nghim ng:
b
b(n-1)/2 (mod n)
n

Ta c mi lin h gia s gi nguyn t Euler c s b v s gi nguyn t c s b


xt trong chng 2.

73

nh l 4.19. Mi s gi nguyn t Euler c s b u l s gi nguyn t c s b.


Chng minh. Ch cn bnh phng hai v ca ng d thc tho mn bi cc s gi
nguyn t Euler.
iu ngc li khng ng. Chng hn, c th thy rng s 431 l s gi nguyn t
c s 2, nhng khng l s gi nguyn t Euler c s 2.
nh l 4.20. Mi s gi nguyn t mnh c s b u l s gi nguyn t Euler c
s b.
Chng minh. Cho n l s gi nguyn t mnh c s b. Khi , nu n-1=2st, trong
r
t l, th, hoc bt 1(mod n), hoc b 2 t -1(mod n), vi r no 0 r s-1. Gi
m

aj

l phn tch ca n thnh tha s nguyn t. Ta xt ring hai trng hp.

V
V
n
n
M
M
a
a
tth
h
..C
C
o
o
m
m

j =1

Th nht, bt 1(mod n). Gi s p l mt c nguyn t ca n. Khi ordpb|t, v do


ordpb l s l. Mt khc, ordpb l c ca (p)=p-1, nn n phi l c ca
(p-1)/2. Vy ta c
b(p-1)/2 1(mod p)

b
b
p =1, v do , n =1. Mt khc ta c:


1(mod p). Vy n l s gi nguyn t Euler c s b.

Theo tiu chun Euler,


b(n-1)/2=(bt) 2

Trng hp th hai: b 2 t -1(mod n). Nu p l mt c nguyn t ca n th


r
b 2 t -1(mod n).
Bnh phng c hai v ca dng d thc ny ta c
b2

r +1

1(mod p).

T suy ra ordpb|2r+1t, nhng ordpb khng l c ca 2rt. Nh vy, ordpb=2r+1c,


trong c l mt s nguyn l. Mt khc, v ordpb|(p-1), 2r+1|ordpb, nn
2r+1|(p1).
Nh vy, ta c: p=2r+1d+1, trong d l s nguyn. V

( ord pb )/2

-1(mod p)

nn ta c:
r +1
b
( ord b / 2 )(( p 1)/ ord pb )
( p 1)/ ord pb
(p-1)/2
=b p
( 1)
= ( 1) ( p 1)/ 2 c (mod p)
p b

b
V c l nn t suy ra =(-1)d.
p

By gi gi s n c phn tch thnh tha s nguyn t dng:


74

n= p j a j .
j =1

Theo chng minh phn trn, cc c nguyn t pi c dng pi=2r+1di+1, v ta c:


a

m
m
b
b
n = p = ( 1) ai d i .
i =1
i =1 i i

Mt khc, d thy rng,


m

n 1+2r+1 ai d i (mod 22r+2).


i =1

V
V
n
n
M
M
a
a
tth
h
..C
C
o
o
m
m

Do

t2s-1=(n-1)/2 2r ai d i (mod 2r+1),


i =1

tc l

t2s-1-r

a d
i

(mod 2)

i =1

b(n-1)/2= (b 2 t ) 2

s 1 r

( 1) 2

s 1 r

= ( 1) ai d i (mod n)
i =1

Nh vy,

b
b(n-1)/2 (mod n),
n

v n l s gi nguyn t Euler c s b.

Ch rng, iu ngc li khng phi lun lun ng: tn ti nhng s gi nguyn


t Euler c s b khng l gi nguyn t mnh c s . V d n=1105, b=2.
Tuy nhin, vi nhng iu kin b sung, mt s gi nguyn t Euler s l gi
nguyn t mnh cng c s. Ta c nh l sau.
nh l 4. 21. S n gi nguyn t Euler c s b l s gi nguyn t mnh c s b
b
nu n 3(mod 4), hoc =-1.
n
Chng minh. Trng hp th nht: n 3(mod 4). Khi n-1=2.t v t l. V n l s
gi nguyn t Euler c s b nn

75

b
bt=b(n-1)/2 (mod n).
n

Nh vy, n l s gi nguyn t mnh c s b.


Trong trng hp th hai, ta vit n-1=2st, trong t l, s l s nguyn dng. V n
l s gi nguyn t mnh c s b nn
b2

s 1

b
= b ( n 1)/ 2 (mod n).
n

Theo gi thit ta c:
s 1

-1(mod n).

V
V
n
n
M
M
a
a
tth
h
..C
C
o
o
m
m

bt2

Nh vy n l s gi nguyn t mnh c s b.

Dng s gi nguyn t Euler, ta c th xy dng thut ton xc sut kim tra


mt s l nguyn t hay khng. Thut ton ny c Solovay v Strassen tm ra
u tin nm 1977 ([S-S]).
Ta bt u bng b sau.

B 4.22. Gi s n l mt s nguyn dng l khng chnh phng. Khi tn ti


b
t nht mt s b vi 1<b<n, (b,n)=1, sao cho =-1.
n
Chng minh. Nu n l nguyn t, s b tn ti theo nh l 4.3. Khi n l hp s
khng chnh phng, ta vit n=rs, trong (r,s)=1 v r=pe, vi p l mt s nguyn
t l v e s nguyn dng l. By gi gi s t l mt khng thng d bnh phng
ca s nguyn t p. Ta dng nh l Trung Quc v phn d tm s nguyn b sao
cho 1<b<n, (b,n)=1 v
b t(mod r)

b 1(mod s)

b
Khi ta c =
r

b
b
e
p e = ( 1) = 1, s = 1 , tc l

b
n =-1.

B 4.23. Vi mi hp s l n, tn ti t nht mt s b sao cho 1<b<n, (b,n)=1


v

b
b(n-1)/2 / (mod n).
n

(3.1)

Gi s ngc li, vi mi s nguyn khng vt qu n v nguyn t cng nhau vi


n, ta c

76

b
b(n-1)/2 (mod n).
n
T suy ra, nu (b,n)=1 th
b(n-1) 1(mod n).
Nh vy, n phi l s Carmicheal, v do , n=q1q2...qr l tch ca cc s nguyn t
l khc nhau. Ta s ch ra rng
b(n-1)/2 1(mod n).
i vi mi s nguyn b khng vt qu n v nguyn t cng nhau vi n.

V
V
n
n
M
M
a
a
tth
h
..C
C
o
o
m
m

Gi s ngc li, tn ti b tho mn


b(n-1)/2 -1(mod n).

Dng nh l Trung Quc v phn d, ta tm c s a, 1<a<n, (a,n)=1 sao cho


a b(mod q1)

a 1(mod q2q3...qr)

Nh vy

a(n-1)/2 b(n-1)/2 -1(mod q1)


a(n-1)/2 1(mod q2q3...qr)

Do

a(n-1)/2 / 1(mod n),

tri vi gi thit phn chng (3.1).

Nh vy, vi mi b, 1<b<n, (b,n)=1 ta c:

b(n-1)/2 1(mod n).

T ng d trn v (3.1) ta c:

b
b(n-1)/2 1(mod n),
n

mu thun vi b 4.22. B 4.23 c chng minh.


nh l trn y c dng lm c s cho mt thut ton kim tra nguyn t xc
sut. Ta c nh l sau.
nh l 4.24. i vi mi hp s l n, tn ti khng qu (n)/2 s nguyn dng b
nh hn n, nguyn t cng nhau vi n, sao cho n l s gi nguyn t mnh Euler c
s b.
Chng minh. Theo b 4.23., tn ti s b, 1<b<n, (b,n)=1 sao cho
77

b
b(n-1)/2 / (mod n).
n
Gi s a1,a2,...,am l cc s tho mn 1 aj<n, (aj,n)=1 v

a j
aj(n-1)/2 =1(mod n)
n
Gi s r1,r2,...,rm l thng d dng b nht ca cc s ba1,ba2,...,bam. Cc s rj khc
nhau v nguyn t cng nhau vi n. Ta s chng t rng chng khng tho mn
ng d thc nh i vi cc s aj. Tht vy, nu ngc li

V
V
n
n
M
M
a
a
tth
h
..C
C
o
o
m
m

r j
r j(n-1)/2 = 1(mod n)
n

th ta c:

ba j
ba (n-1)/2
= 1(mod n)
j
n

v nh vy:

b a j
b (n-1)/2 a (n-1)/2

j
n n

T suy ra:

b
b(n-1)/2 (mod n),
n

mu thun vi tnh cht ca b.

Nh vy, tp hp cc s aj v rj khng giao nhau. Gp c hai tp hp ny, ta c


2m s khc nhau, b hn n v nguyn t cng nhau vi n. T suy ra m< (n)/2,
nh l c chng minh.
Nhn xt. T nh l trn, ta thy rng, nu n l mt hp s l, b l s chn ngu
nhin trong cc s 1,2,...,n-1, th xc sut n l gi nguyn t Euler c s b s b
hn 1/2. Ta c nh l sau.
nh l 4.25. (Thut ton kim tra nguyn t xc sut Solovay-Strassen). Cho n
l mt s nguyn dng. Ta chn ngu nhin k s b1,b2,...,bk t cc s 1,2,...,n-1.
i vi mi s nguyn bj, xt ng d thc

78

b j
b (n-1)/2
(mod n)
j
n
-Nu mt trong cc ng d thc khng nghim ng th n l hp s.
-Nu n l nguyn t th mi ng d thc u nghim ng.
-Nu n l hp s, th xc sut mi ng d thc nghim ng l b hn 1/2k.
Nh vy, nu k ln, v n tri qua c kim tra xc sut trn y, th hu nh
chc chn n l s nguyn t.

V
V
n
n
M
M
a
a
tth
h
..C
C
o
o
m
m

Nhn xt. 1) V mi s gi nguyn t mnh c s b u l s gi nguyn t Euler


c s b, nn s cc hp s n tri qua c kim tra xc sut Solovay-Strassen ln
hn s cc hp s tri qua c kim tra Rabin. C hai thut ton kim tra ny u
cn O(k(log2n)3) php tnh bit.
2) Chng hn, nu n l s tri qua kim tra xc sut Solovay-Strassen vi k=40. Khi
n l hp s vi xc sut nh hn 2-40 tng ng 10-12, b hn xc sut phn
cng my tnh mc mt sai lm!

79

Bi tp v tnh ton thc hnh chng 4


I. Bi tp
4.1. Tm tt c cc s t nhin b sao cho 15 v 21 l cc s gi nguyn t c s b.
4.2. Chng minh rng tn ti 36 c s b (modulo 91) 91 l s gi nguyn t c
s b.
4.3. Gi s p v 2p-1 u l s nguyn t. Chng minh rng n=p(2p-1) l s gi
nguyn t i vi mt na s c s b.

V
V
n
n
M
M
a
a
tth
h
..C
C
o
o
m
m

4.4. Chng minh rng tn ti v hn s nguyn t dng 4k+1.

4.5. Chng minh rng tn ti v hn s nguyn t c cc dng sau y: a) 8k+3,


b)8k+5, c) 8k+7.
4.6. Chng minh rng nu p l mt s nguyn t dng 4k+3 v q=2p+1 cng l s
nguyn t, th q|Mp=2p-1.
4.7. Chng minh rng 23|M11, 47|M23,503|M251.

4.8. Chng minh rng 1105 l s gi nguyn t Euler c s 2 v khng gi nguyn


t mnh c s 2.
4.9. Chng minh rng 15841 l: a) s gi nguyn t mnh c s 2; b) s gi nguyn
t Euler c s 2; c) s Carmichael.
4.10. Chng minh rng nu n l s gi nguyn t mnh Euler c s a v b th n
cng l s gi nguyn t mnh Euler c s ab.
4.11. Chng minh rng nu n l s gi nguyn t Euler c s 2, v nu n 5(mod 8)
th n l s gi nguyn t mnh c s 2.
4.12. Chng minh rng nu n l s gi nguyn t Euler c s b th n cng l s gi
nguyn t Euler c s n-b.
4.13. Chng minh rng nu n l s gi nguyn t Euler c s 3 v n 5(mod 12) th
n l s gi nguyn t mnh c s 3.

II. Thc hnh trn my tnh

II. 1. Thc hnh kim tra mt s l thng d bnh phng


Cho a, b l cc s nguyn. kim tra xem a c phi l thng d bnh phng ca
b hay khng ta thc hin dng lnh nh sau:
[>quadres(a, b);
Sau du (;) n phm Enter. Nu trn mn hnh hin ln s 1 th a l thng d
bnh phng ca b, nu trn mn hnh hin ln s -1 th khng phi.

80

Th d: 74 c phi l thng d bnh phng ca 101 hay khng?


Ta thc hin lnh
[>quadres(74,101);
-1
74 khng phi l thng d bnh phng ca 101

II. 2. Thc hnh tnh k hiu Legendre


Cho a l s nguyn, p l s nguyn t. tnh k hiu Legendre ca a v p ta thc
hin lnh nh sau:

V
V
n
n
M
M
a
a
tth
h
..C
C
o
o
m
m

[legendre(a,p);
Sau du (;) n phm Enter trn mn hnh s xut hin kt qu.

9
Th d: Tnh .
11
Ta thc hin lnh

[legendre(9,11);

Ch : Khi thc hin lnh tnh l hiu Legendre, my tnh s cho kt qu l 0, 1,


hoc -1. Nu kt qu l 0 th a chia ht cho p. Nu kt qu l 1 th a l thng d
bnh phng ca p. Nu kt qu l -1 th a khng l thng d ca p. Do ta cng
c th dng dng lnh trn kim tra thng d bnh phng.

II. 3. Tnh k hiu Jacobi

Cho b l s nguyn dng l, a nguyn t cng nhau vi b. tnh k hiu Jacobi


ca a v b ta thc hin dng lnh nh sau:
[jacobi(a, b);

Sau du (;) n phm Enter trn mn hnh s xut hin kt qu.

26
Th d: Tnh
35
Ta thc hin lnh:

[> jacobi(26,35);
-1

28
Th d: Tnh
21
Ta thc hin lnh:

81

[> jacobi(28,21);
0
Nu kt qu l 0 th a v b khng nguyn t cng nhau.

II. 4. Tm cn bc 2 modulo mt s
Cho x, n l cc s nguyn. tm cn bc 2 ca x modulo n ta thc hin dng
lnh nh sau:
[>msqrt(x,n);
Sau du (;) n phm Enter trn mn hnh s xut hin kt qu. Nu cn khng tn
ti trn mn hnh s xut hin ch FAIL.

V
V
n
n
M
M
a
a
tth
h
..C
C
o
o
m
m

Th d: Tnh cn bc 2 ca 3 modulo 11.


Ta thc hin nh sau:
[>msqrt(3,11);

Th d: Tnh cn bc 2 ca 3 modulo 7.
Ta thc hin nh sau:
[>msqrt(3,7);

FAIl

II. 5. Thc hnh kim tra s gi nguyn t Euler

kim tra s nguyn dng n cho trc c phi l s gi nguyn t Euler c s b


hay khng ta thc hin theo cc bc sau:
Bc 1: Kim tra tnh nguyn t ca n, ta thc hin bng dnglnh
[>isprime(n);

Sau du (;) n phm Enter. Nu trn mn hnh xut hin ch true th n l s


nguyn t, khi ta khng nh n khng phi l s gi nguyn t Euler c s b.
Nu trn mn hnh xut hin ch false ta tip tc thc hin bc 2.

b
Bc 2: Tnh k hiu Jacobi J:= ca n v b, thc hin bng dng lnh
n
[> J:= jacobi(b,n);
Sau du (;) n phm Enter.

Bc 3: Kim tra ng d thc b ( n 1)/ 2 J (mod n) , thc hin bng dng lnh
[>b^((n-1)/2)-J mod n;

82

Sau du (;) n phm Enter. Nu trn mn hnh xut hin s 0 th n l s gi


nguyn t Euler c s b.
Th d: S 1105 c phi l s gi nguyn t Euler c s 2 hay khng?
Ta thc hin lnh nh sau:
[> isprime(1105);
false
[> J:=J(1105,2);
1

V
V
n
n
M
M
a
a
tth
h
..C
C
o
o
m
m

[> 2^((1105-1)/2)-J mod 1105;


0

Vy 1105 l s gi nguyn t Euler c s 2.

83

Chng 5

Trng v a thc
1. nh ngha.

V
V
n
n
M
M
a
a
tth
h
..C
C
o
o
m
m

Mt trong nhng khi nim c bn ca I s v s hc l cc trng hu hn, Trong


chng ny, Chng ti s trnh by nhng kin thc c bn nht v cc trng hu
hn, cn thit khi tm hiu nhng ng dng mi ca s hc. Ngoi ra, chng ti c
gng minh ho mt trong nhng ng lc ca s pht trin s hc hin i: s
tng t gia s v a thc. Mt vi kt qu gn y v s tng t s c
cp ti.
tin li cho bn c khi s dng, chng ti nhc li y nhng khi nim cn
thit.
Trng l mt tp hp K c qu mt phn t, c trang b hai php tnh cng v
nhn tho mn cc quy tc sau y (cc ch ci la tinh ch cc phn t tu ca
trng)
1. (tnh cht giao hon ca php cng): a+b=b+a

2. (tnh cht kt hp ca php cng): a+(b+c)=(a+b)+c


3. (tn ti phn t 0): Tn ti 0 K sao cho 0+a=a+0=a

4. (tn ti -a): Tn ti -a K sao cho a+(-a)=0

5. (tnh cht giao hon ca php nhn): ab=ba

6. (tnh cht kt hp ca php nhn): a(bc)=(ab)c


7. (tn ti n v): Tn ti 1 K sao cho 1a=a

8. (tn ti a-1): Tn ti a-1 K sao cho aa-1=1

9. (lut phn b ca php nhn i vi php cng): a(b+c)=ab+ac

Nhng v d thng gp nht l: trng Q cc s hu t, trng R cc s thc, trng


C cc s phc. Cc trng u c v hn phn t.
Trong nhiu vn l thuyt cng nh ng dng, ta thng lm vic vi cc trng
ch c hu hn phn t. Chng hn, c th thy rng, cc thng d khng m b nht
modulo p, lp thnh mt trng c p phn t. Sau y, ta s thy rng, chnh l
trng c bn xy dng nn tt c cc trng hu hn.
Gi s p l s nguyn t. K hiu qua Fp trng c p phn t. R rng khi cng p ln
phn t 1 ca trng, ta c 0. Do , pa=0 vi mi phn t a Fp. Vi mt trng
84

K tu , s p khng m b nht sao cho p1=0 c gi l c trng ca trng K.


Chng hn, Q, R, C l cc trng c c trng 0, Fp l trng c c trng bng p.
D thy rng, mi trng hu hn u c c trng khc khng, v c trng ca n
l mt s nguyn t.

2. M rng trng.

V
V
n
n
M
M
a
a
tth
h
..C
C
o
o
m
m

Gi s k K l cc trng, ng thi cc php cng v nhn trong k chnh l cm


sinh bi cc php tnh tng ng trong K. Khi , K c gi l m rng ca trng
k.
V d. C l m rng ca R, R l m rng ca Q.

Nu tn ti cc phn t 1, 2,..., n K sao cho mi phn t ca a K u c th


biu din di dng
a=k1 1+k2 2+...+ kn n

trong k1,k2,...,kn l cc phn t ca trng k, th K c gi l m rng hu hn


ca k. Trong trng hp ny, K l mt khng gian vect hu hn chiu trn k. Ta ni
K l m rng hu hn ca k sinh bi 1, 2,..., n.
V d. C l m rng ca trng R, sinh bi phn t i, ni cch khc, sinh bi nghim
cu phng trnh x2+1=0.
Ta ni C l trng nng ca R bi a thc P(x)=x2+1. Sau y, ta s thy rng, mi
m rng hu hn ca cc trng u c thc hin bng cch tng t nh trn.
nh ngha 5.1. Gi s K l mt m rng ca k. Phn t a K c gi l i s trn
trng k nu n l nghim ca mt a thc vi h s trn trng k.
Nu thm iu kin h s ca lu tha cao nht bng 1 th a thc xc nh duy nht
i vi mi phn t i s trn k. K c gi l trng nng ca k bi a thc P(x)
nu n l m rng ca k bi cc nghim ca a thc P(x).
i vi cc a thc, ta cng c cc tnh cht hon ton tng t nh i vi cc s
nguyn.
i vi mt trng k tu , ta k hiu qua k[x] vnh cc a thc vi h s trong k. a
thc P c gi l chia ht cho a thc Q nu tn ti a thc R sao cho P=QR. Mt
a thc khng chia ht cho a thc no bc nh hn, khc hng s c gi l a
thc bt kh quy. Hai a thc khng c c chung no khc hng c gi l nguyn
t cng nhau. Mt a thc trong k[x] lun lun phn tch c thnh tch ca cc a
thc bt kh quy. Phn tch l duy nht, nu i hi cc a thc ban u cng nh
a thc trong khai trin u c h s ca lu tha cao nht bng 1.
i vi mi a thc P(x) trn trng k, bao gi cng tn ti mt trng K m rng
ca k sao cho P(x) phn tch c thnh cc a thc bc nht trn K. Nh vy, nu
h s ca lu tha cao nht trong P(x) l 1 th P(x) c phn tch di dng:

85

P(x)=(x- 1)(x- 2)...(x- n), vi i K, i=1,...,n.


Nu i vi a thc h s trong k, phn tch trn y c c vi i k, trng k
c gi l ng i s. Ni cch khc, trng ng i s l trng cha mi
nghim ca cc a thc vi h s trong trng . Nh vy, trong trng ng i
s, cc a thc bt kh quy ch c th l a thc bc nht. Chng hn trng s phc
C l trng ng i s, trng cc s thc R khng ng i s. Thng ca hai a
thc vi h s trong k c gi l hm hu t trn k.

V
V
n
n
M
M
a
a
tth
h
..C
C
o
o
m
m

3. Trng hu hn.
Nh ni, trng gm hu hn phn t c c trng khc khng, v c trng l
mt s nguyn t p.
Gi s Fq l mt trng hu hn gm q phn t, c trng p. V Fq cha phn t 1
nn n s cha trng Fp nh mt trng con. Do Fq l trng hu hn nn n l m
rng hu hn ca Fp, ngha l mt khng gian vect r chiu trn Fp. T suy ra
rng Fq gm pr phn t, tc l q=pr.
Ngc li, ta s chng t rng, vi p, r cho trc (p l s nguyn t v r l s nguyn
dng), tn ti trng vi pr phn t. Hn na, cc trng hu hn vi s phn t
nh nhau s ng cu vi nhau, ngha l c tng ng 1-1 gia chng, v tng ng
ny bo ton cc php tnh cng v nhn, phn t 0 v phn t nghch o ca
trng.
Ta c nh l sau.

nh l 5.2. Gi s Fq l trng hu hn vi q=pr phn t. Khi , mi phn t ca


Fq u tho mn phng trnh
Xq-X=0,

v Fq chnh l tp hp cc nghim ca phng trnh . Ngc li, trng nng ca


Fp bi a thc Xq-X l trng hu hn c q phn t.
Chng minh nh l ny hon ton tng t nh chng minh nh l 3.27 Chng 3,
v c dnh cho c gi .
Gi s Fq l trng c q phn t. Ta k hiu qua Fq* tp hp cc phn t khc khng
ca trng Fq. Khi , mi phn t ca Fq* u c nghch o, v Fq* lp thnh mt
nhm Aben. V Fq* c hu hn phn t, nn i vi mt phn t tu a Fq*, tn
ti s nguyn khng m k sao cho ak=1. S k b nht tho mn tnh cht c gi
l bc ca phn t a.
i vi mi phn t a tu , bc ca a lun l mt c ca q-1. Chng minh iu
ny cng hon ton tng t nh khi chng minh bc ca mt s modulo n l c
ca (n) (xem h qu 3.20 Chng 3).

86

Gi s g l mt phn t ca Fq*, v bc ca g ng bng q-1. Khi , tp hp


{g,g2,...,gq-1} chnh l tt c cc phn t ca Fq*. Ta ni g l phn t sinh ca nhm
Fq*.
nh l sau y l mt tng t ca nh l 3.26 trong chng 3.
nh l 5.3. Mi trng hu hn u c phn t sinh. Nu g l mt phn t sinh ca
Fq* th gs l phn t sinh ca Fq* khi v ch khi (s,q-1)=1. Nh vy, tn ti tt c
(q-1) phn t sinh ca Fq*.
By gi ta s m t c th cch xy dng trng Fq t trng Fp.

V
V
n
n
M
M
a
a
tth
h
..C
C
o
o
m
m

d hnh dung, trc tin ta xt vic xy dng trng s phc C nh l mt trng


nng ca s thc R bi a thc P(x)=x2+1. Nh ta bit, c th xem mi s phc
nh mt cp s thc (a,b), v do , c th ng nht mi s phc vi mt a thc
ax+b h s thc. Vi cch tng ng nh vy, khi nhn hai s phc (biu din bi
hai a thc), ta ch vic nhn theo quy tc nhn cc a thc, v thay x2 bi (-1). Ni
cch khc, tp hp cc s phc chnh l tp hp cc a thc vi h s thc, trong
hai a thc c ng nht khi v ch khi hiu ca chng bng a thc P(x)=x2+1.
Ta vit C=R[x]/P(x).
Trng Fq, q=pr, c xy dng t trng Fp theo cch hon ton tng t. Ta xut
pht t mt a thc bt kh quy P(x) bc r vi h s trong Fp, trong h s ca xr
bng 1. Khi ta c:
Fq=Fp[x]/P(x).

Nh vy, cc phn t ca Fq l cc a thc vi h s trong Fp, bc b hn r (v gi s


P(x)=xr+ar-1xr-1+...+a0, khi xr s c thay bi -( ar-1xr-1+...+a0)).
Ta c th xut pht t a thc bt kh quy tu . Cc trng nhn c c s phn t
nh nhau , v ng cu vi nhau.
Ta minh ho nhng iu ni trn qua v d c th.

V d. Xy dng trng F16 t trng F2 bi a thc

P(x)=x4+x3+x2+x+1.

a thc ang xt l mt a thc bt kh quy trn trng F2. Tht vy, nu n c c


khc hng s th phi c c l a thc bc 1 hoc bc 2. Nu c l a thc bc 1,
P(x) c nghim trong F2: iu ny khng xy ra v P(0)=P(1)=1. C bn a thc bc
2 trn F2 l cc a thc x2, x2+1, x2+x, x2+x+1. Th trc tip cho thy rng
khng c cp a thc no c tch bng P(x).
Cc phn t ca F16 l cc a thc bc b hn hoc bng 3, vi h s 0 hoc 1:
-Bc 0: 0,1.
-Bc 1: x, x+1.
-Bc 2: x2, x2+1, x2+x, x2+x+1.
-Bc 3: x3, x3+1, x3+x, x3+x2, x3+x+1, x3+x2+x+1, x3+x2+x, x3+x2+x+1.

87

Quy tc cng v nhn l quy tc cng v nhn thng thng ca cc a thc, vi ch


1+1=0 v x4=-(x3+x2+x+1).
Trong nhiu ng dng, chng hn trong l thuyt thng tin, ngi ta thng vit cc
phn t ca trng Fq theo cc h s ca chng. Nh trong v d trn y, cc phn
t ca trng s l: 0000, 0001, 0010, 0011, 0100, 0101, 0110, 0111, 1000, 1001,
1010, 1100, 1001, 1101, 1110, 1111, cng vi mt bng cho quy tc cng v
nhn ca chng. Ch rng, cc quy tc ny khc vi cc quy tc s hc ng d
modulo q.
Khi bit mt phn t sinh ca trng Fq, ta c th tm cc phn t khc bng cch
nng ln lu tha. Sau y, ta s tm hiu mt thut ton thi gian a thc lm
vic . Thut ton ny s c p dng trong nhng chng sau.

V
V
n
n
M
M
a
a
tth
h
..C
C
o
o
m
m

Trc khi i vo m t thut ton, d hnh dung, ta xt v d sau y. Gi s ta


cn tnh (1994)23(mod 4611). Nu dng cch thng thng (tnh ln lt cc lu tha
ca 1994), ta phi lm 22 php nhn v 22 php chia. gim bt s php tnh phi
lm, ta dng phng php bnh phng lin tip nh sau. Ta c:
(1994)23=(1994)16+4+2+1

Nh vy, ta ch cn tnh modulo ca cc lu tha 1,2,4,8,16 ca 1994. Ni cch


khc, ta ch cn lm php bnh phng lin tip 4 ln, sau nhn cc kt qu
nhng lu tha no tng ng vi s 1 trong biu din s 23 di dng c s 2. Ta
c 23=(10111)2, nn ta nhn kt qu ca nhng lu tha 16,4,2,1.
Cch lm nh trn p dng c cho mi nhm nhn. Gi s g G l phn t ca
nhm nhn G no , ta cn tnh gn, vi n l s t nhin. Ta vit n di dng c s 2:
n= i2i, trong = 1. Khi ta tnh
gn= ( g i ) .
i =1

Thut ton.

S1. (Xut pht) t y 1. Nu n=0, thut ton kt thc.


Nu ngc li, t N n, z g.
S2. (Nhn). Nu N l, t y z.y.

S3. (Chia i N). t N [N/2]. Nu N=0, in ra y v kt


thc thut ton. Ngc li, t z z.z v quay v S2.
C th chng minh tnh ng n ca thut ton vi nhn xt rng, t bc th hai
tr i, ta lun lun c gn=y.zN.
Ta s nh gi phc tp ca thut ton ni trn.
S php nhn phi lm bng s ch s ca n, cng thm s ch s 1 trong cch vit
nh phn ca n, v tr i 1. Nh vy, s php nhn khng vt qu 2[log n]+1, tc l
O(log n). Nu ta tnh trong lp ng d modulo m no , mi php nhn i hi
O(log2 m) php tnh bit, v ton b s php tnh bit cn thit s l O(log nlog2 m).

88

Nh ta thy trn, thc hin cc php tnh trn trng Fq, ta phi lm cc php
tnh i vi cc a thc. Sau y l vi thut ton thc hin cc php tnh .

Thut ton chia


Xt cc a thc vi h s trong trng K tu . Vi mi a thc P, k hiu qua l(P)
h s ca lu tha cao nht. Ta c thut ton , vi a thc cho A, B, B 0, tm
cc a thc Q, R sao cho A=BQ+R, v deg R<deg B:
C1. (Xut pht). t R A, Q 0.
C2. (Kt thc?). Nu deg R<deg B, kt thc thut ton.

l ( R ) deg R-deg B
x
. Sau , t Q Q+S,
l ( B)
R R-S.B, v chuyn sang bc C2.

V
V
n
n
M
M
a
a
tth
h
..C
C
o
o
m
m

C3.(Tm h s). t S

Ta cn lu ngay mt iu. V mt l thuyt, sau bc S3, bc ca R phi gim (v


h s ca xdeg R s bng 0 do nh ngha ca S). Tuy nhin, khi lm vic trn my,
thc t l ta ch c cc s gn ng, nn c th xy ra trng hp h s ca xdeg R tuy
rt nh, nhng khc khng, ngha l bc khng gim, v do khng bo m l
thut ton kt thc! V th, khi vit chng trnh, nht thit phi h s bng 0
sau php tnh R R-S.B.
tm c chung ln nht ca cc a thc, ta c thut ton Euclid sau y.

Thut ton

Cho cc a thc A,B, tm CLN ca A,B.

EP1. (Kt thc?) Nu B=0, in ra A v kt thc thut


ton.
EP2. (Bc Euclid). Gi s A=BQ+R, vi deg R<deg B (tnh
bng thut ton C trnh by trn). t A B, B R, v
quay v bc EP1.
nh l 5.4. C th nhn hoc chia hai phn t ca trng Fq vi O(log3 q) php tnh
bit. Nu k l s nguyn dng th mt phn t ca Fq c th nng ln lu tha k vi
O(log klog3 q) php tnh bit.
Chng minh. Gi s Fq c xy dng bng cch nng trng Fp bi a thc bt kh
quy P(x) bc r. Khi , cc phn t ca trng Fq chnh l cc a thc vi h s
trong trng Fp modulo a thc P(x). nhn hai phn t ca trng Fq, ta phi
nhn hai a thc nh vy. lm vic , ta phi thc hin O(r2) php nhn modulo
p (v cc a thc c bc nh hn r), cng vi mt s php tnh cng. Cc php ny
i hi thi gian t hn. Sau khi c kt qu ca php nhn, ta li phi tnh modulo
a thc P(x), ngha l lm php chia cho a thc P(x) bit c phn d. Php
chia a thc ny i hi O(r) php chia cc s nguyn modulo p v O(r2) php nhn
cc s nguyn modulo p Nh ta bit, mi php nhn s nguyn modulo p c th
thc hin bng O(log2 p) php tnh bit , cn mi php chia modulo p c th lm
(chng hn, dng thut ton Euclid) vi O(log3 p) php tnh bit. Nh vy, ton b s

89

php tnh bit c thc hin khi nhn hai phn t ca trng Fq l:
O(r2log2 p+rlog3 p)=O((rlogp)3)=O(log3 q). Khng nh ca nh l c chng
minh i vi php nhn.
Xt php chia cc phn t ca Fq. chng minh rng c th hin php chia sau
O(log3 q) php tnh bit, ta ch cn chng t rng, nghch o ca mt phn t tm
c bi O(log3 q) php tnh bit, ri p dng kt qu chng minh i vi php
nhn.

V
V
n
n
M
M
a
a
tth
h
..C
C
o
o
m
m

Gi s ta cn tm nghch o ca phn t Q Fq (l mt a thc bc nh hn r, h s


trong Fp). Dng thut chia Euclid cho cc a thc trn trng Fq, ta cn biu din 1
nh l t hp tuyn tnh ca a thc P(x) v Q(x). iu ny lm c bi O(r) php
chia cc a thc bc nh hn r. Mi php chia nh vy cn
O(r2log2 p+rlog3 p)=O(r2log3 p) php tnh bit. Nh vy, ta cn tt c l
O(r3log3 p)=O(log3 q) php tnh bit, iu phi chng minh.
Cn phi xt php tnh nng ln lu tha bc k. Ta c th dng phng php bnh
phng lin tip, v nh vy, s php nhn v bnh phng cn thc hin l O(log k).
S php tnh bit cn thit trong trng hp ny l O(log klog3 q). nh l c chng
minh.

4. S tng t gia s nguyn v a thc.

S pht trin ca s hc, c bit l trong nhng thp k gn y, chu nh hng rt


ln ca s tng t gia s nguyn v a thc. Ni cch khc, khi c gi thuyt no
cha chng minh c i vi cc s nguyn, ngi ta c gng chng minh s
kin tng t cho cc a thc. iu thng d lm hn, c l nguyn nhn ch
yu l v, i vi cc a thc, ta c php tnh o hm, trong khi mt khi nim
tng t cha c i vi cc s nguyn.
Trong tit ny, chng ti c gng thng qua mt vi v d n gin, minh ha vai tr
quan trng ca s tng t ni trn trong cc nghin cu v s hc.
Trc ht, chng ta thy r, gia tp hp cc s nguyn v tp hp cc a thc c
nhng tnh cht rt ging nhau sau y:
1) Cc qui tc cng, tr, nhn, chia hon ton nh nhau cho c hai tp hp.
2) Nu i vi cc s nguyn, ta c cc s nguyn t, th vi cc a thc, ta c cc
a thc bt kh quy.
3) i vi hai s nguyn, cng nh i vi hai a thc, c th nh ngha c chung
ln nht. Hn na, trong c hai trng hp, c chung ln nht ny tm c bng
thut ton Euclid.
4) Mi s nguyn c phn tch thnh cc tha s nguyn t, mi a thc c phn tch
thnh cc a thc bt kh quy.
5) Cc s hu t tng ng vi cc hm hu t.

90

Chng ti dnh cho c gi vic ko di bng danh sch ny. y, chng ti s i


vo mt vi tng t kh nhn thy hn.
Ta n s tng t gia phn tch ra tha s nguyn t v phn tch bt kh quy.
Nu gi thit rng trng k l ng i s, th mi a thc Q(x) k[x] c th phn
tch c di dng sau:
a

Q(x)= P1 1 P2 2 ... Pn n ,
trong Pi(x)=(x- i), i k.

V
V
n
n
M
M
a
a
tth
h
..C
C
o
o
m
m

Nh vy, c th thy rng, trong s tng t gia phn tch bt kh quy v phn tch
ra tha s nguyn t cc nghim ca a thc tng ng vi cc c nguyn t ca s
nguyn. Do , s cc nghim phn bit ca mt a thc c vai tr tng t nh s
cc c nguyn t ca mt s nguyn. T nhn xt , ta i n nh ngha sau y.
nh ngha 5.5. Cho a l mt s nguyn. Ta nh ngha cn ca a, k hiu qua
N0(a), l tch cc c nguyn t ca a:
N0(a)= p .
p |a

Ta s thy rng, s tng t trn y cng vi cc tnh cht ca a thc gi m mt


con ng nhiu hy vng i n chng minh nh l Fermat.
Nm 1983, R. C. Mason chng minh nh l rt p sau y v cc a thc.

nh l 5.6. Gi s a(t), b(t), c(t) l cc a thc vi h s phc, nguyn t cng nhau


tng cp v tho mn h thc
a(t)+ b(t)= c(t)

Khi , nu k hiu qua n0(f) s nghim phn bit ca mt a thc f, th ta c


max{ deg a, deg b, deg c} n0(abc)-1.

Trc khi i vo chng minh nh l, ta chng minh h qu sau y ca nh l


Mason.
H qu 5.7. Khng tn ti cc a thc a, b, c, khc hng s, nguyn t cng nhau,
tho mn phng trnh
an + bn = cn

vi n 3.

Chng minh. Ga s cc a thc a, b, c tho mn phng trnh ni trn. R rng s


nghim phn bit ca a thc anbncn khng vt qu deg a + deg b + deg c. p
dng nh l Mason ta c
n deg a deg a + deg b + deg c - 1
Vit ng thc trn vi b, c, ri cng tng v ba bt ng thc ta c :
n(deg a + deg b + deg c) 3 (deg a + deg b + deg c) - 3

91

Ta c mu thun nu n 3.
Nh vy, nh l Mason cho ta mt chng minh n gin ca nh l Fermat cho cc
a thc. Sau y, ta chng minh nh l Mason.
Chng minh nh l Mason. t f = a/b, g= a/c, ta c: f+g=1. Ly o hm hai v
ca phng trnh ny, ta c: f+g=0. Nhm mc ch xt s cc nghim ca a
thc, ta xt cc thng ca o hm v hm s. Ta c:
(f/f)f+(g/g)g=0,

b
f ' /f
=
.
a
g ' /g

Mt khc, gi s R(t) l mt hm hu t c phn tch sau y

V
V
n
n
M
M
a
a
tth
h
..C
C
o
o
m
m

R(t)= (t i ) qi , qi Z .

Tnh ton n gin cho ta:

R/R=

qi

By gi gi s a, b, c tng ng c cc nghim phn bit l i , j , k . Ta c:


a(t)= (t i ) mi , b(t ) = (t j ) j , c(t ) = (t k ) rk .
n

Nh vy,

mi

rk
t k
b
f ' /f
i
=
=
nj
a
g ' /g
r
t t k
j
k

Mu s chung ca cc phn s trong phn t s v mu s ca thng sau cng l


N0= (t i ) (t j ) (t k ).

l mt a thc c bc l n0(abc). Nh vy, N0f/f v N0g/g l cc a thc c bc


khng qu n0(abc)-1. Mt khc ta c:
N f ' /f
b
= 0
a
N 0 g ' /g

V a, b nguyn t cng nhau nn t ng thc ny suy ra bc ca a v bc ca b


u khng vt qu n0(abc)-1. iu tng t cng ng i vi c do vai tr i xng
ca a, b, c trong phng trnh xut pht. nh l c chng minh.
T nh l Mason, ta c th suy ra nhiu h thc gia cc a thc. Chng hn, mt
trong nhng h qu l nh l sau y:
nh l Davenport. Gi s f(t), g(t) l cc a thc, sao cho f 3 g2. Khi ta c
deg(f 3-g2) 1/2 deg f+1
92

Chng ti dnh chng minh nh l ny cho c gi. Khng nh tng t i vi cc


s nguyn vn cn cha c chng minh. Ta c:
Gi thuyt Hall. Gi s x, y l cc s nguyn dng sao cho x3 y2. Khi , vi mi
>0, tn ti C>0 ch ph thuc sao cho
| y2-x3|>Cx1/2-
C th ni thm rng, bt ng thc trong nh l Davenport l tt nht c th: i
vi cc a thc f(t)=t6+4t4+10t2+6, g(t)=t9+6t7+21t5+35t3+63/2t ta c:
deg(f3-g2)=1/2degf+1. Nm 1982, L. V. Danilov cng chng minh rng, s m
1/2 trong gi thuyt Hall l tt nht c th.

V
V
n
n
M
M
a
a
tth
h
..C
C
o
o
m
m

nh l Mason v tng t gia cc s nguyn v a thc gi cho gi thuyt sau


y:
Gi thuyt abc (Oesterl, 1986). Gi s a, b, c l cc s nguyn, nguyn t cng
nhau v tho mn h thc a+b=c. Khi , vi mi >0, tn ti s C sao cho
max(|a|, |b|, |c|)<CN1+ ,

trong N= p l cn ca abc.
p |abc

Hon ton tng t nh trn, t gi thuyt abc c th suy ra nh l Fermat tim


cn: vi n ln, phng trnh Fermat khng c nghim nguyn.
Nhn xt. Gi s p l mt c nguyn t no ca mt trong cc s a, b, c, chng
hn p|a. Khi , nu p ln th trong phn tch ca a ra tha s nguyn t, p phi c
s m tng i nh ( |a| khng vt qu xa cn ca abc theo gi thuyt). iu
ny cng gii thch ti sao phng trnh Fermat khng c nghim vi bc ln: khi
, mi c nguyn t ca an, bn,cn s tham gia vi bc qu ln.
Trn y l mt v d v s tng t gia cc gi thuyt i vi cc s v cc a
thc. Khi nghin cu mt vn no t ra i vi cc s, ngi ta thng nghin
cu ng thi tng t ca n trn trng hm. Phn ln cc gi thuyt ca s hc
c chng minh trc ht trn trng hm. Nh ta thy trong chng minh nh l
Mason, iu quan trng y l c php tnh o hm.
Gn y, Manin (1992) t ra vn : nu nh cc s nguyn tng ng vi cc a
thc mt bin, th cc a thc nhiu bin tng ng vi i tng no? Cu hi
dn n vic xy dng nhng tch mi ca cc lc Spec Z. y l mt
hng nghin cu ang pht trin mnh, v ni dung ca n vt ngoi khun kh
ca cun sch ny.

93

Bi tp v tnh ton thc hnh chng 5


I. Bi tp
5.1. Gi s K l trng c trng p. Chng minh rng, i vi cc phn t ca trng
K, ta c:
(a+b)p=ap+bp.
5.2. Xy dng trng F9 t trng F3 bi cc a thc sau:

V
V
n
n
M
M
a
a
tth
h
..C
C
o
o
m
m

1) x2+1

2)x2-x-1.

5.3. Dng thut ton EP tm CLN ca cc a thc P,Q trong trng Fp, v biu
din dng d=uP+vQ:
1) P=x3+x+1, Q=x2+x+1, p=2.

2) P=x6+x5+x4+x3+x2+x+1, Q=x4+x2+x+1, p=2.


3) P=x3-x+1, Q=x2+1, p=3.

4) x5+x4+x3-x2-x+1, Q=x3+x2+x+1, p=3.

5) x5+88x4+73x3+83x2+51x+67, Q=x3+97x2+40x+38, p=101.

5.4. Vi mi d 6, tm tt c cc a thc bt kh quy bc d trn trng F2.


5.5. Nhng a thc no trong Fp[x] c o hm ng nht bng 0?

5.6. Chng minh rng t gi thuyt abc suy ra nh l Fermat tim cn.
5.7. Chng minh nh l Davenport.

5.8. Cho f, g l cc a thc vi h s nguyn, sao cho f 3-g4 khng ng nht bng 0.
Chng minh rng
deg(f3-g4) 5/3deg g+1.

Pht biu kt lun tng t cho cc s nguyn.

5.9. Da vo nh l Mason, tm nhng h thc mi lin quan n bc ca cc a


thc h s nguyn (tng t bi tp trn y). Th pht biu v chng minh kt lun
tng t di vi cc s nguyn.
5.10. Th a ra mt nh ngha v o hm ca mt s nguyn.

II. Thc hnh trn my tnh


II. 1. Thc hnh tnh s a thc bt kh quy bc d trn trng hu
hn
94

tnh s a thc bt kh quy bc n trn trng hu hn c c s p ta thc hin


dng lnh nh sau;
[>mipolys(n, p);
Sau du (;) n phm Enter trn mn hnh s xut hin s a thc cn tm.
Th d: Tnh s cc a thc bt kh quy bc 6 trn trng c c s 2.
Ta thc hin lnh sau:
[>mipolys(6, 2);
9

V
V
n
n
M
M
a
a
tth
h
..C
C
o
o
m
m

Nh vy c 9 a thc bt kh quy trn trng F2.

II. 2. Thc hnh tm c chung ln nht ca cc a thc trn trng


hu hn
Cho P, Q l cc a thc bin x vi h s trn trng hu hn c c s p. tm c
chung ln nht D ca P v Q v biu din di dng D=sP+tQ ta thc hin dng
lnh sau:
[> Gcdex(P,Q,x,s,t) mod p;

Sau du (;) n phm Enter trn mn hnh s xut hin mt a thc, chnh l c
chung ln nht ca P, Q. Tip tc thc hin lnh:
[>s,t;

Sau du (;) n phm Enter trn mn hnh s xut hin hai a thc s, t cn tm.
Ch lnh Gcdex ch G l ch vit hoa.

Th d1: Tm c chung ln nht D ca cc a thc P, Q trn trng F2 v biu din


di dng D=sP+tQ, trong P=x3+x+1, Q=x2+x+1.
Ta thc hin dng lnh:

[> Gcdex(x^3+x+1,x^2+x+1,x,s,t) mod 2;


1

[>s,t;

1+x,x2

Vy 1=(1+x)P+x2Q.
Th d2: Tm c chung ln nht D ca cc a thc P, Q trn trng F101 v biu
din di dng D = sP + tQ, trong x5 + 88x4 + 73x3 + 83x2 + 51x + 67,
Q=x3+97x2+40x+38
Ta thc hin dng lnh:
[>Gcdex(x^5+88*x^4+73*x^3+83*x^2+51*x+67,x^3+97*x^2+40*x
+38,x,'s','t') mod 101;

95

x + 78
[> s,t;
50x + 20, 51x3 + 26x2 + 27x + 4
Vy x+78=(50x+20)P+(51x3+26x2+27x+4)Q.

II. 3. Thc hnh tm c chung ln nht, bi chung nh nht ca cc


a thc trn trng hu t
Cho P, Q l cc a thc bin x vi h s trn trng hu t.
1. tm c chung ln nht D ca P v Q ta thc hin dng lnh sau:

V
V
n
n
M
M
a
a
tth
h
..C
C
o
o
m
m

[> gcd(P,Q);
Sau du (;) n phm Enter trn mn hnh s xut hin kt qu, chnh l c
chung ln nht ca P, Q.
Th d1: Tm c chung ln nht D ca cc a thc P, Q trn trng hu t trong
P=x2-y2, Q=x3-y3.
Ta thc hin dng lnh:

[> gcd(x^2-y^2,x^3-y^3);

-y+x

Vy c chug ln nht ca P v Q l -y+x

2. tm bi chung nh nht ca P, Q ta thc hin dng lnh nh sau:


[> lcm(P,Q);

Sau du (;) n phm Enter trn mn hnh s xut hin kt qu, chnh l bi
chung nh nht ca P,Q.
Th d 2: Tm bi chung nh nht ca cc a thc P, Q trn trng hu t trong
P=x2-y2, Q=x3-y3.
Ta thc hin dng lnh:

[> lcm(x^2-y^2,x^3-y^3);

-yx3+y4-x4+xy3

Vy bi chung nh nht ca P v Q l -yx3+y4-x4+xy3

96

Chng 6

Vi ng dng vo l thuyt mt m

V
V
n
n
M
M
a
a
tth
h
..C
C
o
o
m
m

Cho n khong cui nhng nm 70, s hc vn c xem l mt trong nhng ngnh


l thuyt thun tu nht ca ton hc, v hu nh khng c ng dng thc t. Quan
nim thay i hn sau khi s hc c p dng xy dng nhng h mt m
kho cng khai. Cc l thuyt mi ca s hc, c bit l s hc thut ton, tm thy
nhng ng dng trc tip vo thc tin. V th chng ti dnh mt chng trnh by
nhng im c bn ca l thuyt mt m, qua , c gi c th thy c vai tr
quan trng ca nhng vn xt n trong l thuyt s thut ton, nh vn
phc tp ca cc thut ton phn tch mt s nguyn dng ra tha s, hay vn
kim tra nguyn t.

1. M Ceasar.

C th ni, mt m c t thi c i. Ngi ta cho rng, ngi u tin p dng


mt m mt cch c h thng m bo b mt thng tin qun s l nh qun s
thin ti ca La M c i, Julius Ceasar. S pht trin ca x hi dn n vic ngy
nay mt m khng nhng ch c dng trong b mt qun s v ngoi giao, m cn
dng, v c th ch yu l dng trong b mt kinh t, thng mi. V th xut hin
nhng i hi mi i vi cc h mt m hin i, khc v nguyn tc so vi mt
m thng dng trc y. Khc vi hot ng qun s hoc ngoi giao, trong hot
ng kinh doanh, s lng n v phi cng trao i thng tin thng l rt ln.
Thm ch, nhng ngi cha h quen bit nhau cng c nhu cu trao i nhng
thng tin mt vi nhau. Bi th, nhng h thng mt m xy dng theo nguyn tc
c kh c th thch hp: trong cc h m , khi bit kho lp m, ta d dng tm
ra kho gii m. Hin nhin, mun gi mt thng bo mt cho mt i tng no ,
ta cn phi bit kho lp m ca h, v th, nhng ngi cng dng mt h m u
bit ht b mt ca nhau. Khi mt b mt c qu nhiu ngi bit th khng cn l b
mt na. Cc h thng mt m hin i, mt m kho cng khai, khc phc c
nhng nhc im : mi ngi tham gia trong h thng ch cn gi b mt kho
gii m ca mnh, trong khi kho lp m c thng bo cng khai. Vic bit kho
lp m khng cho php tm ra kho gi m trong mt thi gian chp nhn c, ngay
c khi s dng nhng my tnh hin i nht. Nhng mt m kho cng khai tm
thy u tin l nhng mt m dng hm s hc.
C mt iu ht sc th v l, ni cho cng, nhng h mt m hin i cng ch l s
ci tin mt m ca Ceasar! V th chng ti bt u vic trnh by mt m Ceasar.
Trc ht chng ta cn thng nht mt s danh t.

97

Vn bn tc l thng bo cn chuyn, c vit bng ngn ng thng thng. y,


ta s xem cc vn bn u c vit bng ting Vit.
Vic chuyn thng bo thnh dng mt m c gi l m ha.
Bn m ho ca vn bn c gi l vn bn mt.
Gii m tc l chuyn mt vn bn mt thnh vn bn ban u.
Ceasar chuyn thng bo mt bng cch sau y. Trc tin, lp tng ng mi ch
ci vi mt s. Nh bng tng ng , ta c th chuyn mt vn bn thnh dng
ch s. Sau ta cng thm 3 vo mi ch s nhn c. Li nh bng tng ng
gia ch v s, ta bin bng ch s mi ny v dng ch vit. Nh vy ta nhn c
mt vn bn mt cn chuyn i. y l qu trnh m ho.

V
V
n
n
M
M
a
a
tth
h
..C
C
o
o
m
m

Khi nhn c vn bn mt, ta gii m bng cch bin n thnh dng ch s nh


bng tng ng gia ch v s, sau tr i 3 mi ch s v li chuyn n v dng
ch li c vn bn ban u.
Ch rng khi php cng hoc tr i 3 a ta vt khi gii hn ca bng tng
ng, ta thay s bng thng d dng b nht modulo s cc phn t ca bng
tng ng gia ch v s.
Sau y ta s xt trn mt v d c th.

Trc ht ta lp tng ng cc ch ci vi cc s theo bng sau:


a

10

11

12

13

14

15

16

17

18

19

20

21

23

24

25

26

27

28

29

22

Bng 1

D nhin ta c th thm cc s ch du, nhng n gin , y ta tm thi vit


cc vn bn khng du.
Nh vy m Ceasar c thnh lp theo cng thc sau:
C P+3(mod 29)

(6.1)

trong P l ch s trong vn bn, cn C l ch s tng ng trong vn bn mt.


Chng hn ta mun m ho thng bo sau y:

98

LY THUYT MT MA KHNG CO GI KHO


Trc ht nhm nng cao tnh bo mt, ta tch thng bo thnh tng nhm 5 ch ci,
trnh vic mt s t ca thng bo d b pht hin cn c vo s ch ci. Nh vy
thng bo cn m ho l:
LYTHU YTM TMAKH NGCO GIKHO
Nh bng 1, ta chuyn thng bo thnh dng ch s:
14 29 24 11 25 29 8 24 15 1 24 15 1 13 11 18 16 10 5 18 10 12 13 11 18
Ap dung cng thc (6.1), bng ch s trn c chuyn thnh:
17 3 27 14 28 3 11 27 18 3 27 18 4 16 14 21 19 13 8 21 13 15 16 14 21

V
V
n
n
M
M
a
a
tth
h
..C
C
o
o
m
m

c vn bn mt, ta ch cn chuyn li thnh dng ch ci theo Bng 1:


OVLU HV VBNL QKEQ KMNLQ

S 3 trong cng thc (6.1) c gi l kho ca m Ceasar, v n c dng m


ho cng nh gii m.
Ta cng c th lp mt h mt m mi bng cch thay s 3 trong cng thc (6.1)
bng mt s k tu khc gia 1 v 29:
C P+k (mod 29)

(6.2)

Trong trng hp ny, kho ca m l k. Vic m ho v gii m c tin hnh


hon ton tng t nh trn.
Ta c th lp m tng qut hn cht t bng cch thay cng thc (6.2) bi cng thc
sau y:
C aP+b(mod 29),

trong a, b l cc s nguyn, v (a, 29)=1. Nhng m nh vy c gi l m bin


i aphin. Vic gii m c tin hnh bng cch gii phng trnh ng d (6.2),
khi bit c, a, b.
Phn tch sau y cho thy tnh bo mt ca m Ceasar l khng cao. Khi bt c
mt vn bn mt, ngi ta c th da vo tn sut xut hin ca cc ch ci on
ra kho ca m. Chng hn nu ch a ni chung xut hin nhiu nht trong cc vn
bn th ch ci no c mt nhiu nht trong vn bn mt c nhiu kh nng l ch a,
t on ra kho. Hn na, ch c 29 cch khc nhau chn kho cho loi m ni
trn, nn dng tm ra kho ca m, nht l khi p dng my tnh. i vi m bin
i aphin, ch cn da vo tn sut xut hin t tm ra hai ch ci tng ng vi 2
ch no trong vn bn mt, ta c th tm ra a, b bng cch gii h hai phng
trnh ng d. Ngoi ra, vic gii nhng h m bin i aphin cng qu d dng i
vi my tnh.
Nh vy, vi nhng yu cu v bo mt cao hn, ngi ta phi dng nhng h mt
m phc tp hn. Sau y l mt vi h m thng dng, t n gin n phc tp.

99

2. M khi.
M khi xut hin nhm chng li vic s dng tn sut xut hin ca cc ch ci
trong vn bn d ra kho gii m. Khc vi cc h m trnh by mc trn, ta
khng m ho tng ch ci ca vn bn, m m ho tng khi ch ci. Trc tin ta
xt trng hp m khi 2 ch. d hiu ta xt v d sau y.
Gi s thng bo cn m ho l
KHNG CO IU BI MT NAO GI C LU
Trc ht ta tch thng bo trn thnh khi hai ch:
KH N GC O U BI M TN AO GI CL U

V
V
n
n
M
M
a
a
tth
h
..C
C
o
o
m
m

Sau cc ch ci c chuyn thnh cc ch s tng ng:


13 11 18 15 10 5 17 7
3 25

9 25

4 12 16 3

24 15

1 17

1012

26 19 5 14

Vi mi m khi hai ch, ta chn mt ma trn cp hai lm kho ca m. Chng hn


ma trn

23 11
A=

9 12

Khi cc khi hai ch s P1P2 trong vn bn c chuyn thnh cc khi hai ch


s C1C2 trong vn bn mt theo cng thc sau y:
C1 23P1+11P2(mod 29)

C2 9P1+12P2(mod 29)

(6.3)

Nh vy thng bo trn y c chuyn thnh:

14 17 28 23 24 5 4 5 18 4 21 6 21 19 7 10 14 2 24 27 8 10 25 8
Tr li cc ch ci tng ng, ta c vn bn mt:

LO XS TC BC B QD TD Q G LI TV EG UE

gii m, ta cn gii h phng trnh ng d (6.3) tm P1,P2. iu thc hin


c nh nh l sau y:
nh l 6.1. Cho h phng trnh ng d

ax+by r(mod m)
cx+dy s(mod m)
t =ad-bc (mod m). Khi , nu ( ,m)=1 th h phng trnh ang xt tn ti
nghim duy nht modulo m, cho bi cng thc sau:
x -1(dr-bs) (mod m),
y -1(as-cr) (mod m),

100

trong -1 l nghch o ca modulo m.


nh l trn y c chng minh hon ton tng t nh trong i s tuyn tnh
(ch cn thay i kin ( ,m)=1 bi iu kin 0).
Trong v d trn y, 3(mod 29), -1 10(mod 29). Nh vy, khi c khi C1C2
trong vn bn mt v bit m kho l ma trn A, ta tm c khi ch tng ng
trong vn bn l P1P2 theo cng thc sau:
P1=10(12C1-11C2) 4C1+6C2(mod 29)
P2=10(23C2-9C1) 26C1+27C2(mod 29).
Tm li, vic m ho v gii m c tin hnh nh cc cng thc:

V
V
n
n
M
M
a
a
tth
h
..C
C
o
o
m
m

C1 23 11 P1 P1 4 6 C1
=
; =

C2 9 12 P2 P2 26 27 C2

Nh vy, vic s dng m khi nng cao rt nhiu tnh bo mt. Tuy vy, kho
ca m vn c th b khm ph nh vic nghin cu tn sut xut hin ca cc khi
ch ci. Chng hn nu ta dng m khi hai ch, th c c thy 292=641 khi trong
ting Vit, v nh vy vn cn kh nng khm ph ra kho ca m nh cc my tnh
hin i. Trong trng hp ta s dng m khi vi nhng khi nhiu ch ci, vic
tm ra kho bng tn sut cc khi ch trn thc t l khng s dng c: Chng
hn khi dng m khi 10 ch ci, s khi ch s l 2910, vt qu kh nng thm d
tn sut xut hin cc khi ch trong ngn ng.
Cc m khi n ch ci c lp v gii hon ton tng t nh trn, trong cc ma
trn C,P l cc ma trn n ct, A l ma trn vung cp n. Ma trn nghch o ca A
tn ti khi nh thc ca A nguyn t cng nhau vi 29, v ma trn P s c tnh
bng quy tc Kramer nh trong i s tuyn tnh (ch cn thay du = bi (mod
29)).
Sau y ta trnh by mt loi h m mi, mt mt n c tnh bo mt rt cao, Mt
khc l c s cho nhng h m hon ton mi: cc h m kho cng khai.

3. M m.

H m ny c Pohlig v Hellman a ra nm 1978.

Gi s p l mt s nguyn t l, v gi s kho lp m e l mt s nguyn dng sao


cho (e,p-1)=1. Cng nh trc y, m ho mt thng bo, trc tin ta chuyn
cc ch ci thnh dng cc ch s tng ng (thm s 0 vo trc nhng s c mt
ch s). Ta dng bng sau y:
a

01

02

03

04

05

06

07

08

09

10

11

12

13

14

15

16

17

18

19

20

21

101

22

23

24

25

26

27

28

29

Sau ta nhm cc s nhn c thnh tng nhm 2m ch s theo nguyn tc sau:


2m l s nguyn chn ln nht sao cho mi s tng ng vi m ch ci (xt nh l
mt s nguyn c 2m ch s) u nh hn p. d hiu, ta gi s p l s nguyn t
trong khong 2929<p<292929. Mi ch ci c biu din bng mt s khng qu
29. Mt s c m ch ci s c biu din bng mt s khng vt qu m ln s 29
vit lin tip. Nh vy, m bo s lun lun nh hn p, m ch c th l 1 hoc
2. Ta ly m=2.

V
V
n
n
M
M
a
a
tth
h
..C
C
o
o
m
m

i vi mt khi P trong vn bn (l mt s 2m ch s), ta lp khi C tng ng


trong vn bn mt theo cng thc sau:
C Pe (mod p), 0 P<p

Vn bn mt s cha nhng khi ch s l cc s nguyn nh hn p.

V d. Gi s s nguyn t s dng tin hnh lp m l p=2939 v kho lp m


e=31, nh vy (e,p-1)=(31,2938)=1.
Ta cn m ho thng bo sau:

I HA NI NGAY

Trong trng hp ny, m=2, v ta nhm vn bn nhn c khi chuyn sang ch s


thnh nhm bn ch s:
0712 1101 1618 1216 1001 2928

Ch rng, khi cui cng bn ch s, ta thm ch X trong vn bn, iu ny


khng gy nhm ln khi c thng bo (d nhin c th thay ch X bng bt c ch
ci na khng gy hiu nhm).
Tip theo, ta chuyn cc khi P trong vn bn thnh cc khi C trong vn bn mt
theo cng thc sau:
C P31 (mod 2939), 0<C<2633

Chng hn, m ho khi u tin, ta tnh:

C 071231 (mod 2939)

tnh c C mt cch nhanh chng, ta dng thut ton bnh phng lin tip
xt trong chng 5.
Trc tin, ta vit 31 di dng c s 2: 31=(11111)2. Tnh ton n gin cho ta:
7212 1436, 7124 1857, 7128 1002, 71216 1805(mod 2939).
T biu din ca 31 di dng c s 2, ta c:
71231 (712.1436.1857.1805) 898(mod 2939).

102

Sau khi m ho ton b vn bn, ta nhn c vn bn mt cn chuyn l:


898 2674 1003 746 1786 2614
gii m mt khi C trong vn bn mt, ta cn bit kho gii m d. l s d tho
mn de 1(mod p-1), c ngha l d l mt nghch o ca e modulo p-1. Nghch o
tn ti do gi thit (e,p-1)=1. tm li c khi C trong vn bn, ta ch vic
nng khi C ln lu tha d modulo p. Tht vy,
Cd (Pe)d Pde Pk(p-1)+1 P(mod p)
trong de=k(p-1)+1 i vi s nguyn k no , bi v de 1(mod p-1).

V
V
n
n
M
M
a
a
tth
h
..C
C
o
o
m
m

V d. gii m mt khi trong vn bn mt c m ho bng cch s dng


modulo p=2938 v kho lp m e=31, ta cn tm s nghch o ca e=31 modulo
p-1=2938. Tht ton Euclid m rng gip ta d dng tm c d. Tht vy, theo cc
k hiu ca thut ton Euclid m rng, ta t: u=2938, v=31. Tnh ton theo thut
ton , ta c kt qu sau y:
u1

u2

u3

v1

v2

v3

2938

31

94

31

-94

24

-94

24

-1

95

-1

95

-379

-379

-9

853

-9

853

31

-2938 0

3
2
3

Nh vy, ta c: 31.853-9.2938=1, v d=853


gii m khi C ta dng cng thc

P C853(mod 2938).

phc tp ca thut ton lp m v gii m i vi m m.


Vi mt khi P trong vn bn, ta m ho bng cch tnh Pe(mod p), s cc php tnh
bit cn thit l O((log2 p)3). gii m trc ht ta phi tm nghch o d ca e
modulo p-1. iu ny thc hin c vi O(log3 p) php tnh bit, v ch cn lm mt
ln. Tip theo , tm li c khi P ca vn bn t khi C ca vn bn mt, ta
ch cn tnh thng d nguyn dng b nht ca Cd modulo p: s cc php tnh bit
i hi l O((log2 p)3).
Nh vy, thut ton lp m v gii m c thc hin tng i nhanh bng my
tnh.

103

Tuy nhin ta s chng t rng, vic gii m mt vn bn mt c m ho bng m


ni chung khng th lm c nu nh khng bit kho e. Tht vy, gi s ta bit
s nguyn t p dng lm modun khi lp m, v hn na, gi s bit khi C no
trong vn bn mt tng ng vi khi P trong vn bn, tc l ta bit mt ng d
thc
C Pe(mod p)

V
V
n
n
M
M
a
a
tth
h
..C
C
o
o
m
m

Vn cn li l xc nh e t cng thc trn. S e tho mn iu kin c gi


l lgart c s P ca C modulo p. C nhiu thut ton khc nhau tm lgarit c
s cho modulo mt s nguyn t. Thut ton nhanh nht c bit hin nay i
hi khong exp( log p log log p) php tnh bit. tm logarit modulo mt s
nguyn t c n ch s thp phn, cc thut ton nhanh nht cng i hi s php tnh
bit xp x s php tnh bit cn dng khi phn tch mt s nguyn n ch s thnh tha
s. Nh vy, nu lm vic vi cc my tnh c tc 1 triu php tnh trong mt
giy, khi p c khong 100 ch s thp phn, vic tm logarit modulo p cn khong
74 nm, cn trong trng hp p c khong 200 ch s, thi gian cn thit l 3,8 t
nm!
Cn phi lu rng, c nhng trng hp vic tm ra logarit modulo p c thc
hin bng nhng thut ton nhanh hn rt nhiu. Chng hn khi p-1 ch c nhng
c nguyn t nh, tn ti nhng thut ton c bit cho php tnh logarit modulo p
vi O(log2 p) php tnh bit. R rng nhng s nguyn t nh vy khng th dng
lp m. Trong trng hp , ta c th ly q vi p=2q+1, nu s q cng l s nguyn
t (khi q-1 khng th c cc c nguyn t nh).

M m v h thng c nhiu c th tham gia.

Mt trong nhng u im ca h m m l trong mt h thng c nhiu c th cng


tham gia trao i thng tin, tng cp c th hoc tng nhm nh c th vn c kh
nng s dng kho mt m ang dng to nhng kho mt m chung, b mt i
vi cc c th cn li ca h thng.
Gi s p l mt s nguyn t ln v a l mt s nguyn, nguyn t cng nhau vi p.
Mi c th trong h thng chn mt s k nguyn t cng nhau vi p-1 lm kho cho
mnh. Khi hai c th vi cc kho k1, k2 mun lp mt kho chung trao i thng
tin, c th th nht gi cho c th th hai s nguyn y1 tnh theo cng thc:
y1 a k1 (mod p), 0 < y1 < p

C th th hai s tm ra kho chung k bng cch tnh

k y1

k2

a k1k2 (mod p), 0 < k < p

Tng t c th th hai gi cho c th th nht s nguyn y2

y 2 a k2 (mod p), 0 < y 2 < p


v c th th nht tm ra kho chung k theo cng thc

k y2

k2

a k1k2 (mod p)

104

Ta lu rng, trong cch lp kho chung trn y, cc c th th nht v th hai


khng cn bit kho mt m ca nhau, m ch s dng kho mt ring ca mnh.
Mt khc, cc c th cn li ca mt h thng cng khng th tm ra kho chung k
trong mt thi gian chp nhn c, v lm vic , h phi tnh logarit
modulo p.
Trn y l cch lp kho chung ca hai c th. Hon ton tng t nh vy, tng
nhm cc th c th lp kho chung.

V
V
n
n
M
M
a
a
tth
h
..C
C
o
o
m
m

4. Cc h mt m kho cng khai.


Trong tt c cc h mt m trnh by trn y, cc kho lp m u phi c gi b
mt, v nu kho lp m b l th ngi ta c th tm ra kho gii m trong mt thi
gian tng i ngn. Nh vy nu trong mt h thng c nhiu cp c th hoc nhiu
nhm c th cn trao i thng tin mt vi nhau, s kho mt m chung cn gi b
mt l rt ln, v nh vy, kh c th bo m c. H m m chng ta nghin cu
di y c lp theo mt nguyn tc hon ton mi, trong vic bit kho lp
m khng cho php tm ra kho gii m trong mt thi gian chp nhn c. V th,
mi c th ch cn gi b mt kho gii m ca ring mnh, trong khi kho lp m
c thng bo cng khai. Trong trng hp mt trong cc c th b l kho gii m
ca mnh, b mt ca cc c th cn li khng h b nh hng. L do ca vic c th
xy dng nhng h m nh vy chnh l iu ta ni n khi xt cc h m m:
phc tp ca thut ton tm logarit modulo p l qu ln.
Trc ht, ta ni s qua v nguyn tc ca cc h m kho cng khai. Gi s trong
h thng ang xt c n c th cng trao i cc thng tin mt. Mi cc th chn cho
mnh mt kho lp m k v mt cng thc m ho E(k), c thng bo cng khai.
Nh vy c n kho lp m cng khai k1, k2,...,kn. Khi c th th i mun gi thng bo
cho c th th j, cng nh trc y, mi ch trong thng bo c chuyn thnh s,
nhm thnh tng khi vi di no . Sau , mi khi P trong vn bn c m
ho bng kho lp m E(kj) ca c th th j ( thng bo cng khai), v gi i di
dng C=E(kj)(P). gii m thng bo ny, c th th j ch cn dng kho gii m
(b mt ring cho mnh) Dk j

Dk j (C)= Dk j E k j ( P) = P ,

bi v Dk j v E k j l cc kho gii m v lp m ca cng c th th j. Cc c th


trong h thng, nu nhn c vn bn mt, cng khng th no gii m, v vic bit
kho lp m E k j khng cho php tm ra kho gii m Dk j .
c th ho nguyn tc va trnh by, ta xt v d trn h m kho cng khai c
tm thy u tin nm 1978 bi Rivest, Shamir v Adleman (xem[ RSA]) (thng
c gi l h m RSA).
H RSA c xy dng trn c s m m, trong kho l cp (e,n), gm s m e
v modun n. S n c dng y l tch ca hai s nguyn t rt ln no , n=pq,

105

sao cho (e, (n))=1, trong (n) l hm Euler. m ho mt thng bo, trc
tin ta chuyn cc ch ci thnh cc s tng ng v nhm thnh cc khi vi di
ln nht c th (tu thuc kh nng tnh ton) vi mt s chn ch s. m ho
mt khi P trong vn bn, ta lp khi C trong vn bn mt bng cng thc:
E(P) C Pe(mod n), 0<C<n.
Qu trnh gii m i hi phi bit c mt nghch o d ca e modulo (n).
Nghch o ny tn ti theo iu kin (e, (n) )=1.
Mun gii m mt khi C trong vn bn mt, ta tnh
D(C) Cd (Pe)d Ped P k (n) + 1 ( P ( n ) ) k P P(mod n).

V
V
n
n
M
M
a
a
tth
h
..C
C
o
o
m
m

trong ed=k (n)+1 i vi s nguyn k no , v ed 1(mod (n)), v do nh l


Euler ta c: P ( n ) 1(mod p) , khi (P,n)=1 (ch rng, xc sut P v n khng
nguyn t cng nhau l ht sc nh, xem bi tp 6.7). Cp (d,n) nh vy c gi l
kho gii m.
minh ho, ta xt mt v d n gin, ly n=53.61=3233 v e=17. Trong trng
hp ta c (e, (n))=(17,52.63)=1. Gi s ta cn m ho thng bo sau:
A GI TIN

Trc tin ta chuyn cc ch ci trong vn bn thnh cc s tng ng v nhm


chng thnh tng khi 4 ch s. Ta c:
0701

1026 1224

1209 1628

Ta m ho cc khi nh cng thc

C P17(mod 3233)

Ta li dng phng php bnh phng lin tip. Chng hn, i vi khi u tin, ta
nhn c:
(701)17 140(mod 3233)

M ho ton b vn bn, ta c vn bn mt sau y:


140

721

1814

1819 361

Khi nhn c vn bn mt ny, gii m, ta phi tm mt nghch o d ca e


modulo (3233). Ta c (53.61)=52.60=3120. Dng thut ton Euclid m rng, ta
tnh c d=2753. Nh vy, gii m khi C ta dng cng thc
P C2753(mod 3233), 0 P 3233
C th th li:
C2753 (P17)2753 P(P3120)15 P(mod 3233)
y ta dng nh l Euler nhn c P ( 3253) P3120 1(mod 3233), khi
(P,3233)=1 (iu ny ng vi mi khi trong thng bo ca chng ta)

106

By gi ta ch ra rng, h m RSA tho mn cc nguyn tc ca h m kho cng


khai ni u tit ny. Trc tin, ta ch rng, mi c th phi chn hai s nguyn
t ln p v q, c chng 100 ch s thp phn. iu ny c th l trong t pht nh
mt my tnh. Khi cc s nguyn t p v q c chn, s m dng m ho e s
c ly sao cho (e, (qp))=1. Ni chung nn chn e l s nguyn t tu ln hn
q v p. S e c chn nht thit phi tho mn 2e>n=pq. Nu iu kin ny khng
c tho mn, ta c C=Pe<n, v nh vy tm ra P, ta ch vic tnh cn bc e ca
C. Khi iu kin 2e>n c tho mn, mi khi P khc 0 v 1 u c m ho bng
cch nng ln lu tha v ly ng d theo modulo n.
Ta cn phi chng t rng, vic bit kho lp m (cng khai) (e,n) khng n n
vic tm c kho gii m (d,n).

V
V
n
n
M
M
a
a
tth
h
..C
C
o
o
m
m

Ch rng, tm nghch o d ca e modulo (n), trc tin phi tm c (n).


Vic tm (n) khng d hn so vi phn tch n, bi v, mt khi bit (n) v n, ta s
phn tch c n=pq.
Tht vy, ta c:

p+q=n- (n)+1

p-q= ( p + q ) 2 4qp = ( p + q ) 2 4n

T cc cng thc tm c q v p.

Nu ta chn cc s p v q khong 100 ch s thp phn, th n s c khong 200 ch


s thp phn. phn tch mt s nguyn c ln nh th, vi cc thut ton nhanh
nht hin nay v vi nhng my tnh hin i nht, ta mt khong 3,8 t nm!
C mt vi iu cn lu khi chn cc s p v q trnh ri vo trng hp tch pq
b phn tch nhanh nh nhng thut ton c bit: q v p cn chn sao cho p-1 v q-1
ch c cc tha s nguyn t ln, (p-1,q-1) phi nh, q v p phi c s ch s trong
khai trin thp phn khc nhau khng nhiu.
C th ny ra cu hi: trong mt h thng nhiu c th tham gia, cc kho lp m
li c cng khai, lm sao c th trnh c trng hp mt c th ny mo danh
mt c th khc gi thng bo cho mt c th th ba? Ni cch khc lm sao c
th k tn di cc thng bo mt? Vn ny c gii quyt n gin nh sau:
Gi s ng I cn k tn di thng bo gi ng J. Khi , trc tin, ng I tnh
S D (I) I d i (mod n ).
ki

Ch rng ch c ng I lm c vic ny, v trong cng thc s dng kho gii m


ca ng I. Sau , I s gi cho J thng bo
e

C E k j ( S ) = S j (mod n j ) ,
trong (ej,nj) l kho lp m ca J.
Khi nhn c, gi m, J trc tin dng kho gii m ring ca mnh nhn ra
S:

107

Dk j (C ) Dk j ( E k j ( S )) S
xc minh S ch thc l ch k ca I, J ch cn vic p dng vo S kho lp m
cng khai ca I:

E ki ( S ) E ki Dki ( I ) I
Ch cch l nh trn thch hp khi nj>ni, v khi ta lun c S<nj. Nu ngc li, I
phi tch S thnh tng khi c di b hn nj v m ho tng khi ri mi chuyn.
Nh vy, mt mt J xc nh c ng l thng bo do I gi n, mt khc I
cng khng th t chi vic mnh l ch nhn ca thng bo , v ngoi I ra, khng
ai c kho m Dki mo ch k ca I.

V
V
n
n
M
M
a
a
tth
h
..C
C
o
o
m
m

Trn y l h mt m kho cng khai xut hin u tin. T n nay, c nhiu


h mt m kho cng khai mi ra i. Tuy vy, nguyn tc chung ca cac h m
l s dng nhng thut ton mt chiu, tc l nhng thut ton cho php tm ra
mt i lng no d tng i nhanh, nhng vic tm nghch o (theo mt ngha
no ) ca n i hi thi gian qu ln. c gi no quan tm n vn ny c
th tm c trong nhng ti liu chuyn v l thuyt mt m. Trong chng tip theo,
ta s quay v vi l thuyt mt m kho cng khai khi nghin cu cc ng cong
elliptic.
Cng vi s pht trin ca mt m kho cng khai, c l s n lc bn cnh a ch
v in thoi ca mi c quan, cng ty, cn ghi thm kho lp m ca h!

108

Bi tp v tnh ton thc hnh chng 6.


I. Bi tp
6.1. Bit rng thng bo sau y c m ho bng m Ceasar (vi kho k no
trong khong 1-29), hy tm kho v gii m:
SEMR IEIEH USSOT SLUI EIHE ITSA UIEI LUI ESYB SOSE
MRDEI EIX EIBS ORMCE SXSL GDES MBSO MTMR

V
V
n
n
M
M
a
a
tth
h
..C
C
o
o
m
m

6.2. Dng m khi m ho cu


CO CNG MAI ST CO NGAY NN KIM

vi kho ma trn l

24 22

11 10

6.3. Gii m cu sau y, bit rng n c m ho bng khi vi ma trn


8 4

17 11

OD O XC EP Y NR EY

6.4. C th lp m khi theo cch sau y. Gi s A, B l cc ma trn vung cp hai.


Qu trnh m ho c thc hin bi cng thc
C AP+B(mod 29).

Hy vit cng thc gi m v cho mt v d c th.

6.5. M ho cu sau y bng m m vi p=3137, e=31:


N NI AN TOAN

6.6. Hy gii m vn bn mt sau y, nu bit n c m ho bng m m vi


p=3137, e=31:
0206 0248

1345 2200

6.7. Chng minh rng, khi lp m RSA, nu xy ra trng hp c mt t P no


trong thng bo khng nguyn t cng nhau vi kho n=pq chn, v t ny b
pht hin, th nhn vin phn tch m c th phn tch c n ra tha s nguyn t,
v do , tm c kho gii m.
6.8. Chng minh rng, nu cc s q, p c chn ln th trng hp ri ro ni
trong bi tp 6.7 xy ra vi xc sut rt nh.
6.9. Dng kho vi n=3233, e=17 m ho cu

109

V
V
n
n
M
M
a
a
tth
h
..C
C
o
o
m
m

CHUC MNG NM MI

110

II. Thc hnh tnh ton trn my


lm gim nh cc thao tc trong vic lp m v gii m vn bn i vi m khi
v m m chng ti ch ra cch dng Maple tnh ton.
thng nht ta gi vn bn l thng bo cn chuyn c vit bng ngn ng
thng thng khng c du, P l ch trong vn bn. Bn m ho ca vn bn gi
l vn bn mt, C ch s tng ng trong vn bn mt. Gii m tc l chuyn vn
bn mt C thnh vn bn ban u P.

V
V
n
n
M
M
a
a
tth
h
..C
C
o
o
m
m

Do trong Maple khng c ch ting Vit, nn ta dng k hiu aw, aa, dd, ee, oo,
ow, uw thay cho cc ch , , , , , , tng ng.

II. 1. Thc hnh lp m v gii m khi

1. Lp m: i vi h m khi v m m, ta ng cc ch trong vn bn vi cc s,
chuyn cc s thnh h thng s khc thng qua kho lp m, sau li dng
bng tng ng cc s va tm c ta c vn bn mt cn chuyn.
a1 a 2
Gi s ta cn m ho vn bn P bng m khi 2 ch vi kho lp m l
.
a3 a4
Kho lp m c dng y l ma trn cp 2 2, nn nu s cc ch trong vn bn
P l s chn th vic m ho xy ra bnh thng nhng nu s cc ch trong vn bn
P l s l th ch cui cng ca vn bn P s khng c m ho. khc phc tnh
trng trong trng hp th 2 ta thm vo cui vn bn P mt ch m khng nh
hng n ni dung ca vn bn (chng hn ch x). Ta thc hin theo cc bc sau
y:

Bc 1: Tnh s cc ch trong vn bn P, ta thc hin bng dng lnh:


[>nops([P]);

Sau du (;) n phm Enter trn mn hnh s hin ln s cc ch ci trong vn bn


P, nu l s chn ta khng thay i P, nu l s l ta thm vo cui vn bn P
mt ch ci, v d l ch x.
Bc 2: Thit lp tng ng cc ch ci trong vn bn P (hoc l P sau khi thm
ch ci x) vi cc s thng qua dng lnh sau y:
[>L:=
subs({a=1,aw=2,aa=3,b=4,c=5,d=6,dd=7,e=8,ee=9,g=10,h=11,
i=12,k=13,l=14,m=15,n=16,o=17,oo=18,ow=19,p=20,q=21,r=22
,s=23,t=24,u=25,uw=26,v=27,x=28,y=0},[P]):
Sau du (:) n phm Enter, v y ta khng cn hin th kt qu ca dng lnh
ny nn dng du (:) thay cho du (;). Khi trn mn hnh s hin ln du nhc
([>) thc hin tip dng lnh th 3.
Bc 3: Thc hin dng lnh:
[>N:=nops(L)/2:

111

(Lnh nops(L) dng tnh s phn t ca L )


Sau du (:) n phm Enter trn mn hnh s xut hin du nhc lnh ([>), ta thc
hin tip bc 4.

V
V
n
n
M
M
a
a
tth
h
..C
C
o
o
m
m

Bc 4: Thc hin dng lnh:


[> subs
({1=a,2=aw,3=aa,4=b,5=c,6=d,7=dd,8=e,9=ee,10=g,11=h,12=i
,13=k,14=l,15=m,16=n,17=o,18=oo,19=ow,20=p,21=q,22=r,23=
s,24=t,25=u,26=uw,27=v,28=x,0=y}, [seq (msolve ({xa1*L[2*k-1]-a2*L[2*k],y-a3*L[2*k-1]-a4*L[2*k]},29),k=1..
N)]);
Sau du (;) n phm Enter trn mn hnh s hin ln cc ch tng ng ca vn
bn mt. Ta vit li theo th t thch hp s c vn bn mt cn chuyn.
Nh vy, rt n gin, m ho vn bn no ta ch cn thay cc ch ci trong
vn bn vo v tr ca P (vi ch cc ch ci phi tch bit nhau bi du (,)) trong
dng lnh th nht, th hai v thay cc s a1a2,a3,a4 ca kho lp m vo dng lnh
th t. d hiu ta theo di th d sau y:
Ch : i vi h m ny ta cho tng ng ch y vi s 0.

Th d 1: M ho cu CHUC BAN THANH CNG bng m khi vi kho lp m


23 11
l
.
9 12
Ta thc hin nh sau:

[>nops([c,h,u,c,b,a,n,t,h,a,n,h,c,oo,n,g]);
16

16 l mt s chn do ta thc hin tip dng lnh th hai m khng cn phi thm
ch vo.
[>L:=
subs({a=1,aw=2,aa=3,b=4,c=5,d=6,dd=7,e=8,ee=9,g=10,h=11,
i=12,k=13,l=14,m=15,n=16,o=17,oo=18,ow=19,p=20,q=21,r=22
,s=23,t=24,u=25,uw=26,v=27,x=28,y=0},[c,h,u,c,b,a,n,t,h,
a,n,h,c,oo,n,g]):
[> N:=nops(L)/2:

[>subs
({1=a,2=aw,3=aa,4=b,5=c,6=d,7=dd,8=e,9=ee,10=g,11=h,12=i
,13=k,14=l,15=m,16=n,17=o,18=oo,19=ow,20=p,21=q,22=r,23=
s,24=t,25=u,26=uw,27=v,28=x,0=y},
[seq
(msolve
({x23*L[2*k-1]-11*L[2*k],y-9*L[2*k-1]12*L[2*k]},29),k=1..N)]);

[{y = aa, x = b},{y = t, x = q},{y = ow, x = n},{y = uw,


x = s},{y = t, x = aa},{x = u, y = m},{y = y, x = s},{x
= l, y = aa}]
112

Vy ta c vn bn mt tng ng l B QT N S T UM SY L
Th d 2: M ho cu LY THUYT MT MA KHNG CO GI KHO bng m khi
8 4
vi kho lp m l
.
17 11
Ta thc hin nh sau:
[>nops([l,y,t,h,u,y,ee,t,m,aa,t,m,a,k,h,oo,n,g,c,o,g,i,k
,h,o]);
25
25 l mt s l do ta phi thm mt ch x vo trong vn bn P.

V
V
n
n
M
M
a
a
tth
h
..C
C
o
o
m
m

[>L:=subs({a=1,aw=2,aa=3,b=4,c=5,d=6,dd=7,e=8,ee=9,g=10,
h=11,i=12,k=13,l=14,m=15,n=16,o=17,oo=18,ow=19,p=20,q=21
,r=22,s=23,t=24,u=25,uw=26,v=27,x=28,y=0},[l,y,t,h,u,y,e
e,t,m,aa,t,m,a,k,h,oo,n,g,c,o,g,i,k,h,o,x]):
[> N:=nops(L)/2:

[>subs({1=a,2=aw,3=aa,4=b,5=c,6=d,7=dd,8=e,9=ee,10=g,11=
h,12=i,13=k,14=l,15=m,16=n,17=o,18=oo,19=ow,20=p,21=q,22
=r,23=s,24=t,25=u,26=uw,27=v,28=x,0=y},[seq (msolve ({x8*L[2*k]-4*L[2*k-1],y-17*L[2*k]-11*L[2*k1]},29),k=1..N)]);

[{x = v, y = ee}, {x = g, y = n}, {x = k, y = l}, {x =


u, y = l},{x = uw, y = k}, {x = k, y = uw}, {x = q, y =
y}, {x = l, y = q},{y = v, x = x}, {x = h, y = u}, {x =
p, y = t}, {y = h, x = t},{x = aw, y = u}]
Vy ta c vn bn mt tng ng l V GN KL UL K K QY LQ XV HU PT TH
U
2. Gii m: Gi s ta nhn c vn bn mt C, cn gii m C vi kho ma trn
a1 a 2
A=
ta thc hin trnh t tng bc nh sau:
a3 a4
Bc 1: Lp tng ng mi ch ci trong vn bn mt vi mt s bng dng lnh
nh qu trnh lp m.
[>L:=
subs({a=1,aw=2,aa=3,b=4,c=5,d=6,dd=7,e=8,ee=9,g=10,h=11,
i=12,k=13,l=14,m=15,n=16,o=17,oo=18,ow=19,p=20,q=21,r=22
,s=23,t=24,u=25,uw=26,v=27,x=28,y=0},[C]):
Sau du (:) n phm Enter trn mn hnh s xut hin du nhc lnh ([>), ta thc
hin tip bc 2.
Bc 2: Thc hin dng lnh:
[>N:=nops(L)/2:
113

(Lnh nops(L) dng tnh s phn t ca L )


Sau du (:) n phm Enter trn mn hnh s xut hin du nhc lnh ([>), ta thc
hin tip bc 3.
Bc 3: Thc hin dng lnh:
[>subs
({1=a,2=aw,3=aa,4=b,5=c,6=d,7=dd,8=e,9=ee,10=g,11=h,12=i
,13=k,14=l,15=m,16=n,17=o,18=oo,19=ow,20=p,21=q,22=r,23=
s,24=t,25=u,26=uw,27=v,28=x,0=y}, [seq (msolve ({L[2*k1]-a1*x-a2*y,L[2*k]-a3*x-a4*y},29),k=1..N)]);

V
V
n
n
M
M
a
a
tth
h
..C
C
o
o
m
m

Sau du (;) n phm Enter trn mn hnh s hin ln cc ch tng ng ca vn


bn. Ta vit li theo th t thch hp s c vn bn ban u.
Nh vy, rt n gin, gii m vn bn mt C khi bit kho ma trn A ta ch cn
thay cc ch ci trong vn bn mt vo v tr ca C (vi ch cc ch ci phi tch
bit nhau bi du (,)) trong dng lnh th nht v thay cc s a1a2,a3,a4 ca kho lp
m vo dng lnh. d hiu ta theo di th d sau y:
Th d: Gii m vn bn mt B QT N S T UM SY L bng m khi vi kho
23 11
lp m l
.
9 12
Ta thc hin nh sau:

[>L:=subs({a=1,aw=2,aa=3,b=4,c=5,d=6,dd=7,e=8,ee=9,g=10,
h=11,i=12,k=13,l=14,m=15,n=16,o=17,oo=18,ow=19,p=20,q=21
,r=22,s=23,t=24,u=25,uw=26,v=27,x=28,y=0},[b,aa,q,t,n,ow
,s,uw,aa,t,u,m,s,y,l,aa]):
[> N:=nops(L)/2:

[>subs
({1=a,2=aw,3=aa,4=b,5=c,6=d,7=dd,8=e,9=ee,10=g,11=h,12=i
,13=k,14=l,15=m,16=n,17=o,18=oo,19=ow,20=p,21=q,22=r,23=
s,24=t,25=u,26=uw,27=v,28=x,0=y}, [seq (msolve ({L[2*k1]-23*x-11*y,L[2*k]-9*x-12*y},29),k=1..N)]);

[{y = h, x = c}, {y = c, x = u}, {y = a, x = b}, {y = t,


x = n},{x = h, y = a}, {y = h, x = n}, {y = oo, x = c},
{y = g, x = n}]
Nh vy ta c vn bn l CHUC BAN THANH CNG.

II. 2. Thc hnh lp m v gii m m


1. Lp m: i vi h m m, ta ng cc ch trong vn bn vi cc s, chuyn cc
s thnh h thng s khc thng qua kho lp m, sau li dng bng tng
ng cc s va tm c ta c vn bn mt cn chuyn. Gi s p l mt s nguyn
t l ( m bo tnh an ton s p c chn y phi l s nguyn t tng i

114

ln, chng hn ln hn 2929), e l kho lp m (trong (e,p-1)=1). chuyn mt


vn bn cho i tng c kho lp m l (e,p) tin hnh lp m theo cc bc sau:
Bc 1: Tm m (l s nguyn ln nht sao cho mi s tng ng vi m ch ci u
nh hn p). Ta thc hin dng lnh sau:
[>Lp:=length(p);
Sau du (;) n phm Enter trn mn hnh s hin ln kt qu.
Nu kt qu l mt s l th ly m =(Lp-1)/2.
Nu kt qu l mt s chn xt p=2929...29 trong s ch s ca p bng s ch
s ca p, ta thc hin tip dng lnh:

V
V
n
n
M
M
a
a
tth
h
..C
C
o
o
m
m

[>p-p;

Sau du (;) n phm Enter trn mn hnh s hin ln kt qu. Nu kt qu l mt s


dng ly m=Lp/2. Nu kt qu l mt s m ly m=(Lp-2)/2.
Bc 2: t tng ng mi ch trong vn bn P vi mt s, ta dng dng lnh sau:
[>subs({a=1,aw=2,aa=3,b=4,c=5,d=6,dd=7,e=8,ee=9,g=10,h=1
1,i=12,k=13,l=14,m=15,n=16,o=17,oo=18,ow=19,p=20,q=21,r=
22,s=23,t=24,u=25,uw=26,v=27,x=28,y=29},[P]);
Sau du (:) n phm Enter trn mn hnh s hin ra kt qu.

Bc 3: Chia cc s tng ng tm c thnh tng nhm m s, trong cc s


1,2,3,4,5,6,7,8,9 thay bi 01,02,03,04,05,06,07,08,09. Nu nhm cui cng cha
m s th ta thm vo cc s 28 (tng ng vi ch x). Cng vic ny ta thc hin
bng tay v n n gin. Ri tm cc ch tng ng trong vn bn mt. Ta thc hin
dng lnh:
[>L:=[nhm th nht, nhm th hai,...]:seq (msolve (xL[k]&^e,p),k=1..nops(L));
Sau du (:) n phm Enter trn mn hnh s hin ra kt qu.

Nh vy, rt n gin, m ho vn bn no ta ch cn thay cc ch ci trong


vn bn vo v tr ca P (vi ch cc ch ci phi tch bit nhau bi du (,)) trong
dng lnh bc 2, v thay cc s p,e ca kho lp m vo cc dng lnh bc 1,
bc 3. d hiu ta theo di th d sau y:
Th d: Vi kho lp m l p=2938,e=31. M ho thng bo sau:
I HA NI NGAY
Ta thc hin nh sau:
[>length(2839);
4
4 l mt s chn do ta phi thc hin tip lnh th vi p=2929
[> 2939-2929;

115

10
10 l mt s dng do ly m=4/2=2.
[>subs({a=1,aw=2,aa=3,b=4,c=5,d=6,dd=7,e=8,ee=9,g=10,h=1
1,i=12,k=13,l=14,m=15,n=16,o=17,oo=18,ow=19,p=20,q=21,r=
22,s=23,t=24,u=25,uw=26,v=27,x=28,y=29},[dd,i,h,a,n,oo,i
,n,g,a,y]);
[7, 12, 11, 1, 16, 18, 12, 16, 10, 1, 29]
[>
L:=[0712,1101,1618,1216,1001,2928]:seq(msolve
(L[k]&^31-x,2939),k=1..nops(L));

V
V
n
n
M
M
a
a
tth
h
..C
C
o
o
m
m

{x = 898}, {x = 1853}, {x = 1003}, {x = 2156}, {x =


1786}, {x = 2614}
Vy ta c vn bn mt l

898 1853 1003 2156 1786 2614.

2. Gii m: Khi nhn c mt vn bn mt C gi cho mnh, ta dng kho gii m


(d,n) ca mnh tm ra ni dung nhn c. Ta thc hin cc dng lnh nh sau:
Bc 1: Tm li khi P (tng ng bng cc s) trong vn bn, ta dng dng lnh:
[>L:=[C]: seq (msolve (x-L[k]&^d,p),k=1..nops(L));

Ch khi thay C vo trong dng lnh ny th cc khi phi cch nhau bi (,). Sau
du (;) n phm Enter trn mn hnh s hin ln khi cc s tng ng ca P.
Bc 2: nhn c P ta tch mi khi ca P nhn c thnh cc nhm c hai
s ri tng ng mi nhm vi mt ch ci. Ta thc hin nh sau:
[>P:=[P]:subs({1=a,2=aw,3=aa,4=b,5=c,6=d,7=dd,8=e,9=ee,
10=g,11=h,12=i,13=k,14=l,15=m,16=n,17=o,18=oo,19=ow,20=p
,21=q,22=r,23=s,24=t,25=u,26=uw,27=v,28=x,29=y},[seq((P[
i] mod 29),i=1..nops(P))]);
Ta xt th d sau:

Th d: Hy gii m vn bn mt 898 1853 1003 2156 1786 2614 bit kho gii
m l (853,2939).
Ta thc hin nh sau:

[>L:=[898,1853,1003,2156,1786,2614]:seq(msolve(xL[k]&^853,2939),k=1..nops(L));

{x = 712}, {x = 1101}, {x = 1618}, {x = 1216}, {x =


1001}, {x = 2928}

[>P:=
[07,12,11,1,16,18,12,16,10,1,29,28]:subs({1=a,2=aw,3=aa,
4=b,5=c,6=d,7=dd,8=e,9=ee,10=g,11=h,12=i,13=k,14=l,15=m,
116

16=n,17=o,18=oo,19=ow,20=p,21=q,22=r,23=s,24=t,25=u,26=u
w,27=v,28=x,0=y},[seq((P[i] mod 29),i=1..nops(P))]);

[dd, i, h, a, n, oo, i, n, g, a, y, x]
Vy vn bn nhn c l I HA NI NGAY.

II. 3. Thc hnh lp m v gii m RSA


Qu trnh ny c thc hin tng t i vi h m m, ta ch cn thm vo ch k
ca mnh. i vi h m ny, kho lp m l (e,n) trong n l tch ca hai s
nguyn t ln, kho gii m l (e,d). Ta xt th d sau y:

V
V
n
n
M
M
a
a
tth
h
..C
C
o
o
m
m

1. Lp m:
Th d : Linh c kho lp m l (19,221), Lan c kho lp m l (13,1457). Linh
mun gi cho Lan li nhn sau: Anh mun gp em, Linh. Anh ta thc hin nh
sau:
Bc 1: ng mi ch trong li nhn vi mt s, thc hin bng dng lnh:

[>subs({a=1,aw=2,aa=3,b=4,c=5,d=6,dd=7,e=8,ee=9,g=10,h=1
1,i=12,k=13,l=14,m=15,n=16,o=17,oo=18,ow=19,p=20,q=21,r=
22,s=23,t=24,u=25,uw=26,v=27,x=28,y=29},[a,n,h,m,u,oo,n,
g,aw,p,e,m,l,i,n,h]);
Sau khi n phm Enter ta nhn c kt qu:

[1, 16, 11, 15, 25, 18, 16, 10, 2, 20, 8, 15, 14,
12, 16, 11]
Bc 2: Linh k tn ca mnh, trong qu trnh ny Linh dng n kho gii m ca
mnh l (91,221)
[>L:=[14,12,16,11]:seq(msolve(L[k]&^91x,221),k=1..nops(L));

{x = 27}, {x = 142}, {x = 16}, {x = 80}

Bc 3: Thc hin m ho cc s nhn c, k c ch k ca Linh

[>L:=[1,16,11,15,25,18,16,10,2,20,8,15,14,12,16,11,27,14
2,16,80]:seq(msolve (L[k]&^13-x,1457),k=1..nops(L));

{x = 1}, {x
= 237},{x =
1287}, {x =
= 207},{x =

= 252}, {x = 207}, {x = 1360}, {x = 862}, {x


252}, {x = 226}, {x = 907}, {x = 1002}, {x =
1360}, {x = 679}, {x = 1040}, {x = 252}, {x
1207}, {x = 330}, {x = 252}, {x = 919}

Vy Linh s gi cho Lan li nhn


1
252 207
252
226 907
1002 1287 1360 679
1207 330 252
919
117

1360 862
1040 252

237
207

2. Gii m:
Th d : Khi nhn c li nhn Lan s gii m theo cc bc sau: (kho gii m
ca Lan l (637,1457):
[>L:=[1,252,207,1360,862,237,252,226,907,1002,1287,1360,
679,1040, 252, 207,1207,330,252,919]:seq(msolve
(L[k]&^637-x,1457),k=1..nops(L));

V
V
n
n
M
M
a
a
tth
h
..C
C
o
o
m
m

{x = 1}, {x = 16}, {x = 11}, {x = 15}, {x = 25}, {x =


18}, {x = 16},{x = 10}, {x = 2}, {x = 20}, {x = 8}, {x =
15}, {x = 14},{x = 12}, {x = 16}, {x = 11}, {x = 27}, {x
= 142}, {x = 16},{x = 80}

[>P:=[1,16,11,15,25,18,16,10,2,20,8,15,14,12,16,11,27,14
2,16,80]:subs({1=a,2=aw,3=aa,4=b,5=c,6=d,7=dd,8=e,9=ee,1
0=g,11=h,12=i,13=k,14=l,15=m,16=n,17=o,18=oo,19=ow,20=p,
21=q,22=r,23=s,24=t,25=u,26=uw,27=v,28=x,0=y},[seq((P[i]
mod 29),i=1..nops(P))]);

[a, n, h, m, u, oo, n, g, aw, p, e, m, l, i, n, h, v,


uw, n, r]
Lan gii ra c li nhn l Anh mun gp em Linh v n r, nh vy ngi nhn l
Linh v 4 ch cui l ch k ca Linh. kim tra xem c ng tht hay khng, Lan
thc hin tip cc dng lnh:
[>L:=[27,142,16,80]:seq(msolve(L[k]&^19x,221),k=1..nops(L));

{x = 14}, {x = 12}, {x = 16}, {x = 11}

[>P:=[14,12,16,11]:subs({1=a,2=aw,3=aa,4=b,5=c,6=d,7=dd,
8=e,9=ee,10=g,11=h,12=i,13=k,14=l,15=m,16=n,17=o,18=oo,1
9=ow,20=p,21=q,22=r,23=s,24=t,25=u,26=uw,27=v,28=x,0=y},
[seq((P[i] mod 29),i=1..nops(P))]);

[l, i, n, h]
Vy ngi gi ng l Linh.

118

Chng 7

ng cong elliptic
1 nh ngha.

V
V
n
n
M
M
a
a
tth
h
..C
C
o
o
m
m

Chng ny nhm trnh by nhng khi nim c bn ca mt i tng rt quan


trng ca l thuyt s v hnh hc i s: cc ng cong elliptic. V mt lch s, cc
ng cong elliptic xut hin ln u tin trong cc nghin cu v tch phn elliptic
(t c tn gi ca ng cong). Cc ng cong ny c mt trong nhiu lnh vc
khc nhau ca ton hc v n rt phong ph v mt cu trc. Mt mt, l ng
cong khng k d, tc l cc a tp mt chiu. Mt khc, cc im ca ng cong
lp thnh mt nhm Aben. V th hu nh mi cng c ca ton hc u c p
dng vo nghin cu ng cong elliptic. Ngc li, nhng kt qu v ng cong
elliptic c ngha quan trng i vi nhiu vn khc nhau. Xin ch ra mt vi v
d. V mt l thuyt, nh l ln Fermat c chng minh (trong cng trnh ca A.
Wiles) bng cch chng minh gi thuyt Taniyama-Weil v cc ng cong elliptic.
V mt ng dng, rt gn y, cc ng cong elliptic c dng trong vic xy
dng mt s h mt m kho cng khai.
c th trnh by tng i su v ng cong elliptic, chng ta cn nhiu hiu
bit v hnh hc i s. Bi vy, chng ti ch c th cp y nhng khi nim
c bn nht. Mc ch ca chng ch l lm th no c gi c th hnh dung l
do ti sao ng cong elliptic li c nhiu ng dng nh vy. Mt khc, chng ti
cng gii thiu s lc mt vi thut ton lin quan n ng cong elliptic trn
trng hu hn. Trong khi trnh by, cng ging nh cc phn khc ca cun sch,
chng ti lun c gng dng ngn ng s cp nht c th. Bi vy, i khi phi b
qua chng minh. c gi no quan tm su hn v cc ng cong elliptic, c th
tm c trong cc ti liu [Ha], [Sil].
nh ngha 7.1. ng cong elliptic trn trng K l tp hp cc im (x,y) tho
mn phng trnh
y2+a1xy+a3y=x3+a2x2+a4x+a6,

(7.1)

vi mt im O gi l im ti v cng (s ni r v sau). Hn na, phng trnh


(7.1) phi tho mn iu kin khng k d, tc l, nu vit n di dng F(x,y)=0 th
ti mi im (x,y) tho mn phng trnh, c t nht mt trong cc o hm ring
F / x , F / y khc 0.
iu kin khng k d ni trn tng mg vi iu kin, nu xt tp hp cc im
ni trn nh mt ng cong, th ng cong khng c im bi. Nh vy, nu
biu din y2 nh l mt a thc bc 3 ca x, th a thc khng c nghim bi.
Ch rng, phng trnh trn y khng duy nht: trong nhiu trng K, c th tm
c dng ti thiu ca phng trnh biu din ng cong.

118

Nu ta xt phng trnh (7.1) vi cc h s trong Z, th v Z c th nhng vo trong


mi trng K tu nn c th xt phng trnh trn nh l phng trnh trong trng
K. Mt iu cn lu ngay: phng trnh c th tho mn iu kin khng k d
i vi trng ny, nhng li khng tho mn iu kin i vi trng khc.
Chng hn, nu trng ang xt c c trng 2 th ta c (x2)=0 vi mi x!
im ti v cng ni trong nh ngha l im v cng trong ng cong x nh
tng ng. Ta xt khng gian x nh P2, tc l khng gian m cc im l cc lp
tng ng ca cc b ba (x,y,z), trong x, y, z khng ng thi bng 0, v b ba
(x,y,z) tng ng vi b ba ( x, y, z), 0. Nh vy, nu z 0 th lp
tng ng ca (x,y,z) cha b ba (x/z,y/z,1). Ta c th ng nht mt phng x nh
P2 vi mt phng thng thng (aphin) cng vi cc im ti v hn ng vi z=0.

V
V
n
n
M
M
a
a
tth
h
..C
C
o
o
m
m

Mt ng cong trong mt phng thng thng c th tng ng vi ng cong


trong mt phng x nh bng cch thm vo cc im ti v cng. lm vic ,
trong phng trnh xc nh ng cong, ta ch cn thay x bi x/z, y bi y/z v nhn
hai v ca phng trnh vi mt lu tha thch hp ca z kh mu s.
V d. ng cong elliptic vi phng trnh (7.1) c thm vo cc im ti v
cng c ng cong tng ng trong khng gian x nh:
y2z+a1xyz+a3yz2=x3+a2x2z+a4xz2+a6z3,

(7.2)

nh l sau y cho ta thy c th nh ngha php cng cc im trn ng cong


elliptic trang b cho n cu trc nhm Aben.
nh l 7.2. Xt ng cong elliptic xc nh trn trng tu bi phng trnh
y2+a1xy+a3y=x3+a2x2+a4x+a6,

(7.3)

Ta c th trang b cho tp hp cc im ca ng cong cu trc nhm Aben cng


tnh nh sau:
-Phn t 0 l im ti v cng; (0,1,0).

-im vi to (x1,y1) c nghch o l im vi to ( x1,-y1-a1x1-a3).

- Nu hai im P1=(x1,y1) v P2 = (x2,y2) khng phi l nghch o ca nhau th


P1+P2=P3, P3 =(x3,y3)xc inh nh sau.
t

m=

y1 y 2

x1 x 2

, nu P1 P2 ;

m=

3 x1 + 2 a 2 x1 + a 4 a 1 x1
, nu P1=P2.
2 y1 + a1 x1 + a 3

v tnh x3,y3 theo cng thc


x3=-x1-x2-a2+m (m+a1),
y3=-y1-a3-a1x3+m (x1-x2)
119

Chng minh. Bng tnh ton trc tip da vo phng trnh xc nh ng cong, d
kim tra nh ngha php cng trn y tho mn cc tin ca nhm Aben.
thy r ngha hnh hc ca nh ngha php cng trn y, ta xt trng hp
quan trng sau y ca cc ng cong elliptic trn trng thc R.

2. ng cong elliptic trn trng thc. Trc tin, ta c nhn xt sau y. Trong
nhng trng vi c trng khc 2 v 3, phng trnh (7.1) c th a v dng
Y2=4X3+c4X+c6.

(7.4)

Tht vy, ch cn dng php i bin:

V
V
n
n
M
M
a
a
tth
h
..C
C
o
o
m
m

Y=2y+a1x+a3

X=x+(a12+4a2)/12

n gin, ta thng dng dng sau y, gi l dng Weierstrass ca ng cong:


y2=x3+a4x+a6.

Trong trng hp ny, bit thc ca ng cong l

=-16(4a43+27a62)

Nh vy, iu kin ng cong khng c k d (khng c im bi)l:


4a43+27a62 0.

Ta s s dng dng Weierstrass ca ng cong. Bng tnh ton trc tip ta cc


im theo cng thc cho trong nh l 7.2, ta c th thy lut cng trong nhm lp
bi cc im ca ng cong c m t hnh hc sau y:
Nu cc im P v Q ca ng cong c to x khc khc nhau th ng thng i
qua P v Q s ct ng cong ti mt im th ba. im i xng vi giao im
qua trc honh chnh l dim P+Q.
Trong trng hp P v Q c cng honh , tung ca chng s l cc gi tr i
nhau, v P,Q l hai im i xng nhau qua trc honh. Khi ng thng i qua
P,Q s ct ng cong ti v cng: chnh l im 0 ca nhm cng cc im,
v P,Q l cc phn t nghch o ca nhau.
R rng cng P vi 0, thc hin bng cch ni P vi im ti v cng bng ng
thng song song vi trc tung s ct ng cong ti im i xng vi P qua trc
honh, v nh vy P+0=P.
Trn hnh 1 ta minh ho nhng iu va ni trn qua v d ng cong vi phng
trnh y2=x3-x.
V cc im ca ng cong l cc phn t ca mt nhm cng Aben, ta s dng k
hiu NP ch phn t nhn c bng cch cng lin tip N ln im P.

120

nh ngha 7.3. im P ca ng cong c gi l im bc hu hn nu tn ti


s nguyn dng N sao cho NP=0. S N nh nht tho mn iu kin gi l bc
ca P.

V
V
n
n
M
M
a
a
tth
h
..C
C
o
o
m
m

D nhin khng phi mi im ca ng cong u c bc hu hn.

Hnh 1. ng cong elliptic y2=x3-x trn trng thc

3. ng cong elliptic trn trng cc s hu t.

Trong rt nhiu vn ca Hnh hc i s v s hc, ta thng phi lm vic vi


cc ng cong trn trng s hu t. l cc ng cong cho bi phng trnh
(7.2), trong cc h s l cc s hu t, v ta cng ch xt cc im vi to l
cc s hu t. Nghin cu ng cong elliptic trn trng s hu t cng c ngha l
nghin cu tp hp nghim hu t ca phng trnh (7.2), mt vn quan trng ca
s hc. Trong phn cui chng, ta s thy rng, vn ny cn lin quan n chng
minh nh l ln Fermat.
Gi s E l ng cong elliptic cho. Ta k hiu qua E(Q) tp hp cc im c to
hu t. Nh ta thy, tp hp ny c cu trc nhm Aben. Cc im bc hu
hn ca nhm Aben E(Q) lp thnh nhm con E(Q)tors, gi l nhm con xon ca
E(Q). Khi , E(Q) s l tng trc tip ca E(Q)tors vi nhm con cc im bc v
hn. nh l ni ting ca Mordell ni rng nhm con cc im bc v hn ch c
hu hn phn t sinh, v do ng cu vi nhm Zr, trong r l mt s nguyn
khng m. S r gi l hng ca ng cong, v l mt c trng ht sc quan trng,
cha nhiu thng tin s hc v ng cong. Chng minh cc kt lun ny i hi
phi s dng nhiu kin thc su sc v hnh hc i s,v do vt ra ngoi
khun kh ca cun sch. Ta hn ch y pht biu ca nh l Mordell.
nh l (Mordell). Gi s E l mt ng cong elliptic trn Q. Khi tp hp cc
im ca E vi to hu t E(Q) l mt nhm Aben hu hn sinh. Ni cch khc,
ta c:
E(Q)= E(Q)tors Zr,
trong r l mt s nguyn khng m.

121

Nhm con xon cc im bc hu hn ca mt ng cong c th tnh c khng


kh khn lm, trong khi hng r li ht sc kh xc nh. Thm ch, ngay i vi mt
ng cong c th, ch ra r bng 0 hay khc 0 cng l mt iu ht sc kh khn. Ta
c th thy ngay rng, nu r=0 th ng cong ang xt ch c hu hn im hu t,
trong trng hp r 0, tn ti v hn im hu t trn ng cong. iu tng
ng vi vic phng trnh cho c hu hn hay v hn nghim hu t, mt bi
ton kh ca s hc.
Trong 5, ta s thy rng, bi ton tm im hu t ca ng cong elliptic lin quan
n vic thnh lp nhng h mt m kiu mi, cng nh cc thut ton khai trin
nhanh s nguyn cho trc thnh tha s nguyn t. l nhng ng dng gn y
nht ca l thuyt ng cong elliptic vo cc vn thc tin.

V
V
n
n
M
M
a
a
tth
h
..C
C
o
o
m
m

Nh ni trn, vic xc nh nhm con xon ca ng cong elliptic khng phi


l kh khn. Tuy nhin, vic ch ra tt c cc kh nng ca cc nhm con (ch tn
ti 15 kh nng khc nhau) li l mt bi ton kh, v mi c gii quyt nm
1977 bng nh l ni ting sau y ca B. Mazur.
nh l Mazur. Gi s E l ng cong elliptic trn trng Q. Khi nhm con
xon ca E(Q) ng cu vi mt trong 15 nhm sau y :
Z/mZ, trong 1 m 10, hoc m=12.
Z/2ZxZ/2mZ, vi 1 n 4

Nh vy, nhm xon ca ng cong elliptic c khng qu 16 phn t.

4. ng cong elliptic trn trng hu hn.

chng minh mt phng trnh (h s nguyn) no khng c nghim nguyn,


mt trong nhng phng php thng c dng l nh sau. Ta xt phng trnh
mi, nhn c t phng trnh cho bng cch thay cc h s ca n bi cc
thng d modulo mt s p no . Nu phng trnh ny khng c nghim (ng d
modulo p) th phng trnh xut pht cng khng c nghim. Vic lm c gi
l sa theo modulo p. R rng rng phng trnh mi n gin hn phng trnh
cho, hn na, xt nghim ng d modulo p, ta ch cn th vi hu hn gi tr.
Nu phng trnh cho qu tht v nghim, th trong trng hp chn s p mt
cch may mn, ta c th i n kt lun kh d dng.
Khi nghin cu cc ng cong elliptic, c bit l cc ng cong trn trng s
hu t, ngi ta cng thng dng phng php tng t: sa theo modulo p. Vic
lm n n cc ng cong trn trng hu hn.
Ta cn lu ngay mt iu. Khi sa mt ng cong elliptic bng cch chuyn
cc h s thnh cc ng d modulo p, ta c th nhn c mt ng cong c k d.
Tht vy, bit thc ca ng cong (khc khng) c th dng d 0 modulo p, v khi
, ng cong nhn c c im bi trn trng hu hn. Tuy nhin, r rng iu
ch xy ra khi p l mt c s ca bit thc ca ng cong xut pht, v do ,
ch xy ra vi mt s hu hn gi tr ca p. Ta ni ng cong elliptic cho c sa
xu ti nhng gi tr ca p , v c sa tt ti nhng gi tr p khc.

122

iu cn quan tm u tin khi nghin cu mt ng cong elliptic trn trng hu


hn l: ng cong c bao nhiu im? Gi s E l ng cong elliptic trn
trng Fq c q phn t. Cc im ca ng cong l cc cp (x,y), x, y Fq tho mn
phng trnh trong Fq:
y2=x3+a4x+a6
Nh vy, nu vi gi tr x, x3+a4x+a6 l mt thng d bnh phng modulo q th s
c hai im (x,y) v (x,-y) thuc ng cong. Trong trng hp ngc li, khng c
im no ca ng cong ng vi gi tr x. T , khi q l s nguyn t, theo nh
ngha ca k hiu Legendre, s im ca ng cong ng vi gi tr x l

V
V
n
n
M
M
a
a
tth
h
..C
C
o
o
m
m

x 3 + a4 x + a6
1+

Thm im ti v cng, ta c cng thc tnh s im ca ng cong trong trng


hp q l s nguyn t:

x 3 + a4 x + a6
#E(Fq)=1+ 1 +

q
x Fq

Trong trng hp q khng phi l s nguyn t, trong cng thc trn y, thay cho
k hiu Legedre, ta hiu l k hiu Jacobi, v du ng thc c thay th bi bt
ng thc .
nh l trn y cho ta mt c lng ca s im ca ng cong E trn trng Fq.
nh l Hasse. Gi s N l s im ca ng cong elliptic xc nh trn trng Fq.
Khi ta c:
|N-(q+1) | 2 q .

Bn c c th tm thy chng minh ca nh l ny trong [Sil].

Mt trong nhng ng dng mi nht ca ng cong elliptic trn trng hu hn,


xut hin trong nhng nm gn y, l cc h mt m kho cng khai elliptic. Phn
tip theo c dnh trnh by vn .

5. ng cong elliptic v h mt m kho cng khai.

5.1. H mt m kho cng khai s dng ng cong elliptic da trn phc tp


ca thut ton tm s nguyn x sao cho xB=P, trong P, B l cc im cho trc
ca ng cong (nu s nh th tn ti). Ch rng, cc im ca ng cong lp
thnh mt nhm, v ta c th quan nim xB nh l Bx: bi ton ny hon ton
tng t nh bi ton tm logarit c s b ca mt s p cho trc (xem chng 6).
Trc tin, ta cn xt thut ton tm bi ca mt im trn ng cong.

123

nh l 7.4. Cho E l mt ng cong elliptic trn trng hu hn Fq, P l mt im


ca ng cong. Khi c th tnh to ca im kP bng O(log klog3 q) php tnh
bit.
Trc khi i vo chng minh nh l, ta tm hiu s qua phng php rt thng
thng tm bi ca cc im trn ng cong: phng php nhn i lin tip.
Xt v d sau: gi s cn tnh 205P. Ta vit :
205P=2(2(2(2(2(2(2P+P)+P)+P)+P))+P)
Nh vy, vic tnh 205P c a v 4 php cng hai im ca ng cong v 7
php nhn i mt im cho trc.

V
V
n
n
M
M
a
a
tth
h
..C
C
o
o
m
m

Ta gi thit rng, trng Fq c c trng khc 2, 3. Trong trng hp q=2r hoc


a=3r, c nhng thut ton nhanh hn tnh to ca cc bi ca mt im cho
trc. Nh vy, phng trnh xc nh ng cong c th cho di dng Weierstrass:
y2=x3+ax+b.

Khi , theo nh l 7.2, tng P+Q=(x3,y3) ca hai im khc nhau P=(x1,y1) v


Q=(x2,y2) c tnh theo cng thc sau:
x3= (

y 2 y1 2
) -x1-x2,
x 2 x1

y3=-y1+(

y 2 y1
)(x1-x3).
x 2 x1

(7.6)

(7.7)

Trong trng hp P=Q, ta c cng thc tnh 2P:

3x1 2 + a 2
x3= (
) -2x1,
2 y1

(7.8)

y3=-y1+ (

3x1 + a
) (x1-x3).
2 y1

(7.9)

Nh vy, ta phi dng khng qu 20 php nhn, chia, cng, tr tnh to ca


tng hai im khi bit cc to ca cc im . S cc php tnh bit i hi l
O(log3 q) (xem chng 5). Khi dng phng php nhn i lin tip, ta phi thc
hin O(log k) php tnh cng hai dim hoc nhn i mt im (xem chng 5). Nh
vy, ton b s php tnh bit phi dng l O(log klog3 q). nh l c chng minh.
Tm li, ta c thut ton thi gian a thc tnh bi ca mt im. Ngc li, khi
bit kP v P, vic tm ra k vi nhng thut ton nhanh nht hin nay li i hi thi
gian m. iu ny hon ton tng t nh trong trng hp cc s m modulo p, v
s l c s cho vic xy dng h kho cng khai s dng ng cong elliptic.

5.2. M ho nh cc im ca ng cong elliptic trn trng hu hn.

124

5.2.1. Nh thy trong chng 6, vic chuyn thng bo mt thc hin bng cch
chuyn n thnh dng ch s, m ho thng bo ch s ny v chuyn i. V th,
n gin khi trnh by, ta s xem thng bo cn chuyn l mt s nguyn dng
m no .
Vic u tin l phi chn mt ng cong elliptic E no trn trng hu hn Fq.
Sau , phi tm cch tng ng s nguyn m vi mt im ca ng cong E.
d hiu qu trnh lp m, ta s xem ng cong E c chn. Vic chn
ng cong s c trnh by tit sau.
5.2.2. Tng ng mt s m vi mt im ca ng cong elliptic.

V
V
n
n
M
M
a
a
tth
h
..C
C
o
o
m
m

Cho n nay, cha c mt thut ton quyt nh no hu hiu tm c mt s


ln cc im ca ng cong elliptic. Thut ton m ta trnh by sau y l mt
thut ton xc sut vi thi gian a thc.
Trc ht ta chn mt s k no theo yu cu sau: trng hp thut ton s tin
hnh khng cho kt qu mong mun ch xy ra vi xc sut khng vt qu 2-k. Nh
vy, ni chung k=40 l c th chp nhn c (ta nhc li rng, trong trng hp ,
xc sut sai lm ca mt thut ton s b hn xc sut sai lm ca phn cng ca
my tnh).
Gi s s m nm trong khong 1 m M. Ta lun chn q sao cho q>Mk. Trc tin,
ta tng ng mi s nguyn dng s khng vt qu M vi mt phn t ca trng
hu hn Fq. Vic d dng lm c bng cch sau y. Gi s q=pr, v s s biu
din di c s p c dng s=(c0,c1,...,cr-1)p. Khi , a thc
r 1

S(X)= ci X i
i =0

modulo mt a thc bt kh quy no bc r s tng ng vi mt phn t ca


trng Fq (xem Chng 5).
Nh vy, vi m cho, vi mi j, 1 j k, ta c mt phn t tng ng xj ca trng
Fq. Ta s ch ra mt thut ton , vi xc sut rt ln, tm c mt xj trong s
sao cho tn ti im (xj,yj) trn ng cong E. Khi , ta tng ng s m vi im
Pm=(xj,yj) E va tm c.
Thut ton tm Pm:
El1. t j 1.

El2. Nu j>k: kt thc thut ton. Trong trng hp


ngc li, t Yj xj3+axj+b. Nu tn ti yj sao cho
Yj yj2(mod q), in ra Pm=(xj,yj) v kt thc thut ton. Nu
ngc li, chuyn sang bc El3.
El3. t j j+1 v quay v bc El2.
V mi phn t x Fq, xc sut f(x) l chnh phng bng 1/2, nn thut ton trn
y cho ta tm ra im Pm vi xc sut tht bi l 1/2k.

125

Nh vy, ta c mt thut ton m ho m bng cch tng ng n vi mt im


ca ng cong elliptic E. Tuy nhin, cn nhc li rng, mt trong nhng yu cu
ca m ho l khi bit ng cong E trn Fq, bit Pm, ta phi khi phc c m mt
cch d dng. Trong trng hp ny, yu cu c m bo. Tht vy, gi s
x 1
(trong [ ] l k hiu phn nguyn).
Pm=(x,y). Khi m=
k
5.3. Mt m kho cng khai s dng ng cong elliptic.
Trong chng 6, ta lm quen vi mt h m kho cng khai, trong s dng
phc tp ca php tnh tm logarit c s b modulo p. y, ta c khi nim hon
ton tng t.

V
V
n
n
M
M
a
a
tth
h
..C
C
o
o
m
m

Gi s B,P l cc im ca ng cong elliptic E, k l mt s nguyn v P=kB. Khi


ta ni k l logarit c s B ca P. Trong trng hp E l ng cong trn trng
Fq, q=pr, p 2, bi ton tm logarit ca cc im trn mt ng cong i hi thi
gian m, v do , khng th thc hin c trong khong thi gian chp nhn c
(nu q c chn ln).
By gi gi s c mt tp hp n c th cn trao i thng tin mt vi nhau:
A1,A2,...,An.
Trc tin, ta chn mt ng cong elliptic E trn trng hu hn Fq vi mt im
B E dng lm c s. Nhng thng tin ny c thng bo cng khai. D nhin q
phi l s ln.
Sau , mi c th Aj chn cho mnh kho ej, l mt s nguyn no . Kho ny
c gi b mt, nhng Aj thng bo cng khai phn t ejB. iu ny khng lm l
kho ej do phc tp ca php tnh logarit.
Gi s Aj cn gi thng bo mt m cho Ai. Trc tin, m c tng ng vi im
Pm E nh trnh by trn. Sau , Aj s chn ngu nhin mt s s v chuyn cho
Ai cp im sau: (sB, Pm+s(eiB)), nh eiB c cng khai. Khi nhn c cp
im ny, Ai ch vic ly s sau tr i ei ln s trc nhn c Pm:
Pm=Pm+ s(eiB)- ei(sB).

Ch rng, ch c Ai lm c iu ny v ei c gi b mt, v s s khng th tm


thy trong thi gian chp nhn c mc d bit sB, v l logarit ca (sB) c s B.
Trong h m va trnh by, ta khng cn bit s N ca ng cong E.

5.4. H m tng t m m.
Trong trng hp ny, cc c th chn chung cho mnh mt ng cong elliptic E
trn trng hu hn Fq vi N im. Cc tham s ny c thng bo cng khai.
xy dng h m, mi c th Ai chn cho mnh kho ei, l s nguyn dng nm
gia 1 v N, sao cho (ei,N)=1. Bng thut ton Euclid, Ai tm c di tho mn

126

diei 1(mod N). By gi, gi s Ai cn gi thng bo m cho Aj. Cng nh trc y,


Ai tm im Pm tng ng trn ng cong. Sau ,
1) Bc 1: Ai gi cho Aj thng bo eiPm. D nhin, khi nhn c thng bo ny, Aj
cha th gii m v khng bit ei v di.
2) Bc 2: Aj nhn thng bo c vi ej v gi tr li cho Ai thng bo ej(eiPm).
3) Bc 3: Ai li gi cho Aj thng bo sau khi nhn vi di: di ej(eiPm).
4) Nhn c thng bo cui cng ny, Aj nhn n vi kho dj ca mnh nhn
c Pm=djdieiejPm. Do cch chn ei, di, ej, dj ta c: djdieiej 1(mod N), tc l
P=(1+sN)Pm vi s nguyn s no . V N l s im ca ng cong nn NPm=0,
v nh vy P=Pm. Aj nhn c thng bo ban u.

V
V
n
n
M
M
a
a
tth
h
..C
C
o
o
m
m

rng, trong mi bc trn y, cc kho mt ei,di ca cc c th khng h b


pht hin.

5.5. Chn ng cong elliptic.

C nhiu cch chn ng cong v im B dng lm c s khi lp m. y, ta


trnh by hai cch i theo hai hng ngc nhau. Th nht, chn mt im v mt
ng cong c th. Th hai, ly mt ng cong trn trng s hu t v sa theo
modulo p khc nhau thu c cc ng cong trn trng hu hn.
Chn ng cong v im ngu nhin. Ta lun lun gi thit rng, c trng ca
trng Fq khc 2, 3 (nhng trng hp ny c th xt ring). Khi , phng trnh
ca ng cong c th vit di dng (7.2).
Gi s x, y, a l ba phn t ly ngu nhin ca trng Fq. Ta t b=y2-(x3+ax). C
th kim tra d dng a thc x3+ax+b c nghim bi hay khng (xt bit thc
4a2+27b3). Nu a thc khng c nghim bi, ta c ng cong E cho bi phng
trnh
Y2=X3+aX+b

v im B=(x,y) E. Nu a thc c nghim bi, ta lm li vi mt s a ngu nhin


khc.
Sa theo modulo p. Ta xut pht t mt ng cong elliptic E no trn trng s
hu t, v chn B E l mt im bc v hn. Sau , ta ly mt s nguyn t p
ln no . Nh ni, ng cong chn ch c sa xuvi mt s hu hn s
nguyn t. V th, nu p chn ln th sa theo modulo p s cho ta ng cong
elliptic E modulo p v im B modulo p. Cui cng, cng ch l, cho n nay,
cha c mt thut ton no tng i tt xc nh s im N ca mt ng cong
elliptic trn trng hu hn Fq vi q l s rt ln. Trong trng hp N l tch cu
nhng s nguyn t b, c nhng thut ton c bit tm logarit c s B, v do
, h m m chng ta xt s khng gi c tnh bo mt na. Tuy nhin, c
nhiu phng php xc sut trnh xy ra tnh trng s im N ca ng cong l
tch ca nhng s nguyn t b.

127

6. L-hm ca ng cong elliptic


6.1. Nh ni u chng, cc ng cong elliptic c vai tr rt quan trng trong
nhiu vn ca s hc. Tit ny c mc ch lm cho c gi hnh dung c phn
no ngha ca ng cong elliptic trong Hnh hc i s s hc. Thc ra, y l
mt lnh vc rt phong ph ca ton hc hin i. V th, kh c th trnh by trong
mt cun sch, hn na, li trong mt gio trnh vi yu cu kh s cp. Chng ti
c gng la chn y nhng kt qu v khi nim c bn nht, v cch trnh by l
m t ch khng i vo chi tit.

V
V
n
n
M
M
a
a
tth
h
..C
C
o
o
m
m

C th ni, khi nim quan trng nht trong nghin cu ng cong elliptic l
L-hm. Gi s ta xt ng cong elliptic trn trng s hu t Q. Nu cn thit th
kh mu s cc h s ca phng trnh xc nh ng cong, ta c th gi thit
ngay t u rng, ng cong c cho bi phng trnh vi cc h s nguyn.
nghin cu ng cong cho trn trng s hu t, ngi ta nghin cu ng thi
cc sa theo modulo p ca ng cong ng vi mi s nguyn t p. Ta nhc li
rng, l cc ng cong nhn c bng cch thay cc h s bi cc thng d
modulo p ca chng. C th tn ti mt s hu hn s nguyn t p ti ng cong
nhn c c im bi. Trc ht, ta xt cc s nguyn t p ti ng cong c
sa tt, tc l ta c ng cong elliptic trn trng Fp.

6.2. L-hm ca ng cong elliptic trn trng hu hn.

Gi s E l ng cong elliptic trn trng s hu t Q, c sa tt ti s nguyn t p.


ng thi vi vic xt E modulo p, ta c th xt cc im ca ng cong E trn
mi trng Fq, vi q=pr, r=1,2,...K hiu qua Nr s im ca ng cong E trn Fq,
q=pr.
Nh vy, ta c dy cc s nguyn dng N1,N2,...Nr... nghin cu mt dy no
, mt trong nhng phng php rt hay c dng trong s hc l xt hm sinh
ca chng. Nh hm sinh, ngi ta c th xt cc phn t ca dy mt cch ng
thi, thng qua tnh cht ca hm sinh. Chng hn, hm sinh ca dy s trn y l:
Zp(T)=exp(
r 1

Nr r
T )
r

(7.10)

nh ngha. Hm Zp(T) c gi l Zeta-hm ca ng cong E trn trng Fp.


Zeta-hm c xy dng khng ch vi cc ng cong elliptic, m cn vi nhng
i tng rng hn, l cc a tp x nh. Zeta-hm ca cc a tp x nh c nhiu
tnh cht tng t vi Zeta-hm Riemann. Mt trong nhng tnh cht quan trng
nht ca cc Zeta-hm th hin trong nh l sau y, m ta ch pht biu cho cc
ng cong elliptic.
nh l Weil. Zeta-hm ca mt ng cong elliptic E trn trng hu hn Fq l mt
hm hu t ca T, c dng:

128

Z(T;E/Fq)=

1 aT qT 2
,
(1 T )(1 qT )

trong a l s tham gia trong cng thc tnh s im ca ng cong E trn Fp:
N1=1+q-a. nh thc ca a thc t s m, v hai nghim (phc lin hp) ca n
c tr tuyt i bng q .
Nhn xt. 1) Khi bit Zeta-hm, ta c th khai trin tm cc h s ca n trong
cng thc (7.10), ngha l bit c s im ca E trn trng F p r vi mi r tu .
V Zeta-hm ch ph thuc a=1+q-N1 nn Nr xc nh duy nht qua N1.

V
V
n
n
M
M
a
a
tth
h
..C
C
o
o
m
m

2) Tnh cht nh thc ca t s m c ngha l: |a|<2 q . Nh vy, nh l Hasse l


mt h qu ca nh l Weil.
Mt tng t ca nh l Weil cho cc a tp x nh c gi l gi thuyt Weil,
v c P. Deligne chng minh nm 1973.

6.3. L-hm ca ng cong trn trng s hu t

Nh vy, vi mi s nguyn t p, ta c Zeta-hm Zp(T) ng vi ng cong elliptic


E trn trng Fq, q=pr. nghin cu ng cong E trn trng s hu t, ta c th
xt ng thi cc Zeta-hm a phng Zp(T) bng cch xy dng Zeta-hm
ton cc ca bin phc s.
K hiu qua ap s xc nh bi ap=p+1-Np, trong Np l s im ca ng cong
trn trng Fp. Khi L-hm Hasse-Weil ca ng cong E c nh ngha bi
cng thc
L(E,s)= (1 a p p s + p 1 2 s ) 1 .
p

Nhn xt. Bng cch khai trin tch trong nh ngha L-hm, ta c th thy rng, Lhm tng t nh Zeta-hm Riemann, nh ngha bi cng thc sau y:

( s) = n s .
n =1

D thy rng, Zeta-hm Riemann c th c tnh bi cng thc sau (xem phn bi
tp):

( s) = (1 p s ) 1
pP

i vi Zeta-hm Riemann, cc tnh cht quan trng nht l:


1) ( s) c th thc trin thnh hm phn hnh trn ton mt phng phc vi cc im
n ti s=1.
2) Nu t

129

( s) = s / 2 ( s / 2) ( s)
th ( s) l hm phn hnh trn ton mt phng phc, v tho mn phng trnh hm

( s) = (1 s)
3) ( 2n) =0 vi mi n nguyn dng.
Gi thuyt Riemann ni ting ni rng, cc khng im cn li ca Zeta-hm u
nm trn ng thng Re s=1/2. Ngi ta kim tra gi thuyt i vi mt s
rt ln khng im (hng triu), nhng vn cha chng minh c gi thuyt trong
trng hp tng qut. Gi thuyt ny cng lin quan n nhiu vn ca s hc
thut ton.

V
V
n
n
M
M
a
a
tth
h
..C
C
o
o
m
m

i vi L-hm ca ng cong elliptic, ta c:

Gi thuyt: Hm L(E,s) c th thc trin gii tch ln ton mt phng phc. Hn


na, tn ti mt s nguyn dng N sao cho, nu t

( E , s) = N s / 2 (2 ) s ( s) L( E , s),

th ta c phng trnh hm sau y

( E ,2 s) = ( E , s) .

S N ni trong gi thuyt l mt bt bin quan trng ca ng cong, gi l


conuct ca n.

6.4. Gi thuyt Birch-Swinnerton-Dyer.

Mt trong nhng gi thuyt quan trng khc ca l thuyt cc ng cong elliptic l


gi thuyt sau y ca Birch v Swinnerton-Dyer.
Trc tin ta nhc li rng, nhm cc im hu t ca ng cong elliptic E l tng
trc tip ca nhm cp hu hn vi nhm Zr. S r c gi l hng ca ng cong.
Gi thuyt Birch-Swinnerton-Dyer. Nu hng ca ng cong elliptic E bng r th
L-hm ca ng cong c khng im cp r ti im s=1.
Nh vy, nu L(E,1)=0 th hng r 1, v do , ng cong elliptic c v hn im
hu t. Trong trng hp ngc li, ng cong E ch c hu hn im hu t.
chnh l kt qu quan trng u tin theo hng khng nh gi thuyt BirchSwinnerton-Dyer, c Coates v Wiles chng minh nm 1977.
Thc ra, gi thuyt Birch-Swinnerton-Dyer cn cho cng thc tnh gii hn

lim( s 1) r L( E , s).
s1

Tuy nhin, pht biu chnh xc gi thuyt ta cn n khi nim nhm


Shafarevich ca ng cong, l mt trong nhng khi nim su sc nht ca hnh
hc i s.

130

Nhn xt. kt thc chng ny, chng ti xin ni qua vi li v gi thuyt quan
trng nht trong l thuyt ng cong elliptic: gi thuyt Taniyama-Weil.
Gi s N l mt s nguyn dng. Ta k hiu qua nhm 0(N) nhm cc ma trn
a b
vung cp 2 =
trong a,b,c,d nguyn, ad-bc=1 v c 0(mod N). Nhm
c d
cc ma trn ny tc ng ln na mt phng trn theo cng thc sau:

a b
az + b
.
(z)=

cz + d
c d

V
V
n
n
M
M
a
a
tth
h
..C
C
o
o
m
m

Mt hm f(z) gii tch ti na mt phng trn, k c ti cc im hu t ca trc thc


v bng khng ti cc im , c gi l mt dng modula trng s 2 i vi
nhm 0(N) nu n tho mn h thc sau:
f( z)=(cz+d)2f(z).

T nh ngha trn suy ra rng, nu f(z) l dng modula trng s 2 i vi nhm


1 1
0(N) th ta c f(z+1)=f(z) vi mi z (ly l ma trn
. Nh vy, f(z) c th
0 1
khai trin theo dng sau:

f ( z ) = a n e 2 i n z .
n =1

T ta c th tng ng f vi L-hm ca n:

L f ( s) = a n / n s
n =1

Gi thuyt Taniyama-Weil: Nu E l mt ng cong elliptic trn trng s hu t


cnuct N th L-hm ca ng cong E l L-hm ca mt dng modula trng s 2
i vi nhm 0(N).
Gi thuyt trn y lin quan cht ch n nh l ln Fermat. Tht vy, gi s tn ti
s nguyn t p ln hn 2 sao cho phng trnh Fermat vi s m p c cc nghim
khng tm thng a, b, c.
Ta i du c v vit phng trnh di dng:

ap+bp+cp=0
ng cong elliptic xc nh bi phng trnh
y2=x(x-ap)(x+bp)
c cnuct N=N0(abc) (xem nh ngha N0 chng 5). ng cong ny c
G. Frey nghin cu ln u tin nm 1983. Sau (1986), K. Ribet chng minh
rng, L-hm ca ng cong khng phi l L-hm ca bt k mt dng modula
trng s 2 no i vi nhm 0(N). Nh vy, nu chng minh c gi thuyt

131

Taniyama-Weil th cng chng minh c nh l ln Fermat, bi v nu phng


trnh Fermat c nghim th tn ti mt ng cong elliptic khng tho mn gi
thuyt Taniyama-Weil.
Thng 6 nm 1993, A.Wiles cng b chng minh gi thuyt Taniyama-Weil, cng
tc l chng minh c nh l ln Fermat.

V
V
n
n
M
M
a
a
tth
h
..C
C
o
o
m
m

Bi tp chng 7

7.1. Cho dng cong elliptic trn trng thc y2=x3-36x v cc im trn ng
cong: P=(-3,9), Q=(-2,6). Hy tnh cc im P+Q v 2P.
7.2. Tm bc ca im P=(2,3) trn ng cong y2=x3+1.

7.3. Chng minh rng cc ng cong elliptic sau y c q+1 im trn trng Fq:
1) y2=x3-x, q 3(mod 4).

2) y2=x3-1, q 2(mod 3), q l.

3) y2+y=x3, q 2(mod 3), (q c th chn).

7.4. Tnh s im ca ng cong elliptic y2=x3-x trn trng F71.

7.5. Cho ng cong y2+y=x3 trn trng F2. Hy tm Zeta-hm ca ng cong


v tnh s im ca ng cong trn mi trng Fq vi q=2r, r=1,2,...

132

You might also like